Shankar CSP23T05S

You might also like

Download as pdf or txt
Download as pdf or txt
You are on page 1of 141

.

PRESTORMING – 2023
TEST 5
SCIENCE AND TECHNOLGOGY- EXPLANATION
INDEX
Q.1) .................................................................................................................................... 4
Q.2) .................................................................................................................................... 5
Q.3) .................................................................................................................................... 6
Q.4) .................................................................................................................................... 8
Q.5) .................................................................................................................................... 9
Q.6) .................................................................................................................................. 11
Q.7) .................................................................................................................................. 12
Q.8) .................................................................................................................................. 13
Q.9) .................................................................................................................................. 14
Q.10) ……. ........................................................................................................................... 15
Q.11) .................................................................................................................................. 16
Q.12) .................................................................................................................................. 17
Q.13) .................................................................................................................................. 18
Q.14) .................................................................................................................................. 19
Q.15) .................................................................................................................................. 21
Q.16) .................................................................................................................................. 21
Q.17) .................................................................................................................................. 23
Q.18) .................................................................................................................................. 25
Q.19) .................................................................................................................................. 27
Q.20) .................................................................................................................................. 29
Q.21) .................................................................................................................................. 30
Q.22) .................................................................................................................................. 31
Q.23) .................................................................................................................................. 34
Q.24) .................................................................................................................................. 35
Q.25) .................................................................................................................................. 36
Q.26) .................................................................................................................................. 38
Q.27) .................................................................................................................................. 40
Q.28) .................................................................................................................................. 41
Q.29) .................................................................................................................................. 43
Q.30) .................................................................................................................................. 44

 1
.


Q.31) .................................................................................................................................. 46
Q.32) .................................................................................................................................. 47
Q.33) .................................................................................................................................. 47
Q.34) .................................................................................................................................. 48
Q.35) .................................................................................................................................. 49
Q.36) .................................................................................................................................. 50
Q.37) .................................................................................................................................. 52
Q.38) .................................................................................................................................. 54
Q.39) .................................................................................................................................. 54
Q.40) .................................................................................................................................. 56
Q.41) .................................................................................................................................. 57
Q.42) .................................................................................................................................. 59
Q.43) .................................................................................................................................. 62
Q.44) .................................................................................................................................. 63
Q.45) .................................................................................................................................. 65
Q.46) .................................................................................................................................. 66
Q.47) .................................................................................................................................. 67
Q.48) .................................................................................................................................. 69
Q.49) .................................................................................................................................. 71
Q.50) .................................................................................................................................. 73
Q.51) .................................................................................................................................. 74
Q.52) .................................................................................................................................. 76
Q.53) .................................................................................................................................. 78
Q.54) .................................................................................................................................. 79
Q.55) .................................................................................................................................. 80
Q.56) .................................................................................................................................. 81
Q.57) .................................................................................................................................. 83
Q.58) .................................................................................................................................. 84
Q.59) .................................................................................................................................. 85
Q.60) .................................................................................................................................. 86
Q.61) .................................................................................................................................. 87
Q.62) .................................................................................................................................. 88
Q.63) .................................................................................................................................. 90
Q.64) .................................................................................................................................. 91
Q.65) .................................................................................................................................. 92
 2
.


Q.66) .................................................................................................................................. 94
Q.67) .................................................................................................................................. 96
Q.68) .................................................................................................................................. 96
Q.69) .................................................................................................................................. 98
Q.70) ................................................................................................................................ 100
Q.71) ................................................................................................................................ 101
Q.72) ................................................................................................................................ 103
Q.73) ................................................................................................................................ 103
Q.74) ................................................................................................................................ 104
Q.75) ................................................................................................................................ 105
Q.76) ................................................................................................................................ 106
Q.77) ................................................................................................................................ 108
Q.78) ................................................................................................................................ 110
Q.79) ................................................................................................................................ 112
Q.80) ................................................................................................................................ 114
Q.81) ................................................................................................................................ 115
Q.82) ................................................................................................................................ 116
Q.83) ................................................................................................................................ 117
Q.84) ................................................................................................................................ 118
Q.85) ................................................................................................................................ 120
Q.86) ................................................................................................................................ 122
Q.87) ................................................................................................................................ 124
Q.88) ................................................................................................................................ 125
Q.89) ................................................................................................................................ 127
Q.90) ................................................................................................................................ 128
Q.91) ................................................................................................................................ 129
Q.92) ................................................................................................................................ 131
Q.93) ................................................................................................................................ 132
Q.94) ................................................................................................................................ 133
Q.95) ................................................................................................................................ 134
Q.96) ................................................................................................................................ 135
Q.97) ................................................................................................................................ 137
Q.98) ................................................................................................................................ 139
Q.99) ................................................................................................................................ 140
Q.100) ................................................................................................................................ 141
 3
.


Q.1) Consider the following statements:


1. Biosimilar drugs are created in living cells that are highly similar to a reference product.
2. Process used to develop generic medicines cannot be applied to the development of biosimilar
medicines.
3. Biosimilars and generic drugs are versions of brand-name drugs used as affordable treatment
options to patients.
Which of the statements given above is/are correct?
(a) 1 and 2 only
(b) 2 and 3 only
(c) 1 and 3 only
(d) 1, 2 and 3
EXPLANATION:

A biosimilar is a biologic drug (biological product) that is "similar" to another biologic medicine that
generally comes from living organisms (living cells of yeast, bacteria, or animal cells) and is highly
similar to the reference product.
A biologic (biological product), or biologic, is a medicine created in. The drug substance is created or
derived from a living organism using recombinant DNA technology. So, statement 1 is correct.
The generic drug is made with chemicals. Conversely, biosimilars (biologics) are from a biological
(natural) source.
While identical generic versions of small molecules can be chemically synthesized, it is impossible to
create identical versions of reference biologic medicines , due to their complexity. Therefore, the
processes used to develop generic medicines cannot be applied to developing biosimilar medicines. So,
statement 2 is correct

A generic drug is an exact copy of a brand-name drug. Generic medicines/drugs work the same as
brand-name medicines.
Biosimilars are versions of brand-name biologics that may offer more affordable treatment options to
patients and are similar to generic drugs. So, statement 3 is correct.

ADDITIONAL INFORMATION:

Biologics ¾ Biologics (also known as biological products) include various products such as
vaccines, monoclonal antibodies, blood components, allergenic, gene therapy,
tissues, and proteins.
¾ They treat various diseases and conditions, such as cancer, kidney, and
autoimmune diseases.
¾ Biological products are regulated by the Food and Drug Administration (FDA)
and are used to diagnose, prevent, treat, and cure diseases and medical
conditions

“Highly A manufacturer developing a proposed biosimilar demonstrates that its product is


Similar” highly similar to the reference product by extensively analyzing (i.e., characterizing)
the structure and function of both the reference product and the proposed
biosimilar.

 4
.


Interchangeable ¾ An interchangeable product is a biosimilar product that meets additional


Product requirements outlined by the Biologics Price Competition and Innovation Act.
To fulfill these additional requirements, information must show that an
interchangeable product is expected to produce the same clinical result as the
reference product in any given patient.

Q.2) Consider the following statements with reference to BS VI emissions norms:


1. In BS VI vehicles the maximum permissible Nitrous oxide emission levels for Petrol vehicles is
more compared to the diesel vehicles.
2. BS VI fuel has reduced sulphur content of 100 ppm.
3. Pollution emissions standards in India are fixed and regulated by the central pollution control
board.
4. Unlike BS IV, BS VI norms deals with selective catalytic reduction and diesel particulate filter.
Which of the statements given above is/are correct?
(a) 1 and 2 only
(b) 2, 3 and 4
(c) 3 and 4
(d) 1, 3 and 4

EXPLANATION:
Bharat Stage (BS) norms are the standards set up by the Indian government which specify the amount
of air pollutants from internal combustion engines, including those that vehicles can emit. BS–VI
compliant vehicles will reduce the emission of hydrocarbons and nitrogen oxides for petrol and diesel
engines. BS-VI compliant petrol vehicles will reduce nitrogen oxide emissions by 25% and diesel
vehicles by approximately 70 %. Thus, permissible Nitrous oxide emission levels for Petrol vehicles is
less compared to the diesel Vehicles. So, Statement 1 is not correct.

Sulphur is a pollutant necessary for lubricating the car's engine. BS-VI fuel will contain a lower
percentage of sulphur. It will be reduced from 50 ppm parts per million (ppm) in BS-IV fuel to 10 ppm
in BS-VI fuel. So, Statement 2 is not correct.

The standards and the timeline of the BS – VI norms are set and regulated by the Central Pollution
Control Board, which is under the Ministry of Environment, Forest and Climate Change. So,
Statement 3 is correct.

Unlike BS IV, BS-VI emissions norms enabled the introduction of advanced emission control
technologies, including Diesel Particulate Filters (DPF) to reduce Particulate Matter (PM) and Selective
Catalyst Reduction (SCR) systems for reduction in Nitrogen Oxides (NOx) emissions in BS-VI vehicles.
So, Statement 4 is correct.

ADDITIONAL INFORMATION:

CENTRAL POLLUTION CONTROL BOARD


About It is a statutory organisation constituted in September 1974 under the Water
(Prevention and Control of Pollution) Act 1974. Further, CPCB was entrusted with

 5
.


the powers and functions under the Air (Prevention and Control of Pollution) Act,
1981.

Functions ¾ Advise the Central Government on any matter concerning preventing and
controlling water and air pollution and improving the quality of air.
¾ Plan and cause to be executed a nationwide programme for the prevention,
control or abatement of water and air pollution;
¾ Co-ordinate the activities of the State Board and resolve disputes among them;
¾ Provide technical assistance and guidance to the State Boards
¾ Plan and organise training of persons engaged in the programme on the
prevention, control or abatement of water and air pollution;
¾ Organise through mass media, a comprehensive mass awareness programme on
the prevention, control or abatement of water and air pollution;
¾ Collect, compile and publish technical and statistical data relating to water and
air pollution and the measures devised for their effective prevention, control or
abatement;
¾ Prepare manuals, codes and guidelines relating to the treatment and disposal of
sewage and trade effluents as well as for stack gas cleaning devices, stacks and
ducts;
¾ Disseminate information in respect of matters relating to water and air pollution
and their prevention and control;
¾ Lay down, modify or annul, in consultation with the State Governments
concerned, the standards for stream or well, and lay down standards for the
quality of air; and
¾ Perform such other functions as may be prescribed by the Government of India.

Q.3) Consider the following statements :


1. Shale gas is a conventional hydrocarbon present in the permeable rock.
2. Guar gum plays a significant role in the extraction of shale gas.
3. India is the largest producer of Guar gum in the world.
Which of the statement given above is/are correct ?
(a) 1 only
(b) 1 and 2 only
(c) 2 and 3 only
(d) 1, 2 and 3
EXPLANATION:
Conventional hydrocarbons can be extracted easily from permeable rocks, but Shale gas is an
unconventional hydrocarbon because the rock it is extracted from acts as the source, reservoir, and
cap rock and is trapped under low-permeable rocks. The gas is produced, stored, and sealed within
impermeable shale and can be released only after the shale is drilled and artificially fractured during
an extraction process. It requires pressurized water or chemicals to break the low-permeable rocks and
extract them. So, Statement 1 is not correct.
¾ The Guar gum is actually in powder form, is made by grinding guar seeds, and has unique
thickening, emulsifying and binding properties.
 6
.


¾ The shale gas industry uses the gum in fracking—a process where a mixture of water (95 percent),
Sand (4.5 percent), and guar gum (0.5 percent) is injected under high pressure into an oil-or gas-
bearing rock to fracture it. The gum's viscous property decreases fluid loss and friction, reducing
energy consumption and increasing gas or oil recovery. So, Statement 2 is correct.

The consumption pattern of guar seeds is largely influenced by the de mands of the
petroleum industry. India accounts for 80 percent of the world’s guar produce, of which 72
percent comes from Rajasthan. About 90 percent of guar gum processed in India is exported.
So, Statement 3 is correct.

ADDITIONAL INFORMATION:

NATURAL GAS

About ¾ Natural gas primarily consists of methane (50 to 90 % volume by


methane), the simplest hydrocarbon.
¾ It also contains small amounts of heavier gaseous hydrocarbons such
as ethane (C2H6), propane (C3H8), and butane (C4H10) and also small
amounts of highly toxic hydrogen sulphide (H2S)
Conventional natural ¾ Conventional natural gas lies above most reservoirs of crude oil. These
gas deposits are trapped under the permeable rocks and can be
tapped/used only through the pipeline. But the natural gas that comes
with the oil is often looked at as an unwanted by product and burned
off.
¾ Burning of associated natural gas represents a waste of a valuable
energy resource and emissions carbon dioxide into the atmosphere
from its burning.
¾ But after the gas is processed, it is piped or compressed into cylinders
for use by consumers. This gas is also used for the production of
petrochemicals and fertilizers.

Unconventional ¾ Unconventional natural gas resources are located where the


natural gas permeability and porosity are so low that the resource cannot be
extracted economically through a vertical well bore and instead
requires a horizontal well bore followed by multistage hydraulic
fracturing to achieve economical production.
¾ Shale gas and coal bed methane are examples of unconventional
natural gas

 7
.


Q.4) Consider the following statements:


1. Eukaryotic cell is a free-floating genetic material with the absence of a nucleus.
2. Prokaryotes can live with or without oxygen while eukaryotes live only with oxygen.
Which of the statements given above is/are correct ?
(a) 1 only
(b) 2 only
(c) Both 1 and 2
(d) Neither 1 nor 2
EXPLANATION:
The name “Eukaryotic” itself means: eu = true, karyon = nucleus. They contain an organised nucleus
with a nuclear envelope. The DNA is enclosed in a nuclear membrane forming a nucleus. So,
Statement 1 is not correct.

Prokaryotes and eukaryotes both may perform both aerobic (oxygen-requiring) and anaerobic (non-
oxygen-based) metabolism. So, both of them can live with or without oxygen. So, Statement 2 is not
correct.

 8
.


ADDITIONAL INFORMATION:
PROKARYOTIC CELLS AND EUKARYOTIC CELLS

Prokaryotic Cells Eukaryotic Cells


¾ The prokaryotic cells are represented by ¾ The eukaryotes include all protists, plants, animals
bacteria, blue-green algae, mycoplasma and fungi.
and PPLO (Pleuro Pneumonia-like ¾ In eukaryotic cells, there is an extensive
Organisms). They are generally smaller compartmentalisation of cytoplasm through
and multiply more rapidly than the membrane-bound organelles. Eukaryotic cells
eukaryotic cells possess an organised nucleus with a nuclear
¾ They may vary greatly in shape and size. envelope.
¾ Prokaryotes have a cell wall ¾ Eukaryotic cells have a variety of complex locomotory
surrounding the cell membrane, except and cytoskeletal structures
in mycoplasma. ¾ Their genetic material is organised into
¾ The genetic material is naked, not chromosomes.
enveloped by a nuclear membrane.

Q.5) Which of the below pesticides are banned by the Government of India for its manufacture, import
and use?
1. Aldrin
2. Endosulfan
3. Methyl Parathion
4. Caprolactam
5. Trifluralin
6. Factamfos
Choose the correct answer using the codes given below :
(a) 1, 3, 4, 5, 6
(b) 1, 2, 3 and 5
(c) 1, 3, 4, 5
(d) 1, 2, 3, 4, 5 and 6
EXPLANATION:
OPTION ELIMINATION STRATEGY
The Government of India produces caprolactam and Factamfos. By this, we can eliminate options (a),
(c) and (d)

 9
.


The Fertilisers And Chemicals Travancore Limited (FACT) is a Public Sector company under the
administrative control of the Department of Fertilizers, Ministry of Chemicals & Fertilizers, Government
of India. The main products of FACT are
¾ FACTAMFOS
¾ AMMONIUM SULPHATE
¾ CAPROLACTAM

The government of India banned several pesticides for manufacture, import and use in India. Some of
them include
¾ Aldrin
¾ Methyl Parathion
¾ Trifluralin.
¾ Endosulfan
¾ Methyl Parathion etc.
So, Option (b) is correct.

ADDITIONAL INFORMATION:

PESTICIDES BANNED IN INDIA

Recently In ¾ The Supreme court has directed the Kerala government to give compensation for
News Endosulfan pesticide exposure victims.
¾ The Supreme Court of India banned the manufacture, sale, and use of Endosulfan
toxic pesticides in 2011.

The ¾ to regulate the import, manufacture, sale, transport, distribution and use of
Insecticides insecticides to prevent risk to humans or animals.
Act Of 1968 ¾ The term pesticide covers insecticides, fungicides, herbicides, rodenticides,
molluscicides, nematicides, plant growth regulators and others
¾ In India, pesticides are regulated under the Insecticides Act of 1968 and
Insecticides Rules of 1971.)
¾ The Central Insecticides Board (CIB), under the Ministry of agriculture and farmers
welfare, is the nodal agency regulating pesticides in India.

Pesticides / 1. Alachlor 24. Trichlorfon


Formulations 2. Aldicarb 25. Lindane
Banned In 3. Aldrin 26. Linuron
India 4. Benzene Hexachloride 27. Maleic Hydrazide
5. Benomyl 28. Menazon
6. Calcium Cyanide 29. Methoxy Ethyl Mercury Chloride
7. Carbaryl 30. Methyl Parathion
8. Chlorbenzilate 31. Metoxuron
9. Chlordane 32. Nitrofen
10. Chlorofenvinphos 33. Paraquat Dimethyl Sulphate
11. Copper Acetoarsenite 34. Pentachloro Nitrobenzene (PCNB)
12. Diazinon 35. Pentachlorophenol
13. Dibromochloropropane 36. Phenyl Mercury Acetate
14. Dichlorovos 37. Phorate

 10
.


15. Dieldrin 38. Phosphamidon


16. Endosulfron 39. Sodium
17. Endrin 40. Sodium Methane Arsonate
18. Ethyl Mercury Chloride 41. Tetradifon
19. Ethyl Parathion 42. Thiometon
20. Ethylene Dibromide (EDB) 43. Toxaphene(Camphechlor)
21. Fenarimol 44. Triazophos
22. Fenthion 45. Tridemorph
23. Heptachlor 46. Trichloro acetic acid

Stockholm ¾ The Stockholm Convention is a global treaty to protect human health and the
Convention environment from persistent organic pollutants (POPs).
¾ POPs are chemicals that remain intact in the environment for long periods,
accumulate in the fatty tissue of living organisms, and are toxic to humans and
wildlife.
¾ Stockholms conventions classify POPs into
x Annex A (Elimination)
x Annex B (Restriction)
x Annex C (Unintentional production)

Q.6) With reference to Raman effect, consider the following statements:


1. The Raman Effect is the very weak effect which deals with scattering of light by molecules of a
medium when they are excited to different energy levels.
2. The Raman spectroscopy is used in identifying the illegal drugs without damaging the packaging
materials.
Which of the statements given above is/are correct ?
(a) 1 only
(b) 2 only
(c) Both 1 and 2
(d) Neither 1 nor 2
EXPLANATION:

C.V. Raman had discovered the Inelastic scattering of light by molecules. He found that, the
monochromatic light is scattered when it is allowed to pass through a substance. The scattered light
contains some additional frequencies other than that of incident frequency. This is known as Raman
effect.
The phenomenon of scattering of light by colloidal particles is called the Tyndall effect.
So, statement 1 is correct

Raman Spectra is widely used in almost all branches of science. Raman Spectra of different substances
enable to classify the maccording to their molecular structure and to analyse the chemical constitution.
Raman spectroscopy identifies the contents of drugs within their packaging without damaging,
measures the composition and uniformity of drug pills, identifies street drugs, and determines drug
authenticity. So, statement 2 is correct

 11
.


Q.7) Which of the following are particulate pollutants?


1. Hydrogen sulphide
2. Mist
3. Fumes
4. Hydrocarbons
5. Smog
Select the correct answer using the code given below :
(a) 1 and 5 only
(b) 2, 3 and 5 only
(c) 2 and 5 only
(d) 1, 2, 3 and 4 only
EXPLANATION:

Particulate matter which is also called as particle pollution is the term for a mixture of solid particles
and liquid droplets found in the air.
Tropospheric pollution occurs due to the presence of undesirable solid or gaseous particles in the air.
The following are the major gaseous and particulate pollutants present in the troposphere,
¾ The Gaseous air pollutants are oxides of sulphur, nitrogen and carbon, hydrogen sulphide,
hydrocarbons, ozone, and other oxidants
¾ The Particulate pollutants are dust, mist, fumes, smoke, smog, etc.
So, Option (b) is correct.

ADDITIONAL INFORMATION:
PARTICULATE MATTER (PM)
About ¾ PM stands for particulate matter (also called particle pollution), the term for a mixture
of solid particles and liquid droplets found in the air.
¾ Some particles, such as dust, dirt, soot, or smoke, are large or dark enough to be
seen by the naked eye. Others are so small they can only be detected using an electron
microscope.
Particle pollution includes:
o PM10: inhalable particles, with diameters that are generally 10 micrometers and
smaller; and
o PM2.5: fine inhalable particles, with diameters that are generally 2.5 micrometers
and smaller.

Source Particle pollution can come from two different kinds of sources — primary and
secondary.
¾ Primary sources cause particle pollution on their own. For example, wood stoves and
forest fires are primary sources.
¾ Secondary sources let off gases that can form particles. Power plants and coal fires
are examples of secondary sources. Some other common sources of particle pollution
can be either primary or secondary — for example, factories, cars and trucks, and
construction sites.

 12
.


¾ Smoke from fires and emissions (releases) from power plants, industrial facilities, and
cars and trucks contain PM2.5.

Q.8) With reference to the Lipids in Human Body, consider the following statements :
1. Pancreas secretes lipase enzyme which converts fat into fatty acid.
2. Saturated fat consists of a double bond between carbon.
3. Very low-density lipoprotein (VLDL) that is produced in the liver is a major cause of heart attack
in human beings.
Which of the statements given above is/are correct?
(a) 1 only
(b) 2 and 3 only
(c) 1 and 3 only
(d) 1, 2 and 3
EXPLANATION:

The pancreas which secretes the enzyme lipase is critical for the digestion and absorption of dietary
fats. They break down triglycerides (fat) into free fatty acids and glycerol. Understanding the lipase
function is crucial for the pathophysiology of fat necrosis and acute and chronic pancreatitis. Also,
they play an essential role in the mechanism of some cholesterol-lowering medications. So, Statement
1 is correct.

A fatty acid is a molecule characterized by the presence of a carboxyl group attached to a long
hydrocarbon chain. Saturated fatty acids do not have any double bonds. A fatty acid is saturated when
every (single) carbon atom in the hydrocarbon chain is bonded to as many hydrogen atoms as possible
(the carbon atoms are saturated with hydrogen).
Whereas Unsaturated fatty acids can have one or more double bonds along their hydrocarbon chain.
A fatty acid with one double bond is called monounsaturated. If it contains two or more double bonds,
we say that the fatty acid is polyunsaturated. So, Statement 2 is not correct.

Very-low-density lipoprotein (VLDL) cholesterol is produced in the liver and released into the
bloodstream to supply body tissues with a type of fat (triglycerides). This is the category of "bad"
cholesterols because they can contribute to the buildup of plaque in your arteries. This buildup is
called atherosclerosis. The plaque that builds up is a sticky substance made up of fat, cholesterol,
calcium, and other substances found in the blood. Over time, the plaque hardens and narrows your
arteries. This limits the flow of oxygen-rich blood to your body. It can lead to coronary artery disease
and heart disease in human beings. VLDL level should be less than 30 mg/dL (milligrams per deciliter).
Anything higher than that puts you at risk for heart disease and stroke. So, Statement 3 is correct.

ADDITIONAL INFORMATION:

LIPIDS

About ¾ Lipids are a heterogeneous group of compounds, including fats, oils, steroids, waxes,
and related compounds, and can be divided into two major classes nonsaponifiable and
saponifiable lipids.
¾ Saturated fatty acids do not have any double bonds. Animal fats are a source of
saturated fatty acids. Unsaturated fatty acids can have one or more double bonds along
their hydrocarbon chain. Plants are the source of unsaturated fatty acids.
 13
.


¾ A fatty acid that cannot be synthesized by the body is an essential fatty acid. Linoleic
and linolenic are the two essential fatty acids (both are unsaturated). Nonessential fatty
acids can be made by the human body and so do not need to be obtained from diet
alone.
¾ Cholesterol is a sterol and an important component of cell membranes. It is also the
basis for the synthesis of other steroids, including the sex hormones estradiol and
testosterone, as well as other steroids such as cortisone and vitamin D.

Q.9) Consider the following statements with respect to Lagrange points(L-points) which is recently seen
in news :
1. At Lagrange points, the gravitational pull of two large masses precisely equals the centripetal
force required for a small object to move with them.
2. Lagrange points in space can be used by spacecraft to reduce fuel consumption.
3. The James Webb space telescope (JWST) will orbit the sun, a million miles away from Earth at
the third Lagrange point.
Which of the statements given above are correct?
(a) 1 and 2 only
(b) 2 and 3 only
(c) 1 and 3 only
(d) 1, 2 and 3
EXPLANATION:

A Lagrangian point is a position or location in space where the combined gravitational forces of two
large masses (two large bodies here may be the Earth and Sun or the Earth and Moon produce
enhanced regions of attraction and repulsion) precisely are equal to the centripetal force that is felt by
a third body which is relatively required for a smaller object to move with them. And these points can
be used by spacecraft as "parking spots" to reduce fuel consumption needed to remain in position. So,
Statements 1 and 2 are correct.

 14
.


The James Webb Space Telescope will not be in orbit around the Earth, like the Hubble Space Telescope
is - it will actually orbit the Sun, 1.5 million kilometers (1 million miles) away from the Earth at what
is called the second Lagrange point or L2 and not in the Lagrange point at L3. L2 is ideal for astronomy
because a spacecraft is close enough to readily communicate with Earth, can keep Sun, Earth, and
Moon behind the spacecraft for solar power, and (with appropriate shielding) provides a clear view of
deep space for our telescopes. So, Statement 3 is not correct.

ADDITIONAL INFORMATION:

LAGRANGE POINTS (L-POINTS)

About ¾ Lagrange points are named in honour of Italian-French mathematician Josephy-


Louis Lagrange.
¾ There are five special points where a small mass can orbit in a constant pattern with
two larger masses.
Five Among the five Lagrange points, three are unstable, and two are stable.
Lagrange
Unstable points Stable points
Points
Lagrange points labeled as L1, L2, The stable Lagrange points - labeled L4 and
and L3 - lie along the line connecting L5 - form the apex of two equilateral triangles
the two large masses. with large masses at their vertices. L4 leads
the orbit of the earth, and L5 follows.
Q.10) With reference to the ‘Seoul Forest Declaration’, which of the following statements are correct?
1. It was adopted in the CoP15 of United Nations Convention to Combat Desertification.
2. AFFIRM and SAFE are the two initiatives launched at this declaration.
3. It aims to move towards a triangular bio-economy and climate neutrality.
Choose the correct answer using the codes given below:
(a) 1 and 2
(b) 2 and 3
(c) 1 and 3
(d) 1, 2 and 3
EXPLANATION:
The Seoul Forest Declaration was adopted in the 15th World Forestry Congress, with a key focus on
how forests can help combat the multiple crises humanity faces, including climate change, biodiversity
loss, land degradation, hunger and poverty. So, Statement 1 is not correct.
Close cooperation among nations is required to address challenges beyond political boundaries. This
was strengthened at the Congress by new partnerships such as the Assuring the Future of Forests

 15
.


with Integrated Risk Management (AFFIRM) Mechanism and the Sustaining an Abundance of Forest
Ecosystems (SAFE) initiative. So, Statement 2 is correct.

¾ A circular bio-economy is a framework for using renewable natural capital to transform and manage
our land, food, health and industrial systems, to achieve sustainable well-being in harmony with
nature.
¾ Climate neutrality refers to achieving net zero greenhouse gas emissions by balancing those
emissions. Hence, they are equal to (or less than) the emissions that get removed through the
planet’s natural absorption.
¾ Seoul Forest Declaration aims to move towards a circular bio-economy and climate neutrality. So,
Statement 3 is correct.

ADDITIONAL INFORMATION:

WORLD FORESTRY CONGRESS

About ¾ The World Forestry Congress is held approximately once every six years.
¾ The first Congress was held in Italy in 1926.
¾ FAO has helped host countries organize the Congress since 1954.
¾ Responsibility for the organization and financing of each Congress lies with the
host country.

Food and ¾ The Food and Agriculture Organization (FAO) is a specialized agency of the
Agriculture United Nations that leads international efforts to defeat hunger.
Organization ¾ FAO aims to achieve food security for all and ensure that people have regular
(FAO) access to high-quality food to lead active healthy lives.
¾ It has195 members - 194 countries and the European Union.
¾ FAO released Food Price Index (FFPI) measures the monthly change in
international prices of a basket of food commodities.

Q.11) Consider the following pairs:


Commonly used Poisonous chemicals found in them
products
1. Noodles Lead

2. Air freshener formaldehyde


3. Cosmetics Phenoxyethanol

4. Whey Proteins Bisphenol-A

How many pairs given above is/are correctly matched?


(a) Only one Pair
(b) Only two pairs
(c) Only three pairs
(d) All four pairs
EXPLANATION:
Exposure to high levels of lead may cause anemia, weakness, and kidney and brain damage. Recently
the controversy around the permissible limits of lead in noodles has arisen. So, Pair (1) is correct.

 16
.


Air fresheners consist of formaldehyde, petrochemicals, p-dichlorobenzene and aerosol pollutants.


Formaldehyde is a toxic compound that may cause cancer and irritation of the throat and nose. It is a
volatile organic compound (VOC). So, Pair (2) is correct.

Phenoxyethanol is a preservative in many cosmetics and personal care products like face wash,
shampoo, etc. The incidents of bad skin reactions and nervous system interaction in infants are the
major concerns of using these cosmetics. So, Pair (3) is correct.

Bisphenol-A is a synthetic compound that is an important addition to many single-use plastics. The
whey proteins are packed in the Bisphenol-A-made containers. When these containers are stored at
room temperature, Bisphenol-A leaches into the protein powder and has health implications. So, Pair
(4) is correct.

ADDITIONAL INFORMATION:

FOOD ADULTERATION

About Food adulteration reduces the quality of food items by adding harmful substances
or subtracting valuable things from them.

Major Food product Adulterants


adulterants
Honey Sugar, glucose and sugar syrup.

Turmeric Powder Chalk powder, lead chromate and metanil yellow.

Ghee Vegetable oil, starch and vanaspati.

Red chili powder Brick powder and Artificial colors.

Pulses Artificial colors.

Rice Pebble, damaged grains and polish.

Apples Wax coating.

Salt Chalk powder

Milk Water, urea, starch, detergent and synthetic milk.


Coconut oil Palm oil, engine oils, argemone oil and paraffin.

Q.12) “Horizon scanning” recently seen in media related to :


(a) It is a technique used in diagnosis of cancer cells in the human body.
(b) It is the early detection and assessment of emerging technologies or threats that helps
policy decisions.
(c) It is a global network of synchronized radio observatories that work in unison to observe radio
sources associated with black holes.
(d) It involves scanning the soil surface to identify physical, chemical and biological characteristics
of soil.
EXPLANATION:

Horizon scanning is a technique to detect the early predictions of potentially important developments
through a systematic examination of threats and opportunities, emphasizing new technology and its
effects that are likely to have important consequences over the next decade.

 17
.


It helps in assessing whether one is adequately prepared for future changes or threats. If performed
consistently and effectively, horizon scanning, when combined with other forecasting tools, can assist
in policy making by identifying important needs or gaps. It is also an effective tool for bringing experts
in different subject areas together to discuss a common issue and develop viable solutions. So, Option
(b) is correct.

Q.13) Consider the following statements:


1. Unlike uranium, thorium alone can be directly used as nuclear fuel in a reactor.
2. Kerala has the highest monazite resources in India.
Which of the statements given above is/are correct ?
(a) 1 only
(b) 2 only
(c) Both 1 and 2
(d) Neither 1 nor 2
EXPLANATION:
Both Uranium and Thorium have got distinctive characteristics governing their utilization in nuclear
reactors. Unlike uranium, thorium alone cannot be directly used as nuclear fuel in a reactor because
it does not contain enough fissile material to initiate a nuclear chain reaction.
So utilization of Thorium is necessary either with uranium or plutonium, without going through the
second stage of Fast Breeder Reactors, to build a sufficient inventory of plutonium first, will be counter-
productive by limiting thorium utilization to a very small fraction of the total available resources in the
country. The utilization of Thorium in the third stage makes it available as a sustainable energy
resource for centuries. With this mode of utilization, Thorium offers not only a sustainable energy
resource but also excellent fuel performance characteristics in a reactor, better than Uranium with
respect to a lower inventory of long-lived nuclear waste. So, Statement 1 is not correct.

The mineral monazite is the country's major resource of rare earth minerals. As of February 2021, the
Atomic Minerals Directorate for Exploration and Research (AMD), a constituent unit of the Department
of Atomic Energy, has found a total of 12.73 million tonnes of monazite reserves in India in that, the
state Andhra Pradesh (not Kerala) has the largest monazite resource in India, having 3.78 million
tonnes of monazite. So, Statement 2 is not correct.

ADDITIONAL INFORMATION:
RARE EARTH MINERALS

About ¾ The group consists of yttrium, Scandium and the 15 lanthanide elements
(lanthanum, cerium, praseodymium, neodymium, promethium, samarium,
europium, gadolinium, terbium, dysprosium, holmium, erbium, thulium,
ytterbium, and lutetium).
¾ Rare earth elements have high electrical conductivity and are often used in
defense and modern electronics like cell phones.
¾ Rare earth elements are not as "rare" as their name implies. Thulium and
lutetium are the two least abundant rare earth elements and magnets. Still,
they each have an average crustal abundance nearly 200 times greater than
the crustal abundance of gold.

 18
.


TYPES OF NUCLEAR MATERIALS

Fissile ¾ Fissile materials are atoms that neutrons can split in a self-sustaining chain
Material reaction to release enormous amounts of energy.
¾ In nuclear reactors, the fission process is controlled, and the energy is harnessed
to produce electricity. In nuclear weapons, fission energy is released all at once
to produce an explosion.
¾ The most important fissile materials for nuclear energy and nuclear weapons are
an isotope of plutonium, plutonium-239, and an isotope of uranium, uranium-
235. Uranium-235 occurs in nature.

Fertile ¾ A material that is not itself fissile (fissionable by thermal neutrons) can be
Material converted into fissile material by irradiation in a reactor.
¾ There are two basic fertile materials: uranium-238 and thorium-232. When these
fertile materials capture neutrons, they are converted into fissile plutonium-239
and uranium-233, respectively.

Q.14) Which of the following statements given below are correct?


1. Swine flu is a zoonotic respiratory disease.
2. WHO classifies swine flu as the Neglected tropical disease.
3. Dizziness, sore throat and watery eyes are the symptoms of swine flu.
Select the correct answer using the code given below:
(a) 1 and 2 only
(b) 2 and 3 only
(c) 1 and 3 only
(d) 1, 2 and 3
 19
.


EXPLANATION:
Swine flu (H1N1) is a zoonotic disease transmitted from pigs to humans. It is a respiratory disease
caused by type A influenza viruses that regularly cause influenza outbreaks in pigs. Similar to the
regular flu, swine flu can lead to some serious illnesses like lung infection, pneumonia, and other
breathing problems. Also, the flu can make an illness even worse like diabetes or asthma. It also has
been recognized as a global health problem. So, Statement 1 is correct.

Neglected tropical diseases (NTDs) have been identified by the World Health Organization (WHO). NTDs
are found in several countries in Africa, Asia, and Latin America. They are especially common in
tropical areas where people do not have access to clean water or safe ways to dispose of human waste.
But Swine flu is not a neglected tropical disease as per the WHO classification. So, Statement 2 is
not correct.

Symptoms caused by Swine flu are,


¾ Fever
¾ Cough
¾ Runny Nose
¾ Sore Throat
¾ Body Aches
¾ Chills
¾ Fatigue
¾ Dizziness
¾ Red, watery eye
So, Statement 3 is correct.

ADDITIONAL INFORMATION:
NEGLECTED TROPICAL DISEASES

About ¾ World Health Organisation (WHO) classifies 20 diverse conditions prevalent in


tropical areas as Neglected Tropical Diseases.
¾ These diseases mostly affect impoverished communities and disproportionately
affect women and children in tropical areas.
¾ These diseases cause devastating health, social and economic consequences to
more than one billion people.
¾ These diseases are often relate\d to environmental conditions. Many of them are
spread by vectors and associated with the complex life cycle. All these factors
make their public-health control challenging.
Neglected Buruli ulcer, Chagas disease, dengue and chikungunya, dracunculiasis (Guinea-
Tropical worm disease), echinococcosis, foodborne trematodiases, human African
Diseases trypanosomiasis (sleeping sickness), leishmaniasis, leprosy (Hansen's disease),
lymphatic filariasis, mycetoma, chromoblastomycosis, and other deep mycoses,
onchocerciasis (river blindness), rabies, scabies, and other ectoparasitoses,
schistosomiasis, soil-transmitted helminthiases, snakebite envenoming,
taeniasis/cysticercosis, trachoma, and yaws and other endemic treponematoses.

 20
.


Q.15) “Danuri”, the lunar orbiter mission belongs to which one of the following ?
(a) European Union
(b) Japan
(c) China
(d) South Korea
EXPLANATION:

Danuri is the Republic of Korea's (South Korea) first exploratory space mission to travel beyond Earth's
orbit, launched in collaboration with NASA. It is also called Korean Pathfinder Lunar Orbiter, launched
to study Moon's Surface and identify potential landing sites for future missions.
The Korea Aerospace Research Institute (KARI) is the aeronautics and space agency of South Korea
who is managing the manufacturing and operation of the orbiter, and NASA supports the mission with
the development of one among the six scientific payloads as well as aiding spacecraft communications
and navigation. So, Option (d) is correct.

ADDITIONAL INFORMATION:
LUNAR MISSIONS

European ESA is working with the Canadian and Japanese space agencies to prepare the Heracles
Union robotic mission to the Moon.
Heracles is an international program to use the Gateway to the fullest and deliver samples
to scientists on Earth using new technology that is more capable and lighter than previous
missions.

Japan ¾ LUNAR-A Mission

USA ¾ The Apollo mission conducted by the United States made a soft landing on the Moon's
surface.
¾ Artemis I mission

China ¾ The Chang'e program


India ¾ Chandrayaan-1, India's first Moon mission on October 2008.
¾ Chandrayaan 2 is the second mission to the Moon developed by the ISRO, which was
successfully launched on July 22, 2019.

Q.16) With reference to Indian Navy’s Information Fusion Centre-Indian Ocean Region (IFC-IOR),
consider the following statements:
1. It was set up in 2018 to coordinate with regional countries on maritime issues and act as a
regional repository of maritime data.
2. The centre was established as part of the government’s SAGAR (Security and Growth for All in the
Region) framework for maritime co-operation in the Indian Ocean region.
3. It is located in Chennai, Tamil Nadu.
4. The centre also publishes the monthly weather forecasts and weather warnings report.
Consider the above statements and choose the correct statements:
(a) 1 and 2 only
(b) 2 and 3 only
(c) 2, 3 and 4 only
(d) 1, 2 and 4

 21
.


EXPLANATION:

The Information Fusion Centre – Indian Ocean region (IFC-IOR) was set up within the Information
Management and Analysis Centre (IMAC) to coordinate with regional countries on the maritime country
and act as a regional repository of maritime data in 2018. It presently has linkages with 21 partner
countries and 22 multi-national agencies across the globe. So, Statement 1 is correct.

India seeks to deepen economic and security cooperation with its maritime neighbours and assist in
building its maritime security capabilities. For this, India would cooperate on the exchange of
information, coastal surveillance, the building of infrastructure and strengthening their capabilities.
This is a vision of India's SAGAR(Security and Growth For All in the Region) mission. So the government
established, The Information Fusion Centre – Indian Ocean region (IFC-IOR) as part of the
government's SAGAR (Security and Growth For All in the Region) framework for maritime cooperation
in the Indian Ocean region. So, Statement 2 is correct.

IFC-IOR, hosted by the Indian Navy, was established by the Government of India at Gurugram on
December 22,2018, to further Maritime Safety and Security in the Indian Ocean Region.So, Statement
3 is not correct.

Additionally, monthly weather forecasts and weather warnings, and specific studies/reports are also
published by the Centre. So, Statement 4 is correct.

ADDITIONAL INFORMATION:

INDIAN OCEAN RIM ASSOCIATION FOR REGIONAL CO-OPERATION (IOR-ARC)

About ¾ IOR-ARC, a regional cooperation initiative of the Indian Ocean Rim countries,
was established in Mauritius in March 1997 to promote economic and technical
cooperation.
¾ IOR-ARC is the only pan-Indian ocean grouping.
¾ It brings together countries from three continents having different sizes,
economic strengths, and a wide diversity of languages and cultures.

Members ¾ Presently it has 19 members- Australia, Bangladesh, India, Indonesia, Iran,


Kenya, Malaysia, Madagascar, Mauritius, Mozambique, Oman, Seychelles,
Singapore, South Africa, Sri Lanka, Tanzania, Thailand, UAE and Yemen.
¾ There are five Dialogue Partners, namely China, Egypt, France, Japan and UK
¾ Two Observers namely, the Indian Ocean Research Group (IORG) and Indian
Ocean Tourism Organisation (IOTO), Oman.

 22
.


Q.17) Consider the following statements:


1. Nano urea is a patented, indigenously made nitrogenous fertilizer developed by the Indian
Council of Agriculture Research.
2. Urea has the highest nitrogen content of all solid nitrogenous fertilizers in common use.
3. Globally, India is the second biggest consumer of Urea.
Which of the statements given above is/are correct?
(a) 1 and 2 only
(b) 2 and 3 only
(c) 3 only
(d) 1, 2 and 3
EXPLANATION:

Nano urea liquid is a nanotechnology-based fertilizer to increase the growth of crops by restoring
nitrogen to plants as an alternative to conventional Urea.
Nano-urea (liquid nano urea)is a product developed by the Indian Farmers and Fertilizer Cooperative
(IFFCO).
Nano urea is a patented and indigenously made liquid containing Urea nanoparticles, the most crucial
chemical fertilizer for farmers in India. So, Statement 1 is not correct.
Urea has the highest nitrogen content of all solid nitrogenous fertilizers in common use (46.7%)
because it has the lowest transportation costs per unit of nitrogen nutrients. So, Statement 2 is
correct.
India is the 2nd largest consumer of Urea (As of May 28) in the world but only the third-largest
producer.

 23
.


India consumes around 33 million tonnes of Urea annually, of which almost 70% is domestically
produced, while the rest is imported from other countries.
India is the 2nd biggest consumer of fertilizers and 3rd biggest producer of fertilizer.
So, Statement 3 is correct.

ADDITIONAL INFORMATION:

NANO UREA

About ¾ French chemist Hillaire Rouelle discovered Urea in 1773.


Urea ¾ Urea is chemical nitrogen fertilizer, white in colour, which artificially provides
nitrogen, a major nutrient required by plants.
¾ Urea (NH2)2CO, is a colourless organic chemical compound also known by the
International Non-proprietary Name (rINN) carbamide, established by the
World Health Organization. It is highly soluble in water and has a pKa close to
zero.
¾ The primary raw material used to manufacture Urea is natural gas.

The Volume of ¾ India has become the first to start commercial production of Nano urea
Urea produced worldwide.
in India FY ¾ In the financial year 2022, more than 26 million metric tons of Urea were
2002-2022 produced in India. There was an increase of more than 37 percent in
comparison with 2002.
¾ Urea production in India presented an increasing trend with some fluctuations
over a period considered.

 24
.


¾ The liquid nano urea is produced by Indian Farmers Fertiliser Cooperative


(IFFCO)
¾ It is essentially Urea in the form of a nanoparticle.
¾ According to IFFCO, liquid nano urea contains 4 percent total nitrogen (w/v)
evenly dispersed in water. The size of a nano-nitrogen particle varies from 20-
Liquid Nano
50 nm. (A nanometre is equal to a billionth of a metre.)
Urea
¾ A single half-liter bottle of the liquid can compensate for a 45kg sack of Urea
that farmers traditionally rely on.
¾ The Nano urea liquid will increase the production of crops with improved
nutritional quality.
¾ Cheaper than conventional Urea, the new product is also expected to reduce
the environmental pollution caused by the granular form by reducing its
excessive application that exacerbates soil, water and air pollution with climate
change problems.
Indian Farmers ¾ Indian Farmers Fertiliser Cooperative Limited (IFFCO) is one of India's biggest
Fertilizer cooperative societies wholly owned by Indian Cooperatives, and it was founded
Cooperative in 1967.
Limited ¾ IFFCO's mission is "to enable Indian farmers to prosper through timely supply
of reliable, high-quality agricultural inputs and services in an environmentally
sustainable manner and to undertake other activities to improve their welfare."
¾ And they are Building a value-driven organization with an improved and
responsive customer focus, and a true commitment to transparency,
accountability and integrity in principle and practice.

Q.18) Consider the following statements, with reference to Human Immune system :
1. When our body encounters a pathogen for the first time, it produces anamnestic immune
response.
2. T-lymphocytes produce antibodies into our blood to fight with pathogens, while B- lymphocytes
help T- cells to produce them.
 25
.


3. Hydrochloric Acid (HCL) produced in stomach is an example for innate immune response.
Which of the statements given above is/are correct ?
(a) 1 only
(b) 1 and 2 only
(c) 3 only
(d) 1 and 3 only
EXPLANATION:

The primary immune response is one that is initiated when the immune system does not have prior
experience with the pathogen and encounters it for the first time. Anamnestic immune response is
one which is initiated when the immune system is exposed multiple times to the same pathogen. So,
Statement 1 is not correct.

B-cells initiate the antibody-mediated immune response. Activated B-cells transform into plasma cells
which secrete antibodies. B-cells mature in the bone marrow and then are carried by the blood to the
peripheral lymphoid organs. Whereas, T-cells induces the B-cells to produce antibody and regulates
the immune response by mediating the cell-mediated immune (CMI) response. So, Statement 2 is
not correct.

A healthy individual is generally immune to potentially harmful microorganisms by a number of very


effective mechanisms. Innate immunity is one which provides an immediate but relatively nonspecific
response to contain pathogens at the site of entry into the body. It is also known as natural immunity.
Innate immune defences include inflammatory and acute phase responses, as well as the anatomical
and chemical barriers provided by the skin and mucous membranes.
The hydrochloric acid (HCL) by the stomach plays an important role in protecting the body against
pathogens ingested with food or water. It is an example of Innate immune response.
So, Statement 3 is correct.
ADDITIONAL INFORMATION:

Antigen An antigen is any foreign molecule that can trigger a specific immune response. Most
antigens are either proteins or very large polysaccharides. Another term ‘immunogen’
is also used for antigen.

Antibody Antibody is a protein molecule produced in animals in response to an antigen.


Antibodies belong to the category of proteins called immunoglobulin.

Types of 1. Non-specific immune responses are those which non-selectively protect against
Immune foreign substances or cells without having to recognize their specific identities
Response 2. Specific immune responses (adaptive immune response) depend upon the
immunological recognition of the substances or cells to be attacked. Specific
immune responses are again of two types:
(a) Cell mediated immune responses: Mediated by cytotoxic T-cells and
natural killer cells. These constitute major defence against intracellular
viruses and cancer cells.
(b) Antibody-mediated or humoral immune responses: These responses are
mediated by antibodies secreted by plasma cells, which arise from
activated B-cells. They constitute major form of protection against bacteria
and viruses

 26
.


Q.19) The term 90-90-90 strategy recently seen in news is related to which one of the following diseases?
(a) AIDS
(b) Monkeypox virus
(c) Nipah virus
(d) SARS
EXPLANATION

The United Nations Programme on HIV/AIDS (UNAIDS) has developed an ambitious strategy to end
the AIDS epidemic, which is called as 90-90-90 Strategy,
The Targets were,
¾ By 2020, 90% of all people living with HIV will know their HIV status.
¾ By 2020, 90% of all people with diagnosed HIV infection will receive sustained antiretroviral
therapy.
¾ By 2020, 90% of all people receiving antiretroviral therapy will have viral suppression.
At the end of 2020, 78% of PLHIV knew their HIV status, 83% of PLHIV who knew their HIV status
were on ART, and 85% of PLHIV were on ART.

So, Option (a) is correct.

ADDITIONAL INFORMATION:
HUMAN IMMUNODEFICIENCY VIRUS (HIV)

About ¾ HIV (Human Immunodeficiency Virus) is a virus that attacks the body’s immune
system. If HIV is not treated, it can lead to AIDS (Acquired Immunodeficiency
Syndrome).
¾ There is currently no effective cure. Once people get HIV, they have it for life.
¾ But with proper medical care, HIV can be controlled. People with HIV who
get effective HIV treatment can live long, healthy lives.

 27
.


Targets

Antiretroviral ¾ The daily use of a combination of HIV medicines (called an HIV regimen) to treat
Therapy HIV infection.
¾ Antiretroviral (ARV) HIV drugs share common properties, including a similar
mechanism of action and chemical structure, used to prevent a retrovirus like
HIV from replicating.
¾ Approved antiretroviral (ARV) HIV drugs are divided into seven drug classes
based on how each drug interferes with the HIV life cycle. These seven classes
include
x The nucleoside reverse transcriptase inhibitors (NRTIs)
x Non-nucleoside reverse transcriptase inhibitors (NNRTIs)
x Protease inhibitors (PIs)
x Fusion inhibitors
x CCR5 antagonists
x Post-attachment inhibitors
x Integrase strand transfer inhibitors (INSTIs).

Retrovirus ¾ A type of virus that uses RNA as its genetic material.


¾ After infecting a cell, a retrovirus uses a reverse transcriptase enzyme to convert
its RNA into DNA.
¾ The retrovirus then integrates its viral DNA into the DNA of the host cell, which
allows the retrovirus to replicate.
¾ HIV, the virus that causes AIDS, is a retrovirus.

 28
.


Q.20) With reference to Green Chemistry, consider the following statements :


1. Pseudomonas putida is used in bioremediation of oil spills.
2. H202 is used as antiseptic, disinfectant and in rocketry as a propellant.
Which of the statements given above is/are correct?
(a) 1 only
(b) 2 only
(c) Both 1 and 2
(d) Neither 1 nor 2
EXPLANATION:

¾ Green chemistry is the processes and design of chemical products that reduce or eliminate the use
or generation of hazardous substances. It reduces pollution at its source by minimizing or
eliminating the hazards of reagents, solvents, and products.
¾ Bioremediation is a branch of biotechnology that uses living organisms, like microbes and bacteria,
to decontaminate affected areas. It removes contaminants, pollutants, and toxins from soil, water,
and other environments.
¾ Bioremediation of oil spills by using Pseudomonas putida is a curative measure. Pseudomonas
putida is capable of converting styrene oil into biodegradable plastic PHA. Pseudomonas putida is
a rod-shaped, flagellated, gram-negative bacterium that is found in most soil and water habitats
where there is oxygen.
So, Statement 1 is correct.
¾ Hydrogen peroxide is a chemical compound with the formula H₂O₂. It breaks down quickly and
easily when it comes into contact with air or water, so it’s safer than chlorine chemicals.
¾ Hydrogen peroxide is active against many microorganisms, including bacteria, yeasts, fungi, and
viruses. So, it is used as an antiseptic and disinfectant.
¾ Hydrogen peroxide exothermically decomposes to water and oxygen, making it an ideal oxidizer for
more environment-friendly propulsion systems. Thus using H2O2 as an antiseptic, disinfectant,
and in rocketry, as a propellant.
So, Statement 2 is correct.

 29
.


ADDITIONAL INFORMATION :
GREEN CHEMISTRY

Green chemistry is the processes and design of chemical products that reduce or eliminate the use
or generation of hazardous substances. It reduces pollution at its source by minimizing or
eliminating the hazards of reagents, solvents, and products.
¾ Prevents pollution at the molecular level
¾ It is a philosophy that applies to all areas of chemistry, not a single discipline of chemistry.
¾ Applies innovative scientific solutions to real-world environmental problems
¾ Results in source reduction because it prevents the generation of pollution
¾ Reduces the negative impacts of chemical products and processes on human health and the
environment
¾ Designs chemical products and processes to reduce their intrinsic hazards.

Q.21) Which of the following pairs is not correctly matched?


(a) Blue origin - Spaceflight Company
(b) Tiangong-1 - Space station
(c) Falcon - Submarine cable system
(d) Tokamak reactor - Nuclear fission
EXPLANATION:

¾ Blue Origin is an American Private space flight company Jeff Bezos founded, and it has been testing
the New Shepard rocket and its redundant safety systems since 2012. The program has had
22 successful consecutive missions, including three successful escape tests, showing the crew
escape system can activate safely in any phase of flight. So, Pair 1 is correct.

¾ Tiangong-1 is China's First Space Station, a single-module space station operated by the China
National Space Administration. The module was launched in 2011.
¾ In 2016, Chinese flight controllers lost control of Tiangong-1, which was burned in the
atmosphere and fell into the Pacific Ocean in 2018. So, Pair 2 is correct.

The FLAG Alcatel-Lucent Optical Network (FALCON) is a 11859km submarine cable system connecting
India, Sri Lanka, Maldives, Oman, Kuwait, Bahrain, Qatar, Saudi Arabia, Iraq, Yemen, Sudan
and Egypt.

 30
.


So, Pair 3 is correct.


The Tokamak is a doughnut-shaped experimental machine designed to harness fusion energy. Inside
a Tokamak, the energy produced through the fusion of atoms is absorbed as heat in the vessel's walls.
A fusion power plant will use this heat to produce steam and electricity through turbines and
generators like a conventional power plant.
Thus, Tokamak is Nuclear fusion reactor not a Nuclear Fission reactor. So, Pair 4 is not correct.

Q.22) With reference to GM crops in India, consider the following statements:


1. Bt cotton is the only genetically modified (GM) cash crop that has been approved for commercial
cultivation.
2. Cry1Ac protein is present in Bt cotton as well as Bt Brinjal development.
3. All gene edited products are classified as transgenic products.
Which of the statements given above is/are correct ?
(a) 1 and 2 only
(b) 2 and 3 only
(c) 2 only
(d) 1, 2 and 3
EXPLANATION:

Bt. cotton is the only GM crop approved for commercial cultivation in 2002 by the Genetic Engineering
Appraisal Committee of the Ministry of Environment, Forest and Climate Change for commercial
cultivation. So, Statement 1 is correct.

Cry1Ac gene was taken from the soil bacterium Bacillus thuringiensis and it provides protection from
feeding damage by target lepidopteran pests, including the velvetbean caterpillar (Anticarsia
gemmatalis), soybean looper (Pseudoplusia includens), soybean axil borer (Epinotia aporema), and
sunflower looper (Rachiplusia nu).
Cry 1 Ac was inserted in Bt.cotton as well as Bt.Brinjal. In Bt.Brinjal, it give the plant resistance against
lepidopteran insects like the Brinjal Fruit and Shoot Borer Leucinodes orbonalis and Fruit Borer
Helicoverpa armigera.

 31
.


In Bt cotton plants, Studies have shown that Cry1Ac in Bt cotton is highly selective because it kills
only certain caterpillar species. Bt cotton with Cry1Ac has minimal or no effect on beneficial insects,
including honey bees, lady beetles, spiders, bigeyed bugs, pirate bugs, and parasitic wasps.
However, laboratory research has shown that Cry1Ab protein can indirectly affect green lacewing larvae
that eat Bt-killed caterpillars. Different gene protein types are used in Bt Cotton Development for
different predators.
For example, Cry3A proteins affect Colorado potato beetle larvae; Cry1Ac is used against tobacco
budworms; and European corn borers can be killed with Cry1Ab, Cry1F, Cry1Ac, and Cry9c proteins.
Thus, Cry1Ac is present in Bt Cotton and Bt Brinjal.
So, Statement 2 is correct.

Genome editing is used to precisely and efficiently modify DNA within a cell.
There are three categories of gene editing:
¾ Site-Directed Nuclease (SDN)1,
¾ SDN2
¾ SDN3
The first two largely involve “knocking off” or “overexpressing” certain traits in a genome without
inserting gene material from outside. The SDN3, which involves the insertion of foreign genes, will only
be treated as transgenic products. So, Statement 3 is not correct.

ADDITIONAL INFORMATION:

GENETIC ENGINEERING

About Genetic engineering (also called genetic modification) is a process that uses
laboratory-based technologies to alter the DNA makeup of an organism. This
may involve changing a single base pair (A-T or C-G), deleting a region of DNA
or adding a new segment of DNA.

DNA and RNA DNA (Deoxyribonucleic Acid) RNA(Ribonucleic acid)


¾ Double-stranded molecule ¾ Single-stranded molecule
¾ Contains deoxyribose sugar ¾ Contains ribose sugar
¾ Attached to each sugar is one of ¾ Attached to each sugar is one of
four bases: adenine (A), cytosine four bases: adenine (A), uracil (U),
(C), guanine (G) or thymine (T) cytosine (C) or guanine (G)
¾ DNA has only one function, that ¾ Many species of RNA are in
to bear hereditary information formation mRNA, tRNA, and rRNA
¾ DNA can duplicate on its own with different functions. RNA is the
genetic material in retroviruses
¾ RNA is synthesized on a DNA
template

 32
.


Recombinant DNA Recombinant DNA technology involves enzymes and various laboratory
Technology techniques to manipulate and isolate interesting DNA segments. This method
can combine (or splice) DNA from different species or create genes with new
functions. The resulting copies are often referred to as recombinant DNA. Such

 33
.


work typically involves propagating the recombinant DNA in a bacterial or


yeast cell, whose cellular machinery copies the engineered DNA with its own.

Q.23) Fostering Effective Energy Transition 2022’ report was released by which of the below
organization?
(a) International Energy Agency
(b) World Energy Council
(c) World Economic Forum
(d) International Renewable Energy Agency
EXPLANATION:
Fostering Effective Energy Transition 2022 report is a special edition report on the global energy
transition released by the World Economic Forum (WEF). So, Option (c) is correct.

ADDITIONAL INFORMATION:

WORLD ECONOMIC FORUM

About ¾ The World Economic Forum is the International Organization for Public-Private
Cooperation.
¾ The Forum engages the foremost political, business, cultural and other leaders
of society to shape global, regional and industry agendas.
¾ The headquarters is located in Geneva, Switzerland.

Reports and ¾ The Net-Zero Industry Tracker


indices by ¾ Global Gender Gap Report
WEF ¾ Travel & Tourism Development Index
¾ Global Information Technology Report
¾ Global Competitiveness Report
¾ Enabling Trade Report

 34
.


¾ Global Environment Performance Index


¾ Inclusive Development Index
¾ Human Capital Index
¾ Global Gender Gap Index
¾ Education 4.0 India

Q.24) Consider the following statements in respect to Li-Fe battery :


1. It cannot be recharged in partially discharged conditions.
2. Repeated partial discharge in Li-Fe batteries can cause a battery to ‘remember’ a lower capacity.
3. It uses organic compounds as an electrolyte.
4. Solid-state batteries offer higher safety than Li-ion electrolyte batteries.
Which of the statements given above is/are correct ?
(a) 1 and 4
(b) 2 only
(c) 3 and 4
(d) 1, 2 and 3
EXPLANATION:
A lithium-ion (Li-ion) battery is an advanced battery technology that uses lithium ions as a key
component of its electrochemistry. One of the main advantages of this battery over other batteries is
that it can be recharged at any time even in partially discharged conditions. So, Statement 1 is not
correct.

Repeated partial discharge/charge cycles cannot cause a Li fe battery to ‘remember’ a lower capacity
because there is no memory effect, hence the battery does not need periodic full discharge cycles to
prolong life. The memory effect in batteries is also called the lazy battery effect. If the battery is not
discharged completely before the charging, small crystals are formed on the electrodes and they reduce
the possibility to accept a charge. Therefore if the batteries do not discharge completely one time after
another, the operational times become increasingly shorter. So, Statement 2 is not correct.
¾ The electrolytes used in lithium-ion batteries are a mixture of both organic and inorganic compound
¾ Lithium hexafluorophosphate (LiPF6) salt (inorganic) is dissolved in Organic carbonates (ethanol,
acetonitrile, dimethyl carbonate, dimethyl sulfoxide, and propylene carbonate).
¾ This electrolyte is the non-aqueous solution (The solution in which any liquid other than water acts
as a solvent is called a non-aqueous solution). So, Statement 3 is correct.

The electrolyte used in liquid li-ion batteries is volatile and flammable at high temperatures. This
makes electric vehicles that use Li-Fe batteries more vulnerable to fire and chemical leaks. But the
solid-state batteries use a thin layer of solid electrolyte that carries lithium ions between electrodes.
Thus Solid-state batteries offer higher safety than Li-ion electrolyte batteries. So, Statement 4 is
correct.

ADDITIONAL INFORMATION:

LITHIUM-ION BATTERIES
Working ¾ The battery is made up of an anode, cathode, separator, and electrolyte.
principle ¾ The anode and cathode store the lithium.

 35
.


¾ The electrolyte carries positively charged lithium ions from the anode to the
cathode and vice versa through the separator.
¾ The movement of the lithium ions creates free electrons in the anode which
creates a charge at the positive current collector.
¾ The electrical current then flows from the current collector through a device
being powered (cell phone, computer, etc.) to the negative current collector.
¾ The separator blocks the flow of electrons inside the battery.

Solid-state ¾ Solid-state batteries are Li-ion batteries that use thin layers of solid electrolytes,
batteries which carry lithium ions between electrodes.
¾ This battery has more stability, fast charging capability, and reduce safety
measures.

Q.25) Consider the following statements:


1. Cassini-Huygens Mission has found that Titan has both methane and ethane.

2. Methane is a chemical compound with simplest alkene group, which is a main constituent of
natural gas.
Which of the statements given above is/are not correct?
(a) 1 only
(b) 2 only
(c) Both 1 and 2
(d) Neither 1 nor 2
EXPLANATION:
The Cassini-Huygens mission is a cooperative project of NASA, ESA (European Space Agency) and the
Italian Space Agency.

 36
.


Both Earth and Titan (Saturn's moon - Titan) have nitrogen-dominated atmospheres. However, the
findings of Cassini reveal that, unlike Earth, Titan also has very little oxygen; the rest of the atmosphere
is mostly methane and trace amounts of other gases, including ethane. And at the frigid temperatures
found at Saturn's great distance from the sun, the methane and ethane can exist on the surface in
liquid form. So, Statement 1 is correct.

¾ Alkanes are saturated open-chain hydrocarbons containing carbon-carbon single bonds. Methane
(CH4) is the first member of this family (not alkene).
¾ Natural gas is a fossil energy source formed deep beneath the earth's surface. Natural gas contains
many different compounds. The largest component of natural gas is methane.
¾ Natural gas also contains smaller amounts of natural gas liquids (NGLs, which are also
hydrocarbon gas liquids), and nonhydrocarbon gases, such as carbon dioxide and water vapor. So,
Statement 2 is not correct.

ADDITIONAL INFORMATION:
ALKANES

About ¾ Hydrocarbons are compounds containing carbon and hydrogen.


¾ Hydrocarbons are considered the parent organic compounds, from which other
organic compounds can be derived by replacing one or more hydrogen atoms
with different functional groups.
¾ Alkanes are saturated hydrocarbons. They are very less reactive towards various
reagents; hence, they are also referred to as paraffin (parum means little, affine
means affinity)
¾ The alkanes containing 1 to 4 carbon atoms are gases, whereas those containing
5 to 17 carbon atoms are liquids, and the still higher ones are solids.
Uses of ¾ Alkanes are used as fuel gases, solvents, dry-cleaning agents, lubricants and in
Alkanes ointments (paraffin wax).
¾ Methane is used for illuminating and domestic fuel and for producing other
organic compounds such as haloalkanes, methanol, formaldehyde and
acetylene.
¾ Propane is used as a fuel, refrigerant, and raw material in the petrochemical
industry.
¾ Butane and its isomer–isobutane, are the major constituents of LPG.

ANOTHER MISSION ON SATURN

Dragonfly ¾ NASA's newest mission is to explore the solar system and deliver an 8-bladed
Mission rotorcraft to visit Saturn's largest and richly organic moon, Titan.
¾ This mission is planned to launch in 2027 and arrive in 2034; Dragonfly will
sample and examine dozens of promising sites around Saturn's icy moon.

TITAN

About ¾ Titan is Saturn's largest moon.


¾ Titan is the second largest moon in our solar system afterJupiter's moon
Ganymede
¾ This moon is the only moon in the solar system with a dense atmosphere.

 37
.


¾ Titan's atmosphere is mostly nitrogen (about 95 percent) and methane (about 5


percent), with small amounts of other carbon-rich compounds.
¾ The discovery of a global ocean of liquid water adds Titan to the handful of worlds
in our solar system that could potentially contain habitable environments

Q.26) With reference to UN Convention on the Law of the Sea (UNCLOS), consider the following
statements:
1. It is an international legally binding agreement that establishes the legal framework for marine
and maritime activities.
2. It is the only international convention which stipulates a framework for state jurisdiction in
maritime spaces.
3. State sovereignty extends to the air space over the territorial sea as well as to its bed and subsoil.
Which of the statements given above are correct ?
(a) 1 and 2 only
(b) 2 and 3 only
(c) 1 and 3 only
(d) 1, 2 and 3
EXPLANATION:

The United Nations Convention on the Law of the Sea was adopted in 1982, an international legally
binding agreement that provides the legal framework for the conservation and sustainable use of
marine biological diversity in areas beyond national jurisdiction
The international Convention lays down a comprehensive law and order in the world ocean and seas,
a regime of consecutive maritime zones. So, Statement 1 is correct.

The Convention establishes a comprehensive legal framework to regulate all ocean space, its uses and
resources. It contains, among other things, provisions relating to Internal Waters, Territorial Sea,
Contiguous Zone, Exclusive Economic Zone (EEZ) and the High Seas.
It provides governance for offshore coastal states and those navigating the oceans and specific guidance
for states' rights and responsibilities for the zones.
Hence, it is the only international Convention with an agreement for the framework for
state jurisdiction in maritime spaces, which gives the legal status of the territorial sea, the air space
over the territorial sea, and its bed and subsoil.
The sovereignty of a coastal State extends beyond its land territory and internal waters and, in the case
of an archipelagic State(archipelago means a group of islands-like States such as the Philippines and
Indonesia), its archipelagic waters to an adjacent belt of sea, described as the territorial sea. Its
sovereignty extends to the air space over the territorial sea and its bed and subsoil. So, Statements 2
and 3 are correct.

ADDITIONAL INFORAMATION:

CONVENTION ON THE LAW OF THE SEA (UNCLOS)


Background ¾ The origins of the Convention date from 1 November 1967in the General
Assembly of the United Nations meeting, for "an effective international
regime over the seabed and the ocean floor beyond a clearly defined national
jurisdiction."

 38
.


¾ The Convention concerns the exploration for and exploitation of the seabed,
ocean floor, and subsoil resources beyond national jurisdiction limits (the Area).
The Convention declares the Area and its resources the common heritage of
mankind.

Exclusive ¾ It recognizes the right of coastal States to jurisdiction over the resources of some
Economic 38 million square nautical miles of ocean space. To the coastal State falls the right
Zone to exploit, develop, manage and conserve all resources - fish or oil, gas or gravel,
And nodules or sulphur - to be found in the waters, on the ocean floor and in the
High seas subsoil of an area extending 200 miles from its shore.
¾ The EEZ is a zone between the territorial sea (12 nautical miles) and the high seas
to the maximum extent of 200 nautical miles (no state can act or interfere with
the justified and equal interests of other states).
In the EEZ zone, the coastal State retains exclusive sovereignty over exploring,
exploiting and conserving all natural resources.

International International Seabed Authority (ISA)which regulates activities in the seabed beyond
Seabed national jurisdiction to explore Deep-sea mining, is the process of retrieving mineral
Authority deposits from the deep seabed (ocean below 200m).
(ISA)

Currently ¾ India ratified the UN Convention on the Law of the Sea (UNCLOS) on 29 June
1995.
¾ As of October 2021, India reiterated its support for rule-based maritime systems,
as mandated under the United Nations Convention on the Law of the Sea
(UNCLOS) (territorial waters and Exclusive Economic Zone).
¾ As of December 2021, India also supports freedom of navigation, overflight, and
unimpeded commerce based on the principles of international law, reflected
notably in the 'United Nations Convention on the Law of the Sea (UNCLOS) 1982.
¾ India, a state party, promotes utmost respect for the UNCLOS, establishing the
international legal order of the seas and oceans.

 39
.


Q.27) Consider the following statements :


1. BCG vaccine has a protective effect against meningitis and disseminated TB in children.
2. BCG vaccine is not included in India’s universal immunisation programme.
3. Extensively drug-resistant tuberculosis (XDR-TB) in HIV patients leads to high mortality.
Which of the statements given above is/are correct?
(a) 1 and 3 only
(b) 2 only
(c) 3 only
(d) 1, 2 and 3
EXPLANATION:

BCG vaccine has a documented protective effect against meningitis and disseminated TB in children
(75–87%).
However, It does not prevent primary infection and, more importantly, does not prevent reactivation of
latent pulmonary infection, the principal source of bacillary spread in the community. The impact of
BCG vaccination on transmission of Mycobacterium tuberculosis (Mtb) is therefore limited.
Although protective, it is clear that BCG vaccination is not 100% efficacious in preventing TBM -
Tuberclosis Meningitis. So, Statement 1 is correct.

Ministry of Health and Family Welfare, Government of India provides several vaccines to infants,
children and pregnant women through the Universal Immunisation Programme.
Vaccines provided under UIP:
¾ BCG (Bacillus Calmette-Guerin vaccine)
¾ OPV (Oral Polio Vaccine)
¾ Hepatitis B vaccine
¾ Pentavalent Vaccine
¾ Rotavirus Vaccine
¾ PCV (Pneumococcal Conjugate Vaccine)
¾ FIPV (Fractional Inactivated Poliomylitis Vaccine)
¾ Measles/ MR vaccine
 40
.


¾ JE vaccine (Japanese encephalitis vaccine)


¾ DPT booster (Diphtheria, Tetanus and Pertussis)
¾ Tetanus and adult diphtheria (Td) vaccine
So, Statement 2 is not correct.
¾ Extensively drug-resistant tuberculosis (XDR-TB) is a form of TB which is resistant to at least four
of the core anti-TB drugs.
¾ XDR-TB involves resistance to
x The two most powerful anti-TB drugs, isoniazid and rifampicin,
x Any of the fluoroquinolones (such as levofloxacin or moxifloxacin)
x At least one of the three injectable second-line drugs (amikacin, capreomycin or kanamycin).
The persons infected with HIV are more likely to experience XDR-TB. XDR-TB is virtually untreatable
with available TB medications. Thus, this increase the mortality rate among HIV patients. So,
Statement 3 is correct.

ADDITIONAL INFORMATION:
ANTIMICROBIAL RESISTANCE

About ¾ Antimicrobial Resistance (AMR) occurs when pathogens change over time and
no longer respond to medicines making infections harder to treat and increasing
the risk of disease spread, severe illness and death.
¾ As a result of drug resistance, antibiotics and other antimicrobial medicines
become ineffective and infections become increasingly difficult or impossible to
treat.

Antimicrobials ¾ Antimicrobials are medicines used to prevent and treat infections in humans,
animals and plants.
¾ It includes
x Antibiotics
x Antivirals
x Antifungals
x Antiparasitics

Q.28) Consider the following statements with reference to Mars :


1. The planets appear to move slower when they are farther from the sun and faster when they are
nearer.
2. Mars Orbiter Mission(MOM) is ISRO’s first interplanetary mission to planet Mars with an orbiter
craft designed to orbit Mars in an eccentric orbit of 372 km to 80,000 km.
3. Density of Martian atmosphere is higher than Earth’s atmospheric density.
Which of the statements given above is/are correct ?
(a) 1 only
(b) 1 and 2 only
(c) 2 and 3 only
(d) 3 only

 41
.


EXPLANATION:

According to Kepler’s Law of planetary mission, the line that joins any planet to the sun sweeps equal
areas in equal intervals of time. This law comes from the observations that planets appear to move
slower when they are farther from the sun than when they are nearer.

And also the speed at which a planet orbits the Sun changes depending on how far it is from the
Sun. When a planet is closer to the Sun the Sun’s gravitational pull is stronger, so the planet moves
faster. When a planet is further away from the sun the Sun’s gravitational pull is weaker, so the planet
moves slower in its orbit. Since orbital velocity or orbital speed changes according to the distance from
the Sun, its areal velocity remains constant. The areal velocity is the rate at which area is swept out
by a particle as it moves along the curve. So, Statement 1 is correct.
Mars Orbiter Mission (MOM), India's first interplanetary mission by the Indian Space Research
Organisation (ISRO) to planet Mars was launched onboard PSLV-C25 on November 05, 2013. It carries
an orbiter craft designed to orbit Mars in an elliptical orbit of 372 km by 80,000 km. This mission can
be termed as a challenging technological mission and a science mission considering the critical mission
operations and stringent requirements on propulsion, communications, and other bus systems of the
spacecraft. So, Statement 2 is correct.
The Martian atmosphere is an extremely thin sheet of gas, principally carbon dioxide, that extends
from the surface of Mars to the edge of space. Earth’s atmosphere is over 100 denser than Mars's.
Whereas Mars's atmosphere is less dense and has a much thinner atmospheric volume compared to
Earth, but there are many similarities. Gravity holds the atmosphere to the Martian surface and within
the atmosphere, very complex chemical, thermodynamic, and fluid dynamics effects occur. The
atmosphere is not uniform; fluid properties are constantly changing with time and place, producing
weather on Mars just like on Earth. So, Statement 3 is not correct.

ADDITIONAL INFORMATION:

Mars Orbiter Mission (MOM)

Uniqueness ¾ The highly elliptical orbit geometry of MOM enables its Camera (MCC) to take
snapshots of the Full disc of Mars at its farthest point and finer details from the
closest point.
¾ First-time observation of the far side of Deimos, one of the moons of Mars

Achievements ¾ ISRO has become the fourth space agency to successfully send a spacecraft to
Mars orbit.

 42
.


¾ The Mars Colour Camera, one of the scientific payloads onboard MOM, has
produced 1100+ images so far and published a Mars Atlas.
¾ Published more than 35 research papers in peer-reviewed journals.
¾ India’s ability to successfully realize the complex mission to Mars in its first
attempt, in a cost-effective (Rupees 450 Cr) has captured the world's attention
and has propelled India’s image as a credible space fairing nation to greater
heights. This capability could pave the way for greater opportunities for Space
Commerce including launch services and marketing of Satellite Imageries.
¾ It is a mission of national pride that has attracted the attention of students, the
general public, the media, and the international science/ technical community.
¾ Importantly, Mars Orbiter Mission has created enthusiasm among the younger
generation in the country, provoked their curiosity to understand and discuss
space-related techniques, and is maintaining the tempo throughout the mission.

Q.29) With reference to Open Network for Digital Commerce (ONDC) project, consider the following
statements:
1. ONDC promotes open networks for all aspects of exchange of goods and services over digital
networks.
2. ONDC is not platform-centric, consumers can purchase using any platform.
3. ONDC is a regulating body of the e-commerce ecosystem in India.
Which of the following statements is/are correct?
(a) 1 only
(b) 2 and 3 only
(c) 2 only
(d) 1 and 2 only
ADDITIONAL

Open Network for Digital Commerce (ONDC) is a freely accessible government-backed platform that
aims to democratise e-commerce by moving it from a platform-centric model to an open network for
buying and selling of goods and services.
Open Network for Digital Commerce (ONDC) is an initiative to promote open networks for all aspects
of exchanging goods and services over digital or electronic networks. So, Statement 1 is correct.
ONDC, a UPI of e-commerce, seeks to democratise digital or electronic commerce, moving it from a
platform-centric model to an open network.
ONDC plans to enable sellers and buyers to be digitally visible and transact through an open network.
The new framework aims at promoting open networks developed on open-sourced methodology, using
open specifications and open network protocols independent of any specific platform. So, Statement
2 is correct.

As far as a regulatory framework for governing e-commerce activities is concerned, there are no
dedicated e-commerce laws and a regulating body in India.
Various ministries and the Department of the government of India deal with e-commerce.
For instance, the Ministry of Electronics and Information Technology looks after the technical aspects
of e-commerce through the information technology Act, data privacy issues, etc.

 43
.


The Department of Consumer Affairs takes care of consumer protection issues. The Department for
promotion of Industry and Internal Trade deals with foreign investment-related matters on e-
commerce.
The Department of commerce deals with the WTO discussions on e-commerce. Although no dedicated
law governs e-commerce, such activities are governed by several laws and regulations applicable to
various segments of the e-commerce value chain.
Some of these laws come under the purview of the central government, whereas others fall within the
jurisdiction of state governments.
"ONDC is a democratisation of the digital commerce world, and it can trigger off many startups to bring
technology into the remotest corners of India, and hence it is not the regulating body of the e-commerce
ecosystem in India. So, Statement 3 is not correct.

ADDITIONAL INFORMATION:

OPEN NETWORK FOR DIGITAL COMMERCE (ONDC)


¾ Open Network for Digital Commerce (ONDC) is a network based on an open protocol.
About It will enable local commerce across segments, such as mobility, grocery, food order
and delivery, hotel booking and travel, among others, to be discovered and engaged
by any network-enabled application.
¾ It is a not-for-profit organisation offering a network to enable local digital commerce
stores across industries to be discovered and engaged by any network-enabled
applications.
¾ The platform aims to create new opportunities, curb digital monopolies, support
micro, small and medium enterprises and small traders and help them get on online
platforms. It is an initiative of the Department for Promotion of Industry and Internal
Trade (DPIIT) under the Ministry of Commerce and Industry.

Q.30) Consider the following statements:


1. Speed of the sound in air increases with increase in temperature.
2. When a sound waves travels from one medium to another medium, the frequency remains
constant.
3. Sound travels faster in hydrogen medium compared to air medium.
Which of the statements given above is/are correct ?
(a) 1 and 2 only
(b) 1 only
 44
.


(c) 2 and 3 only


(d) 1, 2 and 3
EXPLANATION:

Sound is the form of energy which gives the sensation of hearing. The speed of sound depends upon
medium and temperature. Temperature is also a form of energy. When the temperature increases the
sound waves get extra energy and vibrate more. Thus the speed of the sound will increase with
temperature increase. So, Statement 1 is correct.

The number of vibrations made by a particle in one second is called the frequency of a sound wave.It
depends upon the source of the sound. Thus the frequency will not change when a sound wave travel
from one medium to another. So, Statement 2 is correct.

The speed of the sound will decrease with the increasing density. The density of Hydrogen is lesser
than the density of air. Thus the sound travels faster in Hydrogen medium compared to air medium.
So, Statement 3 is correct.

ADDITIONAL INFORMATION:

SOUND AND COMMUNICATION


Sound ¾ Sound is the form of energy which gives the sensation of hearing. It travels in the
form of waves.

Wave ¾ A wave is a disturbance that moves through a medium when the particles of the
medium set neighbouring particles into motion.
¾ The wave which can travel without medium is called non mechanical or
electromagnetic wave. The wave which requires medium for its propagation is
called mechanical wave.
¾ Mechanical waves are of two types: (i) longitudinal waves and (ii) transverse
waves.
¾ The waves in which vibrations are along the direction of wave propagation are
longitudinal waves.
¾ They travel in the form of rarefactions and compressions. These types of waves
are sound waves. Vibrations produce sound and the sound of frequency 20 Hz
to 20000 Hz is audible to humans. The sound of frequency greater than 20000
Hz is called ultrasonic and less than 20 Hz is called infrasonic
¾ The transverse wave travels in the form of crest and trough.

Wave length The wavelength is the distance between two consecutive crests or troughs in a wave.
Frequency and The change in density from the maximum value to the minimum value, then again
Amplitude to the maximum value, makes one complete oscillation. The number of such
oscillations per unit time is the frequency of the sound wave

 45
.


Q.31) The Treaty of Tumlong is related to which of the below states?


(a) Sikkim
(b) Tripura
(c) Assam
(d) Meghalaya
EXPLANATION:

The Treaty of Tumlong was signed here in 1861 between the British and the Sikkim Rajah, and the
treaty secured protection for travellers to Sikkim and guaranteed free trade, thereby making the state
under British protectorate (British Protectorates was a status held by several of the former British
territories within the British Empire)
A turning point in the history of Sikkim involves the appointment of John Claude White as the Political
Officer of Sikkim.
Sikkim was a British Protectorate under the Treaty of Tumlong, signed in March 1861. So, Option (a)
is correct.

ADDITIONAL INFORMATION:

SIKKIM

About ¾ In the early 18th century, the British Empire sought to establish trade routes with
Tibet, leading Sikkim to fall under British suzerainty until independence in 1947.

Scenario ¾ Three years after India’s Independence in 1947, Sikkim became a protectorate of
After India.
1947 ¾ In 1950, a treaty was signed between the then Sikkim monarch Tashi Namgyal and
India’s then Political Officer in Sikkim, Harishwar Dayal and said that
¾ “Sikkim shall continue to be a Protectorate of India and, subject to the provisions of
this Treaty, shall enjoy autonomy concerning its internal affairs.”

Decision ¾ A referendum was held in 1975, where an overwhelming majority voted to abolish the
To Join monarchy and join India.
India ¾ Fifty-nine thousand six hundred thirty-seven voted to abolish the monarchy and join
India, with only 1,496 voting against it.
¾ Sikkim’s new parliament, led by Kazi Lhendup Dorjee, proposed a bill for Sikkim to
become an Indian state, which the Indian government accepted.

 46
.


Q.32) Consider the following statements:


1. Gravity is always attractive while electromagnetic force is always repulsive.
2. The gravitational force is the force of mutual attraction between any two objects by virtue of their
masses.
3. Electromagnetic force can act over large distances without any medium and is enormously strong
compared to gravity.
4. Neutrinos experience strong electromagnetic force.
Which of the statements given above are correct?
(a) 1 and 3 only
(b) 2 and 4 only
(c) 2 and 3 only
(d) 1 and 4 only
EXPLANATION:

OPTION ELIMINATION STRATEGY


Since the gravitational force is directly proportional to the mass of both bodies, every object is pulled
towards and attractive; hence, an answer could be derived from these facts.

The electromagnetic force is the force between charged particles. It is attractive for unlike charges and
repulsive for like charges.
It is the force between any two objects in the universe. It is an attractive force by virtue of their masses.
Gravitational force is the weakest force among the fundamental forces of nature but has the greatest
large−scale impact on the universe. Unlike the other forces, gravity works universally on all matter and
energy, and is universally attractive.
So, statement 1 is not correct and statement 2 is correct.

Electromagnetic force is the combination of electrostatic and magnetic forces. It is very strong
compared to the gravitational force. The electromagnetic force acts over large distances and does not
need any intervening medium. This means that electromagnetic waves can travel not only through air
and solid materials but also through the vacuum of space. So hence they do not require a medium to
propagate. So, statement 3 is correct.

As Neutrino are electrically neutral, they are not affected by electromagnetic forces. Neutrinos are
affected by only a weak subatomic force of much shorter range than electromagnetism, and are
therefore able to pass through great distances in matter without being affected by it. So, statement 4
is not correct.

Q.33) Consider the following statements:


1. Addition of soluble fluoride to drinking water supply will be beneficial for dental health.
2. Fluoridated toothpastes with 1000 and 1500 ppm fluoride is the standard recommended by
WHO as a public oral health measure for the prevention of caries.
3. Excess fluoride in drinking water can cause disease such as methemoglobinemia.
Which of the statements given above is/are correct?
(a) 1 and 2 only
(b) 2 and 3 only
(c) 1 and 3 only
(d) 1, 2 and 3

 47
.


EXPLANATION:

The International Standards for drinking water states that water should be tested for fluoride ion
concentration. Its deficiency in drinking water is harmful to man and causes diseases such as tooth
decay etc. That’s why Soluble fluoride is often added to drinking water to bring its concentration upto
1 ppm or 1 mg dm–3 for the benefit of dental health. So, Statement 1 is correct.

Twice daily use of fluoride toothpaste for tooth brushing is an effective way to prevent dental caries
(tooth decay or dental cavities) among infants, children, adults, and seniors. Extensive clinical trials
over the last 60 years have shown that fluoride toothpaste delivers fluoride safely and is effective in
preventing caries.
In 2019, World Health Organisation(WHO) recommended concentration offluorine in toothpaste should
between 1000 and 1500 ppm for all age groups to prevent caries which is the most common
noncommunicable disease worldwide. So, Statement 2 is correct.

Excess fluoride in drinking water can cause diseases such as Dental fluorosis, Skeletal fluorosis,
Thyroid, and Neurological problems whereas Methemoglobinemia (Blue Baby syndrome) is caused by
the excess Nitrate content in drinking water. Methemoglobinemia is a blood disorder in which an
abnormal amount of methemoglobin is produced. Methemoglobin is a form of hemoglobin.
Note: Hemoglobin is the protein in red blood cells (RBCs) that carries and distributes oxygen to the
body. So, Statement 3 is not correct.

Q.34) The Regional Anti-Terrorist Structure (RATS) is associated with :


(a) Shanghai Cooperation Organisation
(b) Asian Development Bank
(c) BRICS
(d) Financial Action Task Force
EXPLANATION:

The Regional Anti-Terrorism Structure (RATS) is a permanent organ of the Shanghai Cooperation
Organisation (SCO), which serves to promote cooperation of member states against terrorism,
separatism and extremism, which specifically deals with issues relating to security and defence. So,
Option (a) is correct.

ADDITIONAL INFORMATION:
Asian ¾ The Asian Development Bank (ADB) is a regional development bank established
Development on 19 December 1966 which envisions a prosperous, inclusive, resilient, and
Bank sustainable Asia and the Pacific while sustaining its efforts to eradicate extreme
poverty in the region.
¾ It aims to promote social and economic development in Asia and the Pacific.
¾ Headquartered in Manila, Philippines.

Members of ¾ India is a founding member of ADB


ADB ¾ Current Members - 68
x 49 are from within Asia and the Pacific and
x 19 outside (Non-Members)

 48
.


BRICS ¾ It started as BRIC in 2001 and later, in 2010, South Africa was added to become
BRICS
¾ BRICS grouping brings together the major emerging economies from the world,
comprising 41% of the world population with 3.14 billion people, having 24% of
the world GDP and over 16% share in the world trade.
¾ BRIC group was renamed as BRICS (Brazil, Russia, India, China, South Africa).

Financial ¾ The Financial Action Task Force (FATF) is an inter-governmental body that sets
Action Task international standards.
Force ¾ It was established in 1989, and its Headquarters is in Paris
¾ It aims to prevent these illegal activities and the harm they cause to society
¾ It works as a watchdog for global money laundering and terrorist financing and
stops funding for weapons of mass destruction.
¾ The FATF monitors countries to ensure they implement the FATF Standards fully
and effectively and holds countries accountable that does not comply.

Members of ¾ India has been a member since 2001


Financial ¾ Currently 39 members
Action Task ¾ 37 jurisdictions and
Force ¾ Two regional organizations (the Gulf Cooperation Council and the European
Commission).

Q.35) With reference to Higgs Boson particle recently seen in media, consider the following statements:
1. All matter particles acquire mass only in contact with Higgs field.
2. If the Higgs field disappeared suddenly, all matter would collapse due to dispersion of massless
electrons at the speed of light.
3. Bose-Einstein Condensate is the fifth state of matter formed when elements are superheated.
Which of the statements given above is/are correct?
(a) 1 and 2 only
(b) 2 and 3 only
(c) 1 and 3 only
(d) 1, 2 and 3
EXPLANATION:
The Higgs field is a field of energy that is thought to exist in every region of the universe. The field is
accompanied by a fundamental particle known as the Higgs boson, which is used by the field to
continuously interact with other particles, such as the electron
The Higgs field gives mass to all other particles and every particle in Universe swims due to this field.
Through this interaction, every particle acquires its mass. Particles that are heavier (have a larger

 49
.


mass) than other particles that do not have mass and don't interact with the Higgs field. It gives "the
principle of mass" to all particles (without this field, all particles are exerted with zero mass).
The Higgs field is not considered a force. It cannot accelerate particles, it doesn't transfer energy.
However, it interacts universally with all particles (except the massless ones), providing their
masses. So, Statement 1 is correct.
The Higgs field has an important role in creating and holding atoms and molecules together. All
particles acquire mass only if it interacts with the Higgs field. Particles without interaction do not
acquire mass, weak interaction becomes light, and those that interact intensely become heavy.
When Higgs field disappeared suddenly, a massless particle like the photon travels at the speed of
light. Whereas the electrons are extremely light, with a mass of 0.51 MeV/c2, and it acquires mass from
the Higgs field, a massive particle travels at less than the speed of light. Therefore, all matter would
collapse due to dispersion of massless electrons at the speed of light if this disappears. So, Statement
2 is correct.
Bose-Einstein is referred to as the 'fifth state of matter.' It is a state of matter in which a group of atoms
or subatomic particles are cooled to get absolute zero (-273.15 degrees Celsius, or -460 degrees
Fahrenheit) and not by superheating. So, Statement 3 is not correct.

Q.36) Consider the following statements in respect of a Scramjet Engine and Sounding Rockets :
1. A scramjet engine efficiently operates at hypersonic speeds and allows supersonic combustion.
2. Sounding rockets developed by ISRO are one or two stage gas propellant rockets used for
probing upper atmospheric regions and for space research.
3. BrahMos is a ramjet supersonic cruise missile which is capable of being launched from land,
sea and air.
Which of the statements given above is/are correct ?
(a) 1 and 3 only
(b) 2 only
(c) 3 only
(d) 1, 2 and 3
EXPLANATION:

A ramjet is a form of air-breathing jet engine that uses the vehicle’s forward motion to compress
incoming air for combustion without a rotating compressor. Fuel is injected into the combustion
chamber where it mixes with the hot compressed air and ignites. A ramjet-powered vehicle requires an
assisted take-off like a rocket assist to accelerate it to a speed where it begins to produce thrust. It
works most efficiently at supersonic speeds around Mach 3 (three times the speed of sound) and can
operate up to the speeds of Mach 6. However, the ramjet efficiency starts to drop when the vehicle
reaches hypersonic speeds.
Whereas a scramjet engine is an improvement over the ramjet engine as it efficiently operates at
hypersonic speeds and allows supersonic combustion. In an air-breathing scramjet engine, air from
the atmosphere is rammed into the engine’s combustion chamber at a supersonic speed of more than
Mach two. In the chamber, the air mixes with the fuel to ignite supersonic combustion but the cruiser’s
flight will be at a hypersonic speed of Mach six to seven. So it is called supersonic combustion ramjet
or Scramjet. So, Statement 1 is correct.
Sounding rockets called Rohini was developed by ISRO in 1967 and these are usually one or two-stage
solid (not gas) propellant rockets. They are primarily intended for probing the upper atmospheric

 50
.


regions using rocket-borne instrumentation. They also serve as easily affordable platforms to test or
prove prototypes of new components or subsystems intended for use in launch vehicles and satellites.
So, Statement 2 is not correct

BrahMos is a two-stage missile with a solid propellant booster engine as its first stage, which brings it
to supersonic speed and then gets separated. The liquid ramjet or the second stage then takes a missile
closer to 3 Mach speed in the cruise phase. It is capable of being launched from land, sea and air.
Stealth technology and a guidance system with advanced embedded software provide the missile with
special features. So, Statement 3 is correct.

ADDITIONAL INFORMATION:

HYPERSONIC TECHNOLOGY DEMONSTRATOR VEHICLE


About The Hypersonic Technology Demonstrator Vehicle (HSTDV) is an unmanned
scramjet demonstration aircraft for hypersonic speed flight developed by India's
DRDO. HSTDV is not a weapon itself but is being developed as a carrier vehicle for
hypersonic and long-range cruise missiles.

Scramjet ¾ A scramjet is a form of air-breathing jet engine that uses the vehicle's forward
Engine motion to compress incoming air for combustion without an axial compressor.
¾ Fuel is injected into the combustion chamber, which mixes with the hot
compressed air and ignites.
¾ Scramjets cannot produce thrust at zero airspeeds; they cannot move an aircraft
from a standstill. A scramjet-powered vehicle, therefore, requires an assisted
take-off like a rocket assist to accelerate it to a speed where it begins to produce
thrust.
¾ The scramjet engines operate at hypersonic speeds and allow supersonic
combustion.

Mach number Mach number describes an aircraft's speed compared with the speed of sound in the
air.

 51
.


Q.37) Consider the following statements with reference to NRI investments :


1. An investment made by an Indian entity which is owned and controlled by NRI(s) on a
repatriation basis shall be considered for calculation of Indian foreign investment.
2. RBI’s approval is required for transfer of shares from NRI to a non-resident other than an NRI.
3. Non-Resident investors can invest in specified government securities without being subject to
any investment ceilings.
Which of the statements given above is/are correct ?
(a) 1 only
(b) 2 and 3 only
(c) 1 and 2 only
(d) 3 only
EXPLANATION:

Investments made by NRIs on a non-repatriation basis as stipulated under Schedule IV of Foreign


Exchange Management (non-debt instruments) Rules 2019 are deemed domestic investments at par
with residents' investments.
An investment made by an Indian entity owned and controlled by NRI(s) on a repatriation basis shall
not be considered for calculating Indian foreign investment. So, Statement 1 is not correct.

Non-resident entity’ means a ‘person resident outside India’ and is defined under FEMA.
‘Non-Resident Indian’ (NRI) means an individual resident outside India who is a citizen of India.
As per the Foreign Exchange Management Act, 1999 (FEMA), if the shares are being transferred from
NRI (Non-Resident Indian) to NR (Non-Resident), then it would be compulsory to take the approval of
the Reserve Bank of India. However, there is no prescribed requirement has been made by the RBI for
reporting in this regard under this act. So, Statement 2 is not correct.

The Reserve Bank, in consultation with the Government of India, introduced a separate channel, called
the ‘Fully Accessible Route’ (FAR), to enable non-residents to invest in specified Government of India
dated securities. Eligible investors can invest in specified Government securities without being subject
to any investment ceilings. So, Statement 3 is correct.

 52
.


ADDITIONAL INFORMATION:
NRI INVESTMENTS

About The NRI investment is allowed in mutual funds, real estate and several other industries.
However, NRIs must follow some regulations prescribed under the Foreign Exchange
Management Act (FEMA) for all their stock market investments.

‘FDI’ or ‘Foreign Direct Investment means investment through capital instruments by a person
‘Foreign resident outside India in an unlisted Indian company or ten percent or more of the post-
Direct issue paid-up equity capital on a fully diluted basis of a listed Indian company.
Investment’

Investments with repatriation Investments without repatriation option


option

¾ NRIs may seek prior Non-repatriable investments are those


Methods of permission from the Reserve wherein the principal and gains made by the
investment Bank for investment in sole individual are retained in the country.
proprietorship NRIs are permitted to invest in the following
concerns/partnership firms avenues on a non-repatriable
with a repatriation option. basis they are as follows:
¾ The application will be ¾ Listed equity stocks
decided in consultation with ¾ Futures and options of equities
the Government of India. ¾ IPOs of companies
¾ Mutual funds (equity, debt and hybrid)
¾ Government securities
¾ Treasury bills (T-bills)
¾ Chit Funds
¾ Listed non-convertible or redeemable
debentures
¾ National Pension System

Regulation ¾ Transfer of Shares from a resident Indian to a Non-Resident Indian is regulated by


of Transfer the Foreign Exchange Management Act, 1999 (FEMA).
of Shares ¾ The Reserve Bank of India (RBI) is the main regulatory authority for FEMA
under transactions.
FEMA ¾ With the foreign exchange management act, the main regulation that governs the
transfer of security from a resident Indian to a non-resident Indian is the Foreign
Exchange Management (Transfer or Issue of Security by a Person Resident Outside
India) Regulations, 2017.
¾ Under foreign exchange regulations, the transfer of shares or security is allowed by
a Resident Indian to a Non-Resident Indian.
¾ According to section 5 of the regulations, a person resident outside India should
take prior permission from the RBI for investing. The investment made by the Non-
Resident must be according to the sector caps.
Pricing ¾ The transfer of shares has to be in accordance with the pricing guidelines prescribed
Guidelines from time to time.
for Shares

 53
.


transfer ¾ Companies that follow the pricing guidelines for issue of shares would be allowed to
from retain the share subscription amount. The company maintains this share
Person subscription amount in a Foreign Currency.
Resident in ¾ Prior approval is required from the RBI for maintaining share subscription money.
India to ¾ The principles related to share transfer acts as a two-way process. It applies when
NRI a person Resident in India can transfer shares to a resident outside India (NRI).
Apart from this, it is also applicable where a Non-Resident Indian is allowed to
transfer shares to a resident Indian.

Q.38) Consider the following statements in respect of a Letter of Credit (LoC) and Letter of Undertaking
(LoU) :
1. A letter of credit is more secure because it has the details of the purchase by the importer while
LoU is not linked to the banking system.
2. The RBI banned LoU but not for LoC.
Which of the statements given above is/are correct ?
(a) 1 only
(b) 2 only
(c) Both 1 and 2
(d) Neither 1 nor 2
EXPLANATION:
A letter of Credit is an important tool for international trade.
A letter of Credit is issued by the Bank, which signifies the buyer's creditworthiness. A letter of Credit
is more secure because it has the details of the purchase by the importer, date of issue, expiry date,
the material purchase and other transaction details.
Letter of undertaking (LOU) is a form of bank guarantee under which a bank can allow its customer to
raise money from another Indian bank's foreign branch in the form of short-term Credit. So,
Statement 1 is not correct.
RBI decided to discontinue(ban) the practice of issuance of LoUs/ LoCs (Letter of Comfort) for trade
credits for imports into India.
However, the RBI statement added that banks may continue to issue letters of credit (LCs) and bank
guarantees for trade financing, subject to compliance with RBI guidelines.
So, Statement 2 is correct.

Q.39) With reference to Nuclear Reactors in India, consider the following statements :
1. Madras Atomic Power Station (MAPS) located at Kalpakkam is the first nuclear power station in
India
2. Boron is used for sustaining controlled chain reaction in nuclear reactors.
3. Nuclear poison is a substance which has the capacity for absorbing neutrons in the vicinity of
the reactor core.
Which of the statements given above is/are correct ?
(a) 1 only
(b) 3 only
(c) 2 and 3 only
(d) 1, 2 and 3
 54
.


EXPLANATION:

India's first nuclear reactor was built at Tarapur, Maharastra on 4th August 1956. This reactor was
named Apsara. This was a Boiling Water Reactor (BWR) constructed through Indo-US cooperation.
Whereas India’s third nuclear power station called Madras Atomic Power Station (MAPS), came up at
Kalpakkam, near Chennai. This station was designed and built by India, on its own. All the materials
and equipment were produced in the country. So, Statement 1 is not correct.

Control rods are used in the nuclear reactor to control the chain reaction of nuclear fission by
absorbing the neutrons. Control rods are made out of neutron-absorbing materials such as silver and
boron. So, Statement 2 is correct.

A substance (other than fissionable matter) that has a large capacity for absorbing neutrons in the
vicinity of the reactor core and is intentionally inserted into some types of reactors to decrease the
reactivity of their initial fresh fuel load. This effect may be undesirable in some reactor applications
because it may prevent or disrupt the fission chain reaction, affecting normal operation. These
neutron-absorbing materials are commonly known as nuclear or neutron poison. So, Statement 3 is
correct.

ADDITIONAL INFORMATION:
NUCLEAR REACTIONS

Atoms Atom is the smallest unit of matter. Electron, proton, and neutron are the three
constituent particles of the atom.

Particle Symbol Mass (in kg) Relative charge


Electron e 9.109 389 × 10–31 –1

Proton p 1.672 623 × 10–27 1

Netron n 1.674 928 × 10–27 0

Chemical The formation of a new compound molecule due to the rearrangement of valence
Reaction electrons (The electrons in the outermost shell are known as valence electrons. The
number of its valence electrons determines the valency of an atom) in interacting
atoms and molecules with the release or absorption of energy is called a chemical
reaction.
Example:
C + O2 → CO2 + 4.08 eV

Nuclear In nuclear reactions, the reactants' nuclei, not electrons, interact with each other.
Reactions They result in the formation of new elements. This process is also called the
transmutation of nuclei. During these reactions, a large amount of energy is
released.

Nuclear When slow neutrons hit a heavy nucleus like uranium, it splits into two fragments
Fission with the release of 2-3 neutrons and 200 MeV energy. This process is known as
nuclear fission.

 55
.


Nuclear Chain A chain reaction occurs when more than one emitted neutron induces further fission
Reaction for each primary fission.

Nuclear Nuclear fusion is when two light nuclei combine to form a heavier nucleus. For
Fusion example, four hydrogen nuclei fuse into a helium nucleus by releasing two positrons
(electron-like microscope particles of the same mass but positive charge) and 26.8
MeV energy.

Q.40) Consider the following statements:


1. Amazonia-1 is the first dedicated commercial mission of NewSpace India Limited (NSIL).
2. Antrix Corporation and NewSpace India Limited (NSIL) are under the administrative control of
Department of Space (DoS).
3. Since its inception, ISRO has launched the highest number of foreign satellites compared to
Indian origin.
Which of the statements given above is/are correct ?
(a) 1 only
(b) 2 only
(c) 1 and 3
(d) 1, 2 and 3

 56
.


EXPLANATION:

Amazonia-1 is an earth observation satellite of Brazil, launched in the Indian Launch Vehicle (PSLV
C51) which was the first dedicated commercial Launch for New Space India Limited (NSIL). This forms
the new era of Space reforms in the country.
New Space India Limited (NSIL) is the wholly owned Government of India undertaking/ Central Public
Sector Enterprise (CPSE), under the administrative control of the Department of Space (DOS), Which
is the Commercial arm of the ISRO with the primary responsibility of enabling Indian industries to
take up high technology space related activities and is also responsible for the promotion and
commercial exploitation of the products and services emanating from the Indian space programme.
Likewise, Antrix Corporation Limited (Antrix) is a wholly owned Government of India company under
the administrative control of the Department of Space, ISRO. It is also the commercial arm of ISRO
and it promotes and commercially markets the products and services emanating from the Indian
Space Programme. So, Statements 1 and 2 are correct.

In 1979, Rohini Technology Payload (RTP) was the first Satellite to be placed in orbit by an Indian-
made Satellite launch vehicle (SLV -3 E1). Though it was partially successful, ISRO made a great
beginning. Until February 2022, ISRO has launched a total of 129 satellites of Indian Origin and 342
foreign satellites belonging to 36 countries. Nearly 39 satellites are commercial satellites, and the rest
are Nano-satellites since 1975.

Union Minister of State (Independent Charge) Science & Technology, on February 2022, said India
has 53 operational satellites in space, providing various identified services to the nation. So,
Statement 3 is correct.

Operational
Satellites - 53

Earth Science
Communication Navigation
Observation
Satellite - 21 Satelite - 8 Satellites - 3
Satellite - 21

ADDITIONAL INFORMATION:

IN- SPACe

About ¾ IN SPACe is an autonomous and single window nodal agency in the Department of
Space for the promotion, encouragement, and regulation of space activities of both
government and private entities.
¾ IN-SPACe is supposed to be a facilitator, and also a regulator.
¾ It will act as an interface between ISRO and private parties and assess how best to
utilize India’s space resources and increase space-based activities.

Q.41) Which of the following statements is/are correct ?


1. In Capillary action liquid flow against the force of gravity or magnetic field induction.
2. Completely soluble substances will increase surface tension of a liquid.
 57
.


3. Phototropism, geotropism is caused due to unequal distribution of auxin.


Select the correct answer using the code given below :
(a) 1 only
(b) 1 and 3 only
(c) 2 only
(d) 1, 2 and 3
EXPLANATION:

The phenomenon of rising or depression of liquids in capillary thin tubes or porous materials such as
paper or nonporous material such as liquefied carbon fiber is known as capillary action or capillarity
and this effect can cause liquids to flow against the force of gravity or magnetic field induction. For
example, in the blotting of ink, the spread of water dropped on a cotton cloth and the rising of water
from the roots of a plant to its foliage. So, Statement 1 is correct.

Surface tension is a force per unit length acting in the plane of the interface between the plane of the
liquid and any other substance. It arises due to the excess potential energy of the molecules on the
surface in comparison to their potential energy in the interior.
Adding a solute into a solvent may lead to an increase in surface tension, for example, salt in water and
water in alcohol, due to solute depletion at the interface. The repulsion of the solute from the interface
may originate from electrostatic forces or solute-solvent attraction. So, Statement 2 is correct.
Auxins, gibberellins, and cytokinins are plant growth promoters. Light-induced growth response in
plants is called phototropism. Light controls the distribution of auxins in the shoot tip. A greater quantity
of auxin accumulation occurs in the shaded portion, leading to rapid cell division and faster growth and
the bending or curvature of the stem towards the light. Just like phototropism, geotropism is also caused
by an unequal distribution of auxin.
Geotropism (also called gravitropism) is a response to the stimulus of gravity. For example, if a root is
placed horizontally, the bottom side contains more auxin and grows less - causing the root to grow in
the direction of the force of gravity. The opposite happens in a stem. When a stem is placed horizontally,
the bottom side contains more auxin and grows more - causing the stem to grow upwards against the
force of gravity.
Therefore, both phototropism and geotropism are caused by unequal auxin distribution. So, Statement
3 is correct.

 58
.


Q.42) With reference to Green Manure, consider the following statements :


1. These are crops grown specifically for maintaining soil fertility and structure.
2. Glyricidia, Pongamia and Subabul are used as Green Manure.
3. Green manure usage will subsequently reduce the consumption of Chemical fertilizers in India.
Which of the statements given above is/are correct ?
(a) 1 and 2 only
(b) 2 and 3 only
(c) 1 and 3 only
(d) 1, 2 and 3 only

 59
.


EXPLANATION:

Green, un-decomposed material used as manure is called green manure.


Green manuring is growing in field plants usually belonging to the leguminous family and incorporated
into the soil after sufficient growth. The plants that are grown for green manure are known as green
manure crops.
Green manures are crops grown specifically to maintain soil fertility and structure. So, Statement 1 is
correct.

The Application of green leaves and twigs of trees, shrubs and herbs collected from elsewhere is known
as green leaf manuring.
Forest trees and Plants growing in wastelands, field bunds are the source of green leaf manure
Green leaf manure plant species are neem, mahua, wild indigo, Glyricidia, Karanji (Pongamia glabra)
Calotropis, Avise (Sesbania grandiflora), Subabul and other shrubs and it is not green manure.
The green manurecrop is Dhaincha, Cowpea, Sunhemp and also crops such as summer moong, marsh
pulses and guar act as. So, Statement 2 is not correct

 60
.


Farmers are prevalent in high-intensed agricultural practices so that they will cultivate two-three crops
in a year, which requires lots of chemical fertilisers such as urea, and diammonium phosphate (DAP),
leads to deficiencies of micronutrients (iron and zinc) and thus affecting the productivity of the soil;
therefore, green manuring helps improve soil health and enhance the productivity of the crops. So,
Statement 3 is correct.

ADDITIONAL INFORMATION:
GREEN MANURE

About Green Manure is obtained in two ways: by growing green manure crops or by collecting
green leaf (along with twigs) from plants grown in wastelands, field bunds and forest.
They are normally incorporated back into the soil, either directly or after removal and
composting

Recently Punjab, state agriculture department is promoting the cultivation of green manure and
providing subsidy on the seed at the rate of Rs 2,000 per quintal, which costs Rs 6,300
per quintal without subsidy

 61
.


1.Green manuring improves soil


structure, increases water holding
Advantages
capacity and decreases soil loss by
erosion.

1.Growing of green manure crops in the


off season reduces weed proliferation and
weed growth.
1.Green manuring helps in reclamation of
alkaline soils. Root knot nematodes can be
controlled by green manuring.

Q.43) Consider the following statements :


1. Dark matter attracts and holds the galaxies together but the dark energy repels and causes
expansion of the universe.
2. HAGAR, MACE and Astrosat observatories by India are designed to study Dark Matter and Dark
Energy.
3. Around 95 percent of the universe is made up of "unknown dark matter and dark energy".
Which of the statements given above is/are not correct ?
(a) 1 and 2 only
(b) 1 and 3 only
(c) 2 only
(d) None of the above
EXPLANATION:
Although the names seem to imply that they are similar, dark energy and dark matter are not directly
related. Dark Energy is a form of mysterious energy in space that exerts a negative, repulsive force,
behaving like the opposite of gravity and causing the universe to expand rapidly. Whereas Dark matter
acts as an attractive force of gravity that holds the universe together by acting as an adhesive agent.
So, Statement 1 is correct

 62
.


HAGAR (High Altitude Gamma Ray experiment), an array of seven telescopes, is designed to study very
high energy gamma-ray emissions from celestial objects. HAGAR is a collaborative effort between the
Indian Institute of Astrophysics, Bangalore and the Tata Institute of Fundamental Research, Mumbai.
Major Atmospheric Cherenkov Experiment Telescope (MACE) is the world’s second-largest, ground-
based gamma-ray telescope with a 21-metre-diameter dish to explore the high energy gamma radiation
in the Universe. The project is a collaboration of scientists from BARC, Tata Institute of Fundamental
Research (TIFR) and the Indian Institute of Astrophysics, along with the Electronics Corporation of India
Limited.
Astrosat is a space-based observatory with a single satellite. The satellite can observe the universe in
optical, ultraviolet, low and high energy X-ray regions of the electromagnetic spectrum, which has a
wider range of wavelength bands than other man-made satellites.
Recently, AstroSat has detected UV light from a galaxy called AUDFs01, which is 9.3 billion light-years
away from Earth.
HAGAR and MACE observatories are used to observe Gamma-ray emissions, and Astrosat is a Space
observatory satellite. So, these Observatories does not design to study dark matter and dark energy.
So, Statement 2 is not correct.

Dark energy makes up approximately 68% of the universe and appears to be associated with the vacuum
in space. Whereas Dark matter makes up only about 27% of the universe. But, the remaining 5 % is
the normal matter that makes up all stars and galaxies of the universe. Hence, around 95 percent of
the universe is made up of "unknown dark matter and dark energy". So, Statement 3 is correct.

ADDITIONAL INFORMATION:

DARK MATTER AND DARK ENERGY

About ¾ In 1998, "dark energy" was discovered, a mysterious repulsive force accelerating the
universe's expansion.
¾ Dark energy is the mysterious form of energy that makes up about 68% of the
universe.
¾ Despite both components being invisible, dark Matter came into existence by early
as the 1920s.

Dark Matter LUX-Zeplin PandaX-xT


and Energy
A next-generation Dark matter It is a Dark energy project at China’s Jinping
Experiments
experiment at the Sanford Underground Laboratory.
Underground Research Facility.

Q.44) With reference to Rules of origin (ROO) recently seen in news, consider the following statements:
1. Article 1 of the WTO Agreement defines the Rules of Origin.
2. It is the determination of the nationality of a particular good.
3. ROO are used to implement measures and instruments of commercial policy such as anti-
dumping duties and safeguard measures.
4. Based on ROO, the imported products may receive Most-Favoured Nation (MFN) or Preferential
treatment
Which of the statements given above are correct ?
(a) 1, 2 and 3

 63
.


(b) 1, 2 and 4
(c) 1 and 2
(d) 1, 2, 3 and 4
EXPLANATION:

“Rules of origin” are the criteria used to define where a product was made. Those rules define the
economic nationality of the goods.
Article 1 of the Rules of Origin Agreement defines rules of origin as those laws, regulations and
administrative determinations of general application applied to determine the country of origin of
goods, except those related to granting tariff preferences. So, Statement 1 is correct.

Rules of origin are the criteria needed to determine the national source of a product. Their importance
is derived from the fact that duties and restrictions depend upon the source of imports in several cases.
So, Statement 2 is correct.

Rules of origin are used to implement measures and instruments of commercial policy such as anti-
dumping duties and safeguard measures. So, Statement 3 is correct.

“Most-Favoured-Nation” (“MFN”) treatment requires members to accord the most favourable tariff and
regulatory treatment given to the product of any one Member at the time of import or export of “like
products” to all other Members.
Rules of origin determine whether imported products shall receive most-favoured-nation (MFN) or
preferential treatment. So, Statement 4 is correct.

ADDITIONAL INFORMATION:

RULES OF ORIGIN(ROO)

About ¾ Determining where a product comes from is difficult when raw materials and
parts crisscross the globe to be used as inputs in scattered manufacturing
plants.
¾ Rules of origin are therefore needed to attribute one country of origin to each
product. They are the criteria used to define where a product was made and are
important for implementing other trade policy measures, including trade
preferences (preferential rules of origin), quotas, anti-dumping measures and
countervailing duties (non-preferential rules of origin).
¾ Non-preferential rules of origin apply in the absence of any trade preference, that
Non- is, when trade is conducted on a most-favoured-nation basis. Not all countries
preferential apply specific legislation related to non-preferential rules of origin.
rules of origin ¾ However, some trade policy measures such as quotas, anti-dumping or “made
in” labels may require a determination of origin and, therefore, the application of
non-preferential rules.
¾ Preferential rules of origin apply in reciprocal trade preferences (i.e., regional
trade agreements or customs unions) or non-reciprocal trade preferences (i.e.,
preferences in favour of developing or least-developed countries).
¾ The rules of origin which apply under reciprocal trade preferences or regional
Preferential
trade agreements must conform with the general disciplines of Annex II of the
rules of origin
Agreement on Rules of Origin.

 64
.


¾ In addition, the GATT and the Agreement on Trade Facilitation contain some
provisions related to origin requirements.

¾ Rules of origin are used:


Uses of Rules ¾ To implement measures and instruments of commercial policy such as anti-
of origin dumping duties and safeguard measures;
¾ To determine whether imported products shall receive most-favoured-nation
(MFN) treatment or preferential treatment;
¾ For trade statistics;
¾ For the application of labelling and marking requirements; and
¾ For government procurement

Q.45) With reference to Bisphenol A (BPA), consider the following statements :


1. It is a colourless crystalline solid insoluble in organic solvents.
2. It is used to manufacture polycarbonate plastics and resins.
Which of the statements given above is/are correct ?
(a) 1 only
(b) 2 only
(c) Both 1 and 2
(d) Neither 1 nor 2
EXPLANATION:

Bisphenol A (BPA) is a chemical compound primarily used to manufacture various plastics. It is a


colorless solid soluble in most common organic solvents but has very poor solubility in water. So,
Statement 1 is not correct.
It is primarily used in large quantities in the production of polycarbonate plastics. It is found in various
products, including shatterproof windows, eyewear, water bottles, and epoxy resins that coat metal
food cans, bottle tops, and water supply pipes. So, Statement 2 is correct.

ADDITIONAL INFORMATION:

BISPHENOL A (BPA)

About ¾ Bisphenol A is an industrial chemical widely used to produce polycarbonate


plastics and epoxy resins.
¾ Polycarbonate plastics are mostly used in containers that store foods and
beverages.
¾ Epoxy resins coat the inside of metal products, such as food cans, bottle tops,
and water supply lines. Some dental sealants and composites also may
contain BPA
Health ¾ It may cause male and female infertility, precocious puberty, hormone-dependent
concerns tumors such as breast and prostate cancer, and several metabolic disorders,
including polycystic ovary syndrome (PCOS)
¾ Daily constant exposure leads to bioaccumulation.
¾ It may lead to possible health effects on the brain and prostate gland of fetuses,
infants, and children
¾ It may increase blood pressure, type 2 diabetes, and cardiovascular disease.
 65
.


Q.46) Consider the following statements regarding the Singularity Theory about the black hole :
1. Inside a black hole, matter is compressed down to an infinitely tiny point.
2. All conceptions of time and space completely break down at the center of the black hole.
3. Sun is a massive star and can turn into a black hole at end of the life cycle.
Which of the statements given above are correct ?
(a) 1 and 2 only
(b) 1 and 3 only
(c) 2 and 3 only
(d) 1, 2 and 3
EXPLANATION:
A black hole is a place in space where gravity pulls so much that even light cannot get out. Gravity is
so strong because matter has been squeezed into a tiny space. This can happen when a star is dying.
According to the Singularity theory, at the centre of a black hole is the ultimate no man's land, which
is a place where matter is compressed down to an infinitely tiny point, and all conceptions of time and
space completely break down. So, Statements 1 and 2 are correct.

The Lifecycle of Star is as follows:

Our Sun is an average-sized star. The sun is much smaller in size to become a black hole at the end
of the life cycle. The Sun would need to be about 20 times more massive to end its life as a black hole.
Stars that are born this size or larger can explode into a supernova at the end of their lifetimes before
collapsing back into a black hole, an object with a gravitational pull so strong that nothing, not even
light, can escape. The life cycle of Sun will end up as a white dwarf not as Black hole. So, Statement
3 is not correct.

ADDITIONAL INFORMATION:
BLACK HOLES

Size ¾ Black holes can be big or small.

 66
.


¾ The largest black holes are called "supermassive." These black holes have masses
that are more than 1 million suns together. Scientists have found proof that every
large galaxy contains a supermassive black hole at its center.
¾ The supermassive black hole at the center of the Milky Way galaxy is called
Sagittarius A. It has a mass equal to about 4 million suns and would fit inside a very
large ball that could hold a few million Earths.

Formation ¾ Scientists think the smallest black holes formed when the universe began.
¾ Stellar black holes are made when the center of a very big star falls in upon itself or
collapses. When this happens, it causes a supernova. A supernova is an exploding
star that blasts part of the star into space.
¾ Scientists think supermassive black holes were made at the same time as the galaxy
they are in.

Recently ¾ Scientists from the Event Horizon Telescope(EHT) facility, a collaboration of over 300
in news researchers, revealed the first image of the black hole at the center of the Milky Way.
¾ This image of the black hole referred to as Sagittarius A* (SgrA*) gave further support
to the idea that the compact object at the center of our galaxy is indeed a black hole.
This strengthens Einstein’s general theory of relativity.

Q.47) Consider the following statements about Biomolecules :


1. Carbohydrates are linear chains connected by peptide bonds
2. Insulin in human body is a fibrous protein
3. Triglycerides is a type of fat found in butter
Select the correct answer using the code given below :
(a) 1 and 2 only
(b) 2 and 3 only
(c) 3 only
(d) 1, 2 and 3
EXPLANATION:
¾ Amino acids are the building blocks of proteins which are polypeptides.
¾ A peptide bond is a covalent bond formed between two amino acids. Proteins are linear chains
of amino acids linked by peptide bonds.
¾ A carbohydrate is a large biological molecule, or macromolecule, consisting only of carbon (C),
hydrogen (H), and oxygen (O), usually with a hydrogen: oxygen atom ratio of 2:1. Carbohydrates
are technically hydrates of carbon, structurally it is more accurate to view them as polyhydroxy
aldehydes and ketones.
¾ Carbohydrates are monosaccharides linked into polysaccharide chains by a type of covalent
bond known as a glycosidic bond. So, Statement 1 is not correct.

 67
.


¾ Fibrous proteins are polypeptide chains that run parallel and are held together by hydrogen and
disulphide bonds; a fiber-like structure is formed. Such proteins are generally insoluble in water.
Keratin (present in hair, wool, silk), myosin (present in muscles), etc., are fibrous proteins.
¾ Insulins are Globular proteins where the chain of polypeptides coil around and give a spherical
shape. These are usually soluble in water. So, Statement 2 is not correct

Triglycerides are also called neutral fats. They are essential for providing energy to the body and a
vehicle of energy storage, primarily in adipose tissue. They are mainly found in the vegetable oils,
such as corn (maize), olive, palm, and sunflower, and animal fats, such as tallow, lard and butter.
So, Statement 3 is correct.

ADDITIONAL INFORMATION:

VITAMINS
About ¾ Vitamins are small amounts of organic compounds in the diet to perform specific
biological functions for the normal maintenance of optimum growth and health of

 68
.


the organism. Vitamins are designated by alphabets A, B, C, D, etc. Some of them


are further named as sub-groups e.g., B1, B2 , B6 , B12, etc
¾ Most vitamins cannot be synthesised in our bodies, but plants can synthesise
almost all of them, so they are considered essential food factors. However, the
bacteria in the gut can produce some of the vitamins required by us

Fat-soluble Vitamins that are soluble in fat and oils but insoluble in water are kept in this group.
vitamins These are vitamins A, D, E and K. They are stored in the liver and adipose tissues.

Water B-group vitamins and vitamin C are soluble in water, so they are grouped. Water soluble
soluble vitamins must be supplied regularly in the diet because they are readily excreted in
vitamins urine and cannot be stored (except vitamin B12) in our body.

VITAMIN DEFICIENCY DISEASES

Name of vitamins Source Deficiency diseases

Vitamin A Fish liver oil, carrots, butter Xerophthalmia (hardening of the cornea
and milk of the eye), Night blindness

Vitamin B1 (Thiamine) Yeast, milk, green BeriBeri (loss of appetite, retarded


vegetables and cereals growth)

Vitamin B2 (Riboflavin) Milk, egg white, liver, Cheilosis (fissuring at corners of mouth
kidney and lips), digestive disorders and burning
sensation of the skin.

Vitamin B6 (Pyridoxine) Yeast, milk, egg yolk, Convulsions


cereals and grams

Vitamin B12 Meat, fish, egg and curd Pernicious anemia (RBC deficient in
haemoglobin)
Vitamin C (Ascorbic acid) Citrus fruits, amla and Scurvy (bleeding gums)
green leafy vegetables

Vitamin D Exposure to sunlight, fish Rickets (bone deformities in children) and


and egg yolk osteomalacia (soft bones and joint pain in
adults)

Vitamin E Vegetable oils like wheat Increased fragility of RBCs and muscular
germ oil, sunflower oil, etc. weakness

Vitamin K Green leafy vegetables Increased blood clotting time

Q.48) Which of the following constitute(s) defence mechanisms in the Human Body ?
1. Immunity to defend the body from infections.
2. Stoppage of bleeding (Haemostasis) to prevent blood loss.
3. Resistance to stress mainly through release of hormones.
Select the correct answer using the code given below :
(a) 1 only
(b) 1 and 2 only
(c) 1 and 3 only
(d) 1, 2 and 3

 69
.


EXPLANATION:

The generalized primary forms of host defense are termed "innate," "inborn," or ''nonspecific" immunity.
These initial defensive mechanisms guard the body by contributing protective responses that are
effective against a diverse variety of threats. Generally, the defence mechanisms in our body are as
follows,
¾ Immunity to defend the body from infections
¾ Metabolic defence to metabolize and detoxify foreign chemicals
¾ Stoppage of bleeding (Hemostasis) to prevent blood loss
¾ Resistance to stress mainly through release of the hormone
So, Option (d) is correct.

ADDITIONAL INFORMATION:

IMMUNITY
Innate A healthy individual is generally immune to potentially harmful microorganisms by several
very effective mechanisms. These mechanisms are termed innate or natural immunity.
Innate defence consists of three main components,
¾ Physical barriers (preventing entry of germs)
¾ Phagocytic cells and (Dealing with germs that enter)
¾ Soluble components (complement)

Acquired It is the immunity mediated by lymphocytes and characterized by antigen specificity and
memory. An acquired immunity may be brought about in an individual in two main ways,
¾ By infection, antibodies are produced against the infective agent and by deliberate
artificial immunization. This is termed as actively acquired immunity
¾ By transfer from an actively immunized individual through blood, serum components,
etc. This is called passively acquired immunity

 70
.


Q.49) Consider the following statements related to Droughts :


1. ‘Drought in Numbers 2022’ report released by United Nations Convention to Combat
Desertification.
2. In India, drought early warning and forecasting is provided by the Central Water Commission.
3. Globally, Droughts account for nearly 15 percent of Natural disasters.
Which of the statements given above is/are correct ?
(a) 1 only
(b) 1 and 2 only
(c) 2 and 3 only
(d) 1 and 3 only
EXPLANATION:
The Drought in Numbers 2022 report was released on May 11, 2022, at the 15th Conference of Parties
(CoP15) to the United Nations Convention to Combat Desertification (UNCCD) for the global
commitment to drought preparedness and resilience in all global regions.
The latest assessment analysed droughts and their impacts on life and livelihood over 122 years,
covering 196 countries. So, Statement 1 is correct.

Drought is a recurrent natural feature resulting from the lack of precipitation over an extended period
(e.g., a season or several years).
India Meteorological Department (IMD) is the designated agency for providing Drought early warning
and forecasting.
Hence, Central Water Commission is responsible for providing Flood Forecasting & Warning services
in India. So, Statement 2 is not correct.

Droughts were also the deadliest natural disaster on a global scale, and they represent 15 percent of
natural disasters. So, Statement 3 is correct.

ADDITIONAL INFORMATION:

DROUGHTS
¾ Droughts were also the deadliest natural disaster.
¾ Drought causes economic, environmental and social impacts.
About ¾ UN report says that drought frequency and duration have increased 29 percent
since 2000.
¾ Land degradation, in addition to Climate change, increases droughts.
¾ Northern Italy is facing its worst Drought in 70 years, and more than 43% of US
states are currently experiencing droughts.
¾ Economic losses due to Drought have increased multifold in the past decades.
The scientific ¾ There is strong evidence that human-induced climate change has led to an
consensuson increased risk of Drought.
Drought ¾ There is an increase in average surface temperatures around the world by human
activities.

Drought is ¾ From 1970 to 2019, Drought was one of the hazards that led to the largest
deadly human losses, with a total of approximately 650,000 deaths. Among all the
climate-related deaths, more than 90 percent occurred in developing countries.

 71
.


Drought is ¾ An estimated 55 million people are directly affected by droughts every year,
devastating making it the most serious hazard to livestock and crops in nearly every part of
the world.

Turning the ¾ Limiting global warming to 1.5 degrees Celsius, along with regenerative land and
tide improved water management practices, is expected to reduce the probability of
extreme drought events substantially.

Drought ¾ In the past century, 45 major drought events occurred in Europe, affecting
around the millions of people and resulting in more than USD 27.8 billion in economic
world (1900- losses. Today, an annual average of 15 percent of the land area and 17 percent
2022) of the population within the European Union is affected by Drought.
¾ Over the past century, the highest total number of humans affected by Drought
was in Asia.
¾ Severe Drought affects Africa more than any other continent, with more than 300
events recorded in the past 100 years, accounting for 44 percent of the global
total. More recently, sub-Saharan Africa has experienced the dramatic
consequences of climate disasters becoming more frequent and intense

United Nations ¾ The United Nations Convention to Combat Desertification (UNCCD), adopted in
Convention to 1994, is the legally binding international agreement linking environment and
Combat development to sustainable land management.
Desertification ¾ The Convention addresses specifically the arid, semi-arid, and dry sub-humid
(UNCCD) areas, known as the drylands.

COP ¾ Parties to the Convention meet in Conferences of the Parties (COPs) every two
(Conference of years, as well as in technical meetings throughout the year, to advance the aims
Parties) and ambitions of the Convention and achieve progress in its implementation.
¾ Desertification is a process of degradation of the land. Desertification is the
Desertification degradation process by which a fertile land changes itself into a desert by losing
its flora and fauna; this can be caused by Drought, deforestation, climate change,
human activities, or improper agriculture.

UNCCD 2018- ¾ UNCCD 2018-2030 Strategic Framework is the most comprehensive global
2030 Strategic commitment to achieve Land Degradation Neutrality (LDN) to restore the
Framework productivity of vast expanses of degraded land, improve the livelihoods of more
than 1.3 billion people, and reduce the impacts of Drought on vulnerable
populations to build.

Floods and ¾ Central Water Commission (CWC) issues flood forecasts as a non-structural
Drought measure of flood management throughout the country.
Forecasting ¾ CWC also issues inflow forecasts to identified reservoirs for proper reservoir
regulations during floods

¾ Central Water Commission is a premier Technical Organization of India in the


field of Water Resources. It is presently functioning as an attached office of the
Ministry of Jal Shakti, Department of Water Resources, River Development and
Ganga Rejuvenation, Government of India.
Central Water
¾ The Commission is entrusted with the general responsibilities of initiating,
Commission
coordinating and furthering, in consultation with the State Governments

 72
.


concerned, schemes for control, conservation and utilization of water resources


throughout the country for Flood Control, Irrigation, Navigation, Drinking Water
Supply and Water Power Development.
Q.50) Consider the following statements regarding Antimicrobial resistance :
1. Antibiotic resistance does not occur naturally, but misuse of antibiotics in humans and animal
feed is the main cause.
2. Antibiotic Resistance in Bacteria occurs when the changes in bacteria cause the drugs used to
treat the infection to become less effective.
3. The usage of Colistin in poultries is banned in India.
Which of the statements given above is/are correct?
(a) 1 and 2 only
(b) 1 and 3 only
(c) 2 and 3 only
(d) 1, 2 and 3
EXPLANATION:

Antibiotic / Antimicrobial resistance happens when germs like bacteria and fungi develop the ability
to defeat the drugs designed to kill them. That means the germs are not killed and continue to grow.
Resistant infections can be difficult, and sometimes impossible, to treat. But this resistance is natural,
whereas misuse of antibiotics in humans and animals only accelerates the process. So, Statement 1
is not correct.

Drugs/ Medicines are designed to kill the bacteria which infect human beings. Antibiotic resistance
(ABR) is developed in bacteria when they develop the ability to survive exposure to antibiotics designed
to kill them. So, Statement 2 is correct.

Colistin is an antibiotic for therapeutic purposes in veterinary. But the drug is highly misused in the
poultry industry as a growth promoter for prophylactic purposes. One of the reasons for antibiotic
resistance in India is the unwanted use of Colistin in the poultry industry. In 2019, the Health Ministry
of India banned the manufacture, sale, and distribution of the antibiotic, Colistin and its formulations
for food-producing animals, poultry, aqua farming, and animal feed supplements in a bid to preserve
the drug’s efficacy in humans under provisions of the Drugs and Cosmetics Act, 1940. So, Statement
3 is correct.

ADDITIONAL INFORMATION:

ANTIMICROBIAL RESISTANCE

WHO The “Global action plan on antimicrobial resistance” has 5 strategic objectives,
Strategic ¾ To improve awareness and understanding of antimicrobial resistance.
Objectives ¾ To strengthen surveillance and research.
¾ To reduce the incidence of infection.
¾ To optimize the use of antimicrobial medicines.
¾ To ensure sustainable investment in countering antimicrobial resistance.

The Global The WHO-supported system supports a standardized approach to the collection,
Antimicrobial analysis, and sharing of data related to antimicrobial resistance at a global level to
Resistance inform decision-making, and drive local, national, and regional action.
Surveillance

 73
.


System
(GLASS)

Q.51) Which of the following statements about Gene Bank are correct ?
1. Gene banks are a type of biorepository that preserves genetic material
2. World’s Second largest Gene Bank is located in India
3. Gene Bank is a method of in-situ Conservation of flora and fauna.
Select the correct answer using the code given below :
(a) 1 and 2 only
(b) 1 and 3 only
(c) 2 and 3 only
(d) 1, 2 and 3
EXPLANATION:
In a gene bank, various clones of bacteria carrying the desired genes in their DNA can be stored and
preserved at very low temperatures for future use. A gene bank, a gene library, or a DNA library is,
thus, a collection of bacterial or bacteriophage (virus) clones. Thus Gene banks are a type of
biorepository that preserves genetic material. So, Statement 1 is correct.

The National Bureau of Plant Genetic Resources has set up the world’s second-largest refurbished
gene bank in New Delhi, which is currently home to the germplasms of 452,000 seeds. It can safely
store up to one million germplasms and greatly help the farmers and the scientific community. So,
Statement 2 is correct.

Ex situ Conservation is the Conservation and maintenance of plant samples outside their natural
habitat, either in the form of the whole plant or as a seed, pollen, and tissue or cell culture. Gene
bank is one of the methods of Ex situ Conservation (and not in situ). So, Statement 3 is not correct.
ADDITIONAL INFORMATION:

GENETIC MATERIALS

About ¾ A cell contains a nucleus. The nucleus contains chromosomes; Chromosomes


bear genes. Genes carry hereditary information.
¾ A zygote has the information for the development and differentiation of the
embryo in its genes.
¾ The cells of an individual have the genes for maintaining their structure and
function

Nucleus ¾ A nucleus is a membrane-enclosed organelle within a cell that contains


chromosomes.

 74
.


Chromosome ¾ Chromosomes are threadlike structures made of protein and a single molecule
of DNA that serve to carry the genomic information from cell to cell.
¾ In plants and animals (including humans), chromosomes reside in the
nucleus of cells.
¾ Humans have 22 pairs of numbered chromosomes (autosomes) and one pair
of sex chromosomes (XX or XY), for a total of 46.
¾ Each pair contains two chromosomes, one coming from each parent, which
means that children inherit half of their chromosomes from their mother and
a half from their father.

 75
.


Q.52) Consider the following statements about Space Program of India :


1. Gaganyaan will be the first human-spaceflight program by ISRO
2. The Vyomanauts of Gaganyaan program will be launched by GSLV-MK III
3. Rakesh Sharma is the first and only Indian to travel to space.
Which of the statements given above is/are correct ?
(a) 1 only
(b) 1 and 2 only
(c) 2 and 3 only
(d) 1, 2 and 3
EXPLANATION:

The ISROs program Gaganyaans Objective is to demonstrate the indigenous capability to undertake a
human space flight mission to Lower Earth Orbit (LEO) with a crew of three astronauts to 400 km Low
Earth Orbit. This mission will provide a sustained Indian human space exploration program in the
long run. As part of this program, two uncrewed and one human-crewed mission are approved by the
Government of India (GoI).So, Statement 1 is correct.

The Astronauts will be addressed as "Vyomnauts" in the Gaganyaan Manned Mission 2022. Human-
rated GSLV MkIII is identified as the launch vehicle for the Gaganyaan mission. The Gaganyaan
program aims to carry three Crew to Low Earth Orbit (LEO) and bring them back safely to a
predetermined location on Earth. So, Statement 2 is correct.

Rakesh Sharma an Indian citizen was the first astronaut to go into space. Other Indians who travelled
to space includes Kalpana Chawla and Sunitha Williams.
Thus, Rakesh Sharma was the first and only Indian to travel to space in a Soviet rocket in April 1984.
So, Statement 3 is correct.

 76
.


ADDITIONAL INFORMATION:
GAGANYAN MISSION

About ¾ The Gaganyaan program envisages the demonstration of human spaceflight


with a crew of three astronauts to 400 km LEO.
¾ The Human-rated GSLV MKIII is identified as the launch vehicle for the
Gaganyaan mission. The Gaganyaan program is paramount to crew safety, and
all efforts will be put in to ensure the crew's safe and comfortable travel to space
and back to earth.
¾ This program will be implemented by synergizing the expertise available within
ISRO Centres, National institutions/agencies, academia and industries.
¾ The Gaganyaan Programme envisages laying the foundation for a sustained
Indian human space exploration program in the long run.
¾ The program thereby fulfills the aspirations of an established space-faring
nation.
New ¾ The human-rated launch vehicle
Technologies ¾ Crew escape systems
Used in ¾ Habitable orbital module
Gaganyan ¾ Life support system
Mission ¾ Crew selection and training and associated crew management activities.

Collaborating ¾ Indian Armed Forces


Partners ¾ Defense Research Development organization
¾ Indian maritime agencies - Indian Navy, Indian Coast Guard, Shipping
Corporation of India, National Institute of Oceanography, National Institute of
Ocean Technology.
¾ Indian Meteorological Department
¾ CSIR Labs
¾ Academic institutes
¾ Industry partners
GSLV MK III
About ¾ GSLV MKIII Project was approved in 2002, with a mandate of achieving the
capability to launch a 4-ton class satellite to Geosynchronous orbit by realizing
an indigenously developed launch vehicle.
¾ The development program has been completed through three successful flights
LVM3 X,GSLV MkIII D1 and GSLV MkIII D2.
¾ GSLV MkIII is configured as a three-stage vehicle with two solid strap-on motors
(S200), one liquid core stage (L110) and a high-thrust cryogenic upper stage
(C25).

Characteristics ¾ The performance capability of 4.3 ton to geosynchronous transfer orbit or


geostationary transfer orbit (GTO)
¾ Payload capability to support 10-ton to Low Earth Orbit (LEO) missions
¾ Cost-effective
¾ Improved reliability, operability and redundancy management
¾ The future growth potential of payload with minimal design changes
¾ To support man-rated missions of the Indian Space Programme
 77
.


Q.53) With reference to Human Papillomavirus (HPV), consider the following statement :

1. It is a single-stranded Ribonucleic acid that affects human beings.


2. Cervical cancer is primarily caused due to HPV.
3. Gardasil is an indigenously developed HPV vaccine in India.
Which of the statements given above are correct?
a) 1 and 2 only
b) 2 and 3 only
c) 2 only
d) 1 and 3 only
EXPLANATION:

Human Papillomavirus is Double stranded deoxyribonucleic acid Virus that affects human beings.
HPV is a group of more than 200 related viruses which can be transmitted through sexual and Non-
sexual modes. Non-sexual transmission includes direct skin-to-skin contact.
Sexually transmitted HPV types are classified into Low risk and High Risk. There are nearly 14 high-
risk HPV types. In these, HPV 16 and HPV 18 are responsible for most HPV-related cancers like Cervical
cancer, Oropharyngeal cancer, Vaginal cancer, Penile cancer, and Anal cancer.
Thus, Human Papillomavirus is Double stranded deoxyribonucleic acid, not a Single-stranded
Ribonucleic acid Virus.
So, Statement 1 is not correct.

The cervix is a hollow cylinder that connects the lower part of a woman's uterus to her vagina.

Cervical cancer, mainly caused by Human Papillomavirus infection, is the leading cancer in Indian
women and the second most common cancer in women worldwide.
Most human papillomavirus infection is harmless but persistent infection with high-risk human
papillomavirus (especially type 16 HPV) can cause cancer of the cervix, vulva, vagina, anus, penis, and
oropharynx. The two most common "high-risk" genotypes (HPV 16 and 18) cause approximately 70%
of all cervical cancers.
Though there are several methods of prevention of cervical cancer, prevention by vaccination is
emerging as the most effective option, with the availability of vaccines namely.
¾ Gardasil 9
 78
.


¾ Cervavac
Gardasil 9 is an India’s first gender-neutral HPV Vaccine which is a manufactured by Merck & Co
whereas Cervavac is the India’s first indigenously developed quadrivalent human papillomavirus
vaccine for prevention of cervical cancer developed by the Pune-based Serum Institute of India in
coordination with the Government of India’s Department of Biotechnology (DBT).
So, Statement 2 is correct and Statement 3 is not correct.

Q.54) Which of the below submarines are part of Indian Navy’s ‘Project 75’ Programme ?
1. INS Kalvari
2. INS Arihant
3. INS Karanj
4. INS Sindhukirti
5. INS Vagsheer
Choose the correct answer from the options below :
(a) 1, 2, 3 and 4
(b) 1, 3 and 5
(c) 1, 2, 3, 4 and 5
(d) 1, 2, 4 and 5
EXPLANATION:

Project 75 is a programme by the Indian navy which envisaged building six diesel-electric submarines
based on Scorpene-class submarines. Which includes the following;
¾ INS Kalavari - has been commissioned into the Indian navy
¾ INS Khanderi - has been commissioned into the Indian navy
¾ INS Kharanj - has been commissioned into the Indian navy
¾ INS Vela – trails are underway
¾ INS Vahir – trails are underway
¾ INS Vagsheer – construction is underway
So, statements 1, 3, and 5 are correct.

INS Arihant is India’s first indigenously nuclear submarine under the secretive advanced technology
vessel programme. So, Statement 2 is not correct.

INS Sindhukirti is the oldest operational submarine in the Indian navy. It is Kilo-class submarine and
can launch torpedoes, missiles, and even mines. So, Statement 4 is not correct.

ADDITIONAL INFORMATION:
Project 75I ¾ Project 75(India)- class submarines or P-75I for a short-planned class of diesel-
electric submarines to be built for the Indian navy. The P-75I class is a follow-
on of the P-75 class submarines of the Indian navy.
¾ Under this project, the Indian navy intends to acquire six conventional diesel-
electric submarines.
¾ All six submarines are expected to be constructed in India under the make in
India initiative.
India’s nuclear ¾ INS Arihant will complete its nuclear triad. The air and land wings of the triad
triad Mirage-2000s and Agni ballistic missile were already in place.

 79
.


Q.55) With reference to Plant and Animal cells, consider the following statement:
1. Centriole is present in the animal cell but generally absent in plant cell.
2. Carbohydrates are stored in the animal in form of starch and plants in the form of glycogen.
Which of the statements given above is/are correct ?
(a) 1 only
(b) 2 only
(c) Both 1 and 2
(d) Neither 1 nor 2
EXPLANATION:

Centrioles are paired barrel-shaped organelles located near the nuclear envelope in the cytoplasm of
animal cells. They play a role in organizing microtubules that serve as the cell's skeletal system. They
help determine the locations of the nucleus and other organelles within the cell and help in Cell Division
and Locomotion.

Plant and animal cells are different as the former possess cell walls, plastids, and a large central
vacuole which are absent in animal cells. On the other hand, animal cells have centrioles which are
absent in almost all plant cells. So, Statement 1 is correct.

Carbohydrates are one of the major forms of energy for animals and plants. During the process of
Photosynthesis, plants build carbohydrates using light energy from the Sun. whereas animals eat
plants or other animals to obtain carbohydrates.
Plants store carbohydrates in long polysaccharide chains called Starch, while animals store
carbohydrates as the molecule glycogen. These large polysaccharides contain many chemical bonds
and therefore store much chemical energy. When these molecules are broken down during metabolism,
the energy in the chemical bonds is released and can be harnessed for cellular processes. Thus, plants
store carbohydrates in the form of Starch and animals in the form of Glycogen, and not vice versa. So,
Statement 2 is not correct.

ADDITIONAL INFORMATION:
CELL

About ¾ The cell is the fundamental structural and functional unit of all living organisms
¾ Anton Von Leeuwenhoek first described a live cell.
¾ Robert Brown later discovered the nucleus.
¾ The invention of the microscope and its improvement leading to the electron microscope
revealed all the structural details of the cell.

 80
.


Animal
Cell

Plant Cell

Q.56) With reference to Launch Vehicles of ISRO, consider the following statement :
1. GSLV MK-III uses the most powerful cryogenic engine developed by India.
2. PSLV uses India’s first Indigenous Cryogenic engine.
3. SSLV uses both solid and liquid propellants.
Which of the statements given above is/are correct ?

 81
.


(a) 1 and 3 only


(b) 2 and 3 only
(c) 2 only
(d) 3 only
EXPLANATION:

India’s Most Powerful Launch vehicle, GSLV Mk. III is a three-stage launch vehicle is an evolved version
of India's Geosynchronous Satellite Launch Vehicle, developed and operated by the Indian Space
Research Organization.
The vehicle is designed to carry 4-ton class of satellites into Geosynchronous Transfer Orbit (GTO) or
about 10 tons to Low Earth Orbit (LEO).GSLV MkIII successfully injected Chandrayaan-2 and
Gaganyaan is yet to launch in this launch vehicle.

GSLV MkIII is configured as a three-stage vehicle with


¾ Two solid strap-on motors (S200) – Largest Solid Boosters in the world with 204 tonnes of Solid
Propellant.
¾ One liquid core stage (L110) – Twin Liquid Engine
¾ High thrust cryogenic upper stage (C25) – Fully Indigenous high thrust Cryogenic engine with a
propellant loading of 28 tons.
So, statements 1 is correct.

PSLV (Polar Satellite Launch Vehicle) is an indigenously-developed launch vehicle of the ISRO. PSLV
isa medium-lift launcher with a reach up to various orbits, including the Geo Synchronous Transfer
Orbit, Lower Earth Orbit, and Polar Sun Synchronous Orbit.
PSLV has a four-stage system comprising a combination of solid and liquid-fuelled rocket stages.
¾ The first stage at the very bottom is solid fuelled having six strap-on solid rocket boosters wrapped
around it.
¾ Second stage is liquid fuelled whereas the third stage has a solid fuelled rocket motor.
¾ At the fourth stage, the launcher uses a liquid propellant to boost in the outer space.
GSLV Mk III is the first launch vehicle which uses the cryogenic engine whereas Polar Satellite Launch
Vehicle doesn’t use cryogenic engines.
So, Statement 2 is not correct.

 82
.


Small Satellite Launch Vehicle (SSLV) is a 3 stage Launch Vehicle configured with three Solid fuel-
based stages and a liquid fuel-based velocity trimming module (VTM) which is used to place satellites
in the orbit. The SSLV is specifically designed to launch Satellites to serve the emerging global market
for launching small satellite.

Thus, SSLV uses Solid and Liquid Propellants whereas it doesn’t use Cryogenic propellants. So,
Statement 3 is correct.

Q.57) Consider the following statements about Satellite Based Internet Systems :
1. Satellite internet constellations like starlink provide lesser latency than fiber optic internet.
2. Internet services provided by satellite constellations could be affected by moisture and
precipitation.
3. 'Responsible Space' is an initiative by Starlink to outline the approaches to promote
sustainability and safe operation of satellite systems in space.
Which of the statements given above is/are correct?
(a) 1 only
(b) 2 only
(c) 1 and 3 only
(d) 1, 2 and 3
EXPLANATION:

Latency is an expression of how long a data packet travels from one designated point to another.
Starlink is a satellite internet constellation operated by SpaceX, providing satellite Internet access
coverage to nearly 40 countries.
Satellite internet provides higher latency compared to cable and fibre internet. Cable and fibre internet
offers latency in the range of 20 to 50 milliseconds (ms), whereas satellite internet ranges can be as

 83
.


high as 600 ms because Satellite internet data travels a long way as satellites are positioned 22,000
miles above the earth. So it offers high latency. So, Statement 1 is not correct.

Internet Service Provided by Satelite Constellations is affected by storms and Precipitation.


In heavy snowfall, communications can be restored by removing snow accumulations around the
satellite dish. So, Statement 2 is correct.

Responsible space (RS) is an initiative by OneWeb which used to describe practices that drive
sustainability within the space industry, avoiding harming our lower Earth orbit (LEO) environment
while developing this new frontier in mobility, communications and connectivity so that it works for,
and benefits, generations to come. Thus it is not an initiative by Starlink. So, Statement 3 is not
correct.

Q.58) With reference to the Nuclear Power plant, consider the following statements :
1. Light water reactor uses normal water as a moderator.
2. Graphite is used as a coolant and moderator in the Pressurise heavy water reactor.
3. Fast breeder Reactors can operate without moderators.
Which of the statements given above are correct?
(a) 1 and 2 only
(b) 2 and 3 only
(c) 1 and 3 only
(d) 1, 2 and 3
EXPLANATION:

Light water is simply ordinary or normal water that does not contain large amounts of deuterium,
making it distinct from heavy water. Although this water does contain small numbers of heavy
water molecules, it isn't enough to make any significant changes in its properties. Light water plays an
important role in the generation of electricity from nuclear energy in Light water reactors, as it can
serve both as a moderator and a coolant to carry away the energy generated by nuclear fission. So,
Statement 1 is correct.

A pressurized heavy water reactor is a type of nuclear reactor that makes use of heavy
water (H212OH212O) as its coolant and moderator. Heavy water contains
an isotope of hydrogen called deuterium and Graphite is also used as a moderator because they reflect
fast-moving neutrons, but it is not used as a coolant in the Pressurized Heavy water reactor. So,
Statement 2 is not correct.

Moderators are used to slow down the neutrons in the Nuclear reactor.
Neutrons produced by fission have high energies and move rapidly. These so-called fast neutrons do
not cause fission as efficiently as slower-moving ones. so they are slowed down in most reactors by the

 84
.


moderator. A liquid or gas moderator, commonly water or helium, cools the neutrons to optimum
energies for causing fission. These slower neutrons are also called thermal neutrons because they are
brought to the same temperature as the surrounding coolant.
In contrast to most normal nuclear reactors, however, a fast reactor uses a coolant that is not an
efficient moderator, such as liquid sodium, so its neutrons remain high-energy. Although these fast
neutrons are not as good at causing fission, they are readily captured by an isotope of uranium (U238),
which then becomes plutonium (Pu239). This plutonium isotope can be reprocessed and used as more
reactor fuel or in producing nuclear weapons.
Thus, the Fast Breeder Reactor can operate without Moderators. So, Statement 3 is correct.

Q.59) Consider the following statements :


1. Basavanna was a 12th century philosopher and a social reformer during the reign of the
Kalachuri-dynasty king Bijjala I.
2. Basavanna spread social awareness through his poetry, known as Vachanaas.
3. Basava Jayanthi is a Hindu festival celebrated by Lingayats in the North-East India.
Which of the statements given above is/are correct ?
(a) 1 and 2 only
(b) 2 only
(c) 1 and 3 only
(d) 2 and 3 only
EXPLANATION:

Basavana was an Indian 12th century statesman, philosopher, and poet, Lingayat saint in shiva
focused Bhakti movement and shaivite social reformer during the reign of Kalyani chalukya / Kalachuri
dynasty King Bijala 1. So, Statement 1 is correct.
Basavana advocated that every human being was equal, irrespective of caste and that all manual labor
was equally important. He spread social awareness through his poetry, known as Vachana. So,
Statement 2 is correct.
Basavana Jayanthi marked the birth anniversary of basavana, the founder of the Lingayat sect. The
day witnesses various celebratory events across the states like Karnataka, Maharashtra, and Andhra
Pradesh. Basavana Jayanthi is a government holiday in Karnataka, not in the Northeast states. So,
Statement 3 is not correct.

ADDITIONAL INFORMATION:

¾ In Karnataka, the Bhakti movement arose under the Virashaivas. Started by


Basavana in the 12th century, its followers became known as Lingayats.
¾ They were against caste hierarchy and the notion of pollution attached to certain
groups.
¾ It stood against Brahmanical superiority and, to 1a certain extent, patriarchy.
¾ Some of the works credited to Basavanna include Vachana, such as the Shat-
sthala-vachana, Kala-jnana-vachana, Mantra-gopya, Ghatachakra-vachana and
Raja-yoga-vachana.
Facts ¾ Basava advocated the wearing of Ishtalinga, a necklace with a pendant that
contains a small Shiva Linga.

 85
.


¾ As the Chief Minister of his kingdom, he introduced new public institutions such
as the AnubhavaMantapa (hall of spiritual experience).
The AnubhavaMantapa welcomed men and women from all socio-economic
backgrounds to discuss spiritual and mundane questions of life in the open.
¾ He is also known as Bhaktibhandari (literally, the treasurer of devotion), or
Basaveswara (Lord Basava).
¾ On 14 November 2015, the statue of Basavanna along the bank of the river
Thames at Lambeth in London was inaugurated. Furthermore, Basavanna is the
first Kannadiga in whose honor a commemorative coin has been minted in
recognition of his social reforms.

Q.60) With reference to the Ribose Nucleic Acid (RNA) consider the following statements :
1. In the cell of a human being RNA is present in mitochondria and the nucleus of the cell.
2. t-RNA carries information in the form of codes from DNA towards ribosomes for protein
synthesis.
Which of the statements given above is/are not correct ?
(a) 1 only
(b) 2 only
(c) Both 1 and 2
(d) Neither 1 nor 2
EXPLANATION:

DNA, or deoxyribonucleic acid, is the hereditary material in humans and almost all other organisms.
Nearly every cell in a person’s body has the same DNA. Most DNA is located in the cell nucleus (where
it is called nuclear DNA), but a small amount of DNA can also be found in the mitochondria (where it
is called Mitochondrial DNA or mtDNA).
Mitochondria are structures within cells that convert the energy from food into a form that cells can
use. Thus, in the cell of a human being DNA is present in the mitochondria and the nucleus of the
cell.
Whereas Ribonucleic acid (RNA) is a molecule that is present in the majority of living organisms and
viruses. It is made up of nucleotides, which are ribose sugars attached to nitrogenous bases and
phosphate groups. The nitrogenous bases include adenine, guanine, uracil, and cytosine. So,
Statement 1 is not correct.

Three main types of RNA are involved in protein synthesis. They are messenger RNA (mRNA), transfer
RNA (tRNA), and ribosomal RNA (rRNA).
In that, mRNA (messenger RNA) will transcribe information from DNA and it contains the genetic
blueprint to make proteins and carry that to the site of protein synthesis in the cytoplasm. Whereas
the tRNA carries amino acids to the ribosome during protein synthesis. So, Statement 2 is not
correct.

ADDITIONAL INFORMATION:
DIFFERENCES BETWEEN DNA AND RNA

DNA RNA

¾ Double-stranded molecule ¾ The single-stranded molecule


 86
.


¾ Contains deoxyribose sugar ¾ contains ribose sugar. Thymine is replaced by


¾ The pyrimidine base complementary to uracil
Adenine is Thymine ¾ Pyridine base complementary to adenine is uracil
¾ DNA has only one function, which is to No thymine is RNA
bear hereditary formation ¾ Many species of RNA such as mRNA, tRNA, and
¾ DNA can duplicate on its own rRNA with different functions RNA is the genetic
material in retroviruses.
¾ RNA is synthesized on a DNA template
Q.61) Recently Proton therapy is invented by researchers. It is a more efficient cancer treatment in the
world. Consider the following statements related to proton therapy :
1. Proton therapy uses x-rays to treat cancer.
2. Cyclotron is used for generating high-power energy radiation in proton therapy.
3. Proton therapy has less chance of damaging other healthy tissue surrounding malignant
tumours.
Which of the statements given above is/are correct ?
(a) 1 only
(b) 2 and 3 only
(c) 1 and 3 only
(d) 2 only
EXPLANATION:
Proton therapy is a type of radiation therapy called Proton beam radiation therapy(PBRT).
A proton is a positively charged particle. At high energy, protons can destroy cancer cells. Doctors may
use proton therapy alone. They may also combine it with x-ray radiation therapy, surgery,
chemotherapy, and/or immunotherapy.
It uses protons rather than x-rays which is used, in regular radiation therapy. X-rays continue to give
radiation doses as they leave the person's body. This means that radiation damages nearby healthy
tissues, possibly causing side effects.
In the case of proton therapy, there is less radiation dose outside of the tumor. It precisely delivers a
beam of protons to disrupt and destroy tumor cells.
So, Statement 1 is not correct.
A machine called a synchrotron or cyclotron speeds up protons. The high speed of the protons creates
high energy. This energy makes the protons travel to the desired depth in the body. The protons then
give the targeted radiation dose to the tumor.
So, Statement 2 is correct.

Proton therapy is given with painless radiation through the skin from a machine outside the body. It
may allow for a higher radiation dose to the tumor. This increases the chances that all the tumor cells
targeted by the proton therapy will be destroyed.
Around 60% less radiation can be delivered to the healthy tissues surrounding the malignant tumor,
lowering the risk of radiation damage to these tissues.
It is also used for treating children because it reduces the chance of harming healthy, growing tissue.
So, Statement 3 is correct.

ADDITIONAL INFORMATION:
 87
.


PROTON THERAPY

Benefits ¾ It may cause fewer and less severe side effects, such as low blood counts, fatigue,
and nausea during and after treatment.

¾ Proton Therapy Requires highly specialized and Costly equipment.


Drawbacks ¾ Cost more than x-ray radiation therapy.
¾ Not all cancers can be treated with proton therapy.

Cancers ¾ Central nervous system cancers


treated with ¾ Prostate cancer
proton ¾ Spinal and pelvic sarcomas
therapy
¾ Noncancerous brain tumors
¾ Eye cancer
¾ Head and neck cancers
¾ Lung cancer
¾ Liver cancer

Recently in On September 24th, 2022, the occasion of World Cancer Research Day, the Apollo
news Proton Cancer Centre (APCC) made a pilot study that indicates that proton therapy
has fewer side effects in prostate cancer treatment.

Radiation Radiation therapy is a cancer treatment that uses high-energy X-rays or other
Therapy particles to destroy cancer cells.

Maintenance ¾ The concept of maintenance or prolonged therapy has been adopted to eliminate
Therapy or suppress cancer that exists over a prolonged period and is also called
continuous therapy.
¾ To delay the growth of advanced cancer after the initial treatment.

Q.62) Which of the following is the best explanation of the Dead zone ?
(a) It is an area present in the mountains where flora and fauna don’t survive.
(b) It is a permafrost region on the continent of Antarctica.
(c) It is an area of hypoxia in the ocean.
(d) It is the region beyond the geostationary orbit where unused satellites can park.
EXPLANATION:

"Dead zone" is a more common term for hypoxia, which refers to a reduced oxygen level in the Water.
It is an area in the world's oceans and lakes with less oxygen( hypoxia) dissolved in the Water.
Because of this, most marine life either dies or, if they are mobile such as fish, leaves this area, and
as only few organisms can survive in hypoxic conditions.
Hypoxic zones can occur naturally, but scientists said that hypoxic areas are created or enhanced
by human activity, For e.g. Dead zones occur because of a process called eutrophication. Many
physical, chemical and biological factors combine to create dead zones, but nutrient pollution is the
primary cause of those zones created by humans. So, Option (c) is correct.

 88
.


ADDITIONAL INFORMATION:

DEAD ZONE

Hypoxia ¾ Hypoxic areas have increased dramatically during the past 50 years.
¾ Hypoxia occurs when algae and other organisms die from a lack of oxygen and
available nutrients. Hypoxia events often follow algal blooms.
¾ Bacteria decompose the cyanobacteria, algae, and phytoplankton sink to the seafloor
and this lack of oxygen creates dead zones where most aquatic species cannot
survive.

Marine ¾ The Gulf of Mexico has a seasonal hypoxic zone that forms every year in late summer.
Dead ¾ Scientists have identified 415 dead zones worldwide.
 89
.


Zones ¾ The Baltic Sea is also home to seven of the world's 10 largest marine dead zones.
Recently Increased runoff from agricultural fertilizers and sewage has sped up the
in the eutrophication process. Overfishing of Baltic cod has intensified.
news -

Q.63) If earth swaps its position with Mars, then what will be the possible consequences ?
1. May lead to the extinction of living creatures
2. Decrease in the length of the day on the Earth
3. Orbital period of the planet around the sun increases
Select the correct answer using the code given below :
(a) 1 and 2 only
(b) 2 and 3 only
(c) 1 and 3 only
(d) 1, 2 and 3
EXPLANATION:

Mars would be benefitted, as a fairly modest increase in Mars's temperature would melt the polar caps
and liberate gases from the soil, flipping the Martian climate into a new, cozier state nearly as warm
as Earth.

 90
.


But Earth would get the short end of the deal as now, the Sunlight would be half as intense and the
planet would freeze over.
Because of this arrangement some living creatures on Earth, which need plenty of sunlight to survive,
may fail to withstand such freezing temperatures.
So, Statement 1 is correct.

Assuming that when Earth occupies the Martian position its rotation time will be equal to that of
Martian rotation time (24 hrs- now), this makes the length of the day to be lengthier (increases and not
decreases) than that before, in the Earth.
So, Statement 2 is not correct.

The orbital period (time taken to complete one revolution around the Sun) of the martian position is
around 687 days and hence the orbital period increases, when Earth goes to the Martian position. So,
Statement 3 is correct.

Q.64) Consider the following statements with respect to Prathan Mantri Svamitva Scheme :
1. It is a central Sector Scheme of the Ministry of Rural development.

 91
.


2. It aims to provide the ‘Record of Rights’ to village household owners possessing houses in
inhabited areas in villages.
3. The scheme is aimed at reducing property related disputes and also enables bank loans.
Which of the statements given above is/are correct ?
(a) 1 and 3 only
(b) 2 only
(c) 3 only
(d) 2 and 3 only
EXPLANATION:

Pradhan Mantri Svamitva Scheme is a central sector scheme of the Ministry of Panchayati Raj. So,
Statement 1 is not correct.

A record of Rights is a legal document that details the land and who owns it. You will get all the
essential information about any land with the help of this document. The main aim of this scheme is
to provide a Record of Rights to village household owners who possess houses in inhabited areas with
legal ownership rights. So, Statement 2 is correct.

To bring financial stability to the citizens in rural India by enabling them to use their property as a
financial asset for taking loans and other financial benefits and by making the proper ownership of the
land, it will reduce property-related disputes. So, Statement 3 is correct.

ADDITIONAL INFORMATION:

SVAMITVA scheme

About ¾ SVAMITVA (Survey of villages and mapping with improvised technology in village
areas) scheme is a new initiative of the Ministry of Panchayati Raj. It aims to
provide rural people with the right to document their residential properties so
that they can use their property for economic purposes.
¾ The scheme is for surveying the land parcels in the rural inhabited area using
Drone technology.

Objectives ¾ To bring financial stability to the citizens in rural India by enabling them to use
their property as a financial asset for taking loans and other financial benefits.
¾ Creation of accurate land records for rural planning.
¾ Determination of property tax, which would accrue to the GPs directly in States
where it is devolved or else add to the State exchequer.
¾ Creation of survey infrastructure and GIS maps that can be leveraged by any
department for their use.
¾ To support in the preparation of a better-quality Gram Panchayat Development
Plan (GPDP) by making use of GIS maps.
¾ To reduce property-related disputes and legal cases.

Q.65) Consider the following statements :


1. Pure water is a good conductor of electricity.
2. Water is amphoteric in nature.
3. Water contracts on heating between 0 °C and 4 °C.

 92
.


Which of the statements given above is/are correct ?


(a) 1 only
(b) 2 only
(c) 2 and 3 only
(d) 1 and 3 only
EXPLANATION:

Distilled Water is Pure Water; distilled Water is free of salts and a poor conductor of electricity. When
salt such as common table salt (sodium chloride (NaCl)is dissolved in distilled Water, a salt solution is
obtained, which is the conductor of electricity.
Pure Water is an excellent insulator and does not conduct electricity. Water in nature is most nearly
pure in its vapour state, and Water is an excellent solvent and stops being an excellent insulator once
it starts dissolving substances around it.
Even a small number of ions in a water solution enables it to conduct electricity. When water contains
these ions, it will conduct electricity. So, Statement 1 is not correct.
A substance or compound with both acid and a base is known as amphoteric.
According to the Bronsted-Lowry theory of acids and bases, acids are proton donors, and bases are
proton acceptors.
Water's amphoteric in nature chemical reaction as follows:
As an acid:
The water molecule has hydrogen atoms and, therefore, could act as an acid in a reaction

As a base:

Water has two molecules that make a molecular autoionization (Self-ionization) reaction, in which one
water molecule acts as an acid and another as a base.
H2O + H2O ֖ H3O+ + O.H.−
Water is thus the most common example, which behaves as an acid and a base and therefore
is amphoteric in nature.
So, Statement 2 is correct.

In general, the volume of liquid increases with an increase in temperature.


However, the volume of Water does not increase with a temperature between 0 to 4°C.
The volume of a given amount of Water decreases as it is cooled from room temperature until its
temperature reaches 4 °C. Below 4 °C, the volume increases, and therefore the density decreases,
which means that Water has a maximum density at 4 °C and hence the water contracts.
The density of Water reaches a maximum value of 1 g mL–1 or 1000 kg m–3 at 4°C, and its volume
increases when the density decreases. This phenomenon is the anomalous behaviour of Water. So,
Statement 3 is correct.

 93
.


Q.66) Which of the following rays are used in computed tomography ?


1. X-rays
2. Gamma rays
3. Beta rays
4. Magnetic resonance
Select the correct answer using the code given below :
(a) 1 only
(b) 2 and 3 only
(c) 1, 3 and 4 only
(d) 1, 2, 3 and 4
EXPLANATION:

Computed tomography (C.T.) imaging, also known as "CAT scanning" (Computerized Axial
Tomography), provides a different form of imaging known as cross-sectional imaging or "slices" of
anatomy, like the slices in a loaf of bread
Computed tomography, or C.T., scans are medical imaging tests that use ionizing radiation to create
cross-sectional (slices) pictures inside selected areas of the body from different angles
And it combines a series of x-ray (radiography) images into a three-dimensional picture. So, Option
(1) is correct.

 94
.


Beta particles (β) are high energy, high speed electrons (β-) or positrons (β+) that are ejected from the
nucleus by some radionuclides during a form of radioactive decay called beta-decay.
Beta-particles, being less ionising than alpha-particles, can travel through many centimetres or even
metres or air and through millimetres of skin or tissue. Sufficient intensity of beta-radiation can cause
burns, rather like severe sunburn. If beta-emitting radionuclides are inhaled or ingested, they can also
do damage to internal cells and organs.
So, Option (2) is not correct.

A gamma ray (g) is a packet of electromagnetic energy (photon) emitted by the nucleus of
some radionuclides following radioactive decay. Gamma photons are the most energetic photons in
the electromagnetic spectrum
The key difference between gamma rays and X-rays is how they are produced. Gamma rays originate
from the settling process of an excited nucleus of a radionuclide after it undergoes radioactive decay
whereas X-rays are produced when electrons strike a target or when electrons rearrange within an
atom.
Similar to all exposure to ionising radiation, high exposures can cause direct acute effects through
immediate damage to cells. Low levels of exposure carry a stochastic health risk where the probability
of cancer induction rises with increased exposure.
So, Option (3) is not correct.

MRI scanners are particularly to image the body's non-bony parts or soft tissues.
MRIs employ powerful magnets which produce a strong magnetic field that forces protons in the body
to align with that field. They differ from computed tomography (C.T.) because they do not use the
ionizing radiation of x-rays; instead, it uses a strong magnetic field.
MRI uses strong magnetic fields and non-ionizing radiations to accurately detect pathological and
physiological changes in living tissue.
So, Option (4) is not correct.

 95
.


Q.67) Consider the following statements:


1. Liquid petroleum gas is odourless and colourless gas consisting of Propane and butane.
2. Liquid petroleum gas is lighter than compressed natural gas.
3. CNG is heavier than air.
How many statement(s) given above is/are correct?
(a) Only one statement
(b) Only two statements
(c) Only three statements
(d) None of the statements
EXPLANATION:

LPG fuel, or liquefied petroleum gas, is a liquefied gas and is a by product derived while extracting
crude petroleum. LPG weighs twice as much as air and is colorless, odourless (While odorant is added
to it only to ensure safety) and is a highly flammable explosive gas. It is comprised of propane mixed
with butane, traces of propylene and butylene. So, Statement 1 is correct.
CNG is Compressed Natural Gas, which is mainly methane compressed at a pressure of 200 to 248
bars. Compressed natural gas (CNG)is produced by compressing natural gas under high pressure, and
it is a cleaner fuel in transport vehicles (buses, trucks and cars) and contains Methane. CNG is a good
alternative to petrol and diesel because it causes less pollution.
In the order of density, LPG> Air > CNG. Therefore, Liquid petroleum gas is heavier than compressed
natural gas. So, Statement 2 is not correct.

Compressed Natural gas is lighter than Air because natural gas density is lower than Air's density and
rapidly dissipates into the Air when released. So, Statement 3 is not correct.

Q.68) With reference to the Fortification of Food, consider the following statements :
1. Fortification guidelines are given by the Food Safety and Standard Authority of India.
2. According to the rules given by the FSSAI, milk packed in the pouch should be mandatorily
fortified with Vitamin A and D.
3. Golden Rice produced through genetic fortification requires greater changes in management
practices during cultivation.
Which of the statements given above is/are correct ?
(a) 1 and 2 only
(b) 2 and 3 only
(c) 1 and 3 only
(d) 1, 2 and 3
EXPLANATION:

Fortification means deliberately increasing the content of essential micronutrients in food to improve
the nutritional quality of food and provide health benefits with minimal risk to the public's health.
The Food Safety and Standards Authority of India (FSSAI) statutory body under the Food Safety and
Standards Act, 2006, frames regulations, lays orders and guidelines under Food Safety and Standards
(Food Products Standards and Food Additives) Regulations 2011 for Packaging and Labelling of Food
Businesses, Food Product Standards and Food Additives Regulation and it is not given by Food safety
and standard organization. So, Statement 1 is correct.

 96
.


FSSAI – Food Safety and Standard Authority of India in December 2020, issued a draft notification,
for mandatory fortification of packaged toned, double toned, skimmed milk or standardised milk with
Vitamin A and Vitamin D. This will ensure nationwide availability of fortified milk and a positive impact
in mitigating micronutrient malnutrition among population which is crucial to make concerted efforts
for scaling up and sustaining efforts towards quality fortification while making fortification mandatory
in the country.
As per the FSSAI standards, the milk needs to be fortified with Vitamin A and D at a level of 270 μg RE
- 450 μg RE per litre and 5 μg -7.5 μg per litre respectively.
Fortifying milk with micronutrients is a good strategy for addressing micronutrient malnutrition, since
it is consumed by all groups of people. So, Statement 2 is correct.

Biofortification or biological fortification refers to nutritionally enhanced food crops


that are developed and grown using modern biotechnology techniques, conventional plant breeding,
and agronomic practices.
Golden Rice is biofortified with provitamin A, and rice is the staple crop and can effectively control
vitamin A deficiency (VAD).
Golden Rice is rice genetically engineered to produce and accumulateβ-carotene (beta carotene
provitamin A, a plant pigment that the body converts into vitamin A as needed) in the endosperm (the
edible part of the grain) and hence this compound gives yellow-orange or golden colour.
Like ordinary rice, Golden Rice does not require any special cultivation practices, and generally has
the same yield and agronomic performance. So, Statement 3 is not correct.

ADDITIONAL INFORMATION:

Codex ¾ Codex Alimentarius is an international food standard that issues guidelines and a
Alimentarius code of practice that contributes to the safety, quality and fairness of this
international food trade.
¾ The efforts are being internationally coordinated by Food and Agriculture
organization (FAO) and the World Health Organization (WHO).

Carotenoids ¾ Carotenoids, including β-carotene, are naturally occurring plant pigments widely
found in coloured fruits, carrots, and green vegetables.
¾ Plants do not contain Vitamin A, but only its precursor, β-carotene, also known as
provitamin A. Animals, including man, synthesize Vitamin A from a few carotenoids
eaten in the diet. Hence, animal meat products contain Vitamin A.
¾ Only some carotenoids have provitamin A activity, but β-carotene is the most
common and important among them. Rice is the most important staple food for
millions of people in developing countries. Hence, the delivery of β-carotene with
Golden Rice's help could reduce chronic health problems caused by vitamin A
deficiency (VAD).

 97
.


Fortification

FSSAI Norms

1 kg of
Vitamin B-
fortified rice Folic acid
12 (0.75-
will contain (75-125
1.25
iron (28 mg- micrograms)
microgram)
42.5 mg)

¾ Fortification is the addition of key vitamins and minerals such as iron, iodine, zinc,
and Vitamin A and D to staple foods such as rice, milk and salt to improve their
nutritional content.

Golden Rice ¾ The beta carotene in Golden Rice, which was made possible by the addition of two
new enzymes, is identical to the beta-carotene found in green leafy and yellow-
colored vegetables, orange-colored fruit, and even in many vitamin supplements
and food ingredients.

Q.69) With reference to lithium element consider the following statements:


1. Lithium is the lightest metal on earth.
2. Excess intake of lithium can cause black foot disease.
3. Region of the lithium triangle consists of Chile, Argentina, and Bolivia.
Which of the statement given above is/are correct ?
(a) 1 only
(b) 2 and 3 only
(c) 1 and 3 only
(d) 1, 2 and 3
EXPLANATION:
OPTION ELIMINATION STRATEGY
Density is directly proportional to mass. Lithium is one of the first seven metals in the periodic table
and is an alkali metal group; these group metals are considered light-density metals. Hence,
statement 1 could be correct.

Density is the amount of matter per unit of volume. The less matter there is in a space, the lighter the
substance is. So, the lightest metallic element is the one with the lowest density. The lightest or least
dense element that is a metal is lithium (also known as white gold). Lithium has atomic number 3 on
the periodic table, with a density of 0.534 g/cm3.The density of water is about 1 g/cm3, so lithium
floats on water. So, Statement 1 is correct.

 98
.


Lithium toxicity, also called lithium overdose, can lead to coma, brain damage, or even death.
Moreover, Lithium can induce serotonin syndrome, a potentially fatal and life-threatening condition.
Whereas Blackfoot disease is a vascular disease associated with long-term exposure to
Arsenic(inorganic). So, Statement 2 is not correct

The remote regions of the three countries that make up the Lithium Triangle are Argentina, Bolivia and
Chile, which account for more than 63 percent of the world's lithium reserves. Salar de Uyuni in Bolivia,
has the richest lithium deposits in the world. So, Statement 3 is correct.

ADDITIONAL INFORMATION:

LITHIUM

About ¾ Lithium is a soft, silver-white metal within the alkali metal group on the periodic table.
¾ Group 1 of the Periodic Table consists of the elements: Lithium, sodium, potassium,
rubidium, caesium and francium and these are collectively known as the alkali metals.
Uses ¾ Lithium metal is used to make useful alloys such as' white metal' bearings for motor
engines, with aluminum to make aircraft parts.
¾ It is used in thermonuclear reactions.

 99
.


¾ Lithium is also used to make electrochemical cells.


¾ It is in rechargeable batteries for mobile phones, laptops, digital cameras and electric
vehicles.

Facts
According to the w Lithium resources
US Geological worldwide total about
Survey 89 million tonnes (mt)

• Chile
Top countries • Australia
with resources • Argentina
• China

Lithium ¾ In 2021, India's first lithium deposit was discovered (Marlagalla – Allapatna area in
reserves Mandya district of Karnataka with 1,600 tonnes.
in India

Major
lithium
producers
in the
world
(2022)

Q.70) Consider the following statements with reference to Wetlands :


1. Pantanal is the world’s largest wetland located in South Africa.
2. In India, the Wetlands (Conservation and Management) Rules, excludes river channels, paddy
fields as wetlands.
3. Wetlands in India are announced by the Ministry of Jal Sakthi.
Which of the statements given above is/are not correct ?
(a) 1 and 2 only
 100
.


(b) 2 only
(c) 1 and 3 only
(d) 2 and 3 only
EXPLANATION:

Pantanal is the world’s largest tropical wetland and one of the most pristine in the It is spread across
three South American countries—Bolivia, Brazil and Paraguay. So, Statement 1 is not correct.

Wetland is an area of marsh, fen, peatland or water; whether natural or artificial, permanent or
temporary, with water that is static or flowing, fresh, brackish or salt, including areas of marine water
the depth of which at low tide does not exceed six meters, but does not include river channels, paddy
fields, human-made water bodies/tanks specifically constructed for drinking water purposes and
structures specifically constructed for aquaculture, salt production, recreation and irrigation purposes.
So, Statement 2 is correct.

The union environment ministry will declare wetlands in India. So, Statement 3 is not correct.
ADDITIONAL INFORMATION:

India adds 10 more wetlands designated as Ramsar sites to make total 64 sites covering an area
of 12,50,361 ha in the country. The 10 new sites include: Six (6) sites in Tamil Nadu and One (1) each
in Goa, Karnataka, Madhya Pradesh and Odisha.

Koonthankulam Bird Sanctuary Tamil Nadu


Satkosia Gorge Odisha

Nanda Lake Goa

Gulf of Mannar Marine Biosphere Reserve Tamil Nadu


Ranganathituu BS Karnataka

Vembannur Wetland Complex Tamil Nadu

Vellode Bird Sanctuary Tamil Nadu

Sirpur wetland Madhya Pradesh

Vedanthangal Bird Sanctuary Tamil Nadu

Udhayamarthandapuram Bird Sanctuary Tamil Nadu


Q.71) Consider the following statements with reference to the Sunspots :
1. Increase in the number of Sunspot cause an increase in the area geo-tail than the normal
sunspot activity.
2. If the number of Sunspots increases on the sun, then decrease in the temperature of the corona
of the sun.
3. Sunspots are the regions with higher magnetic pressure.
Which of the statements given above is/are correct ?
(a) 2 only
(b) 1 and 3 only
(c) 1 and 2 only
(d) 1, 2 and 3

 101
.


EXPLANATION:

Geo tails are formed due to the interactions between the Sun's magnetic fields and Earth's magnetic
fields.
Sunspots are areas where the magnetic field is about 2,500 times stronger than Earth's, much higher
than anywhere else on the Sun, increase in presence of which will lead to more compression of the earth
side facing envelop (refer diagram), and the area of the geo tail will thus be increased than during the
normal sunspot activity. So, Statement 1 is correct.

Sunspots are areas of a high magnetic field. Because of the strong magnetic field, the magnetic pressure
increases while the surrounding atmospheric pressure decreases.
This lowers the temperature relative to its surroundings because the concentrated magnetic field
prevents the flow of hot, new gas from the Sun's interior to the surface. Thus the temperature of the
corona of the Sun will be decreased. So, Statements 2 and 3 are correct.

ADDITIONAL INFORMATION:

GEO TAIL

About ¾ The Sun emits the solar wind, a continuous stream of charged particles.
¾ These particles are embedded in the extended magnetic field of the Sun. Since
the Earth has a magnetic field, it obstructs the solar wind plasma.
¾ This interaction results in the formation of a magnetic envelope around Earth.
¾ On the Earth's side facing the Sun, the envelope is compressed into a region
approximately three to four times the Earth's radius.
¾ On the opposite side, the envelope is stretched into a long tail, extending beyond
the Moon's orbit.

Terminologies ¾ Sunspots are areas that appear dark on the surface of the Sun. They appear dark
because they are cooler than other parts of the Sun’s surface. The temperature
of a sunspot is still very hot though around 6,500 degrees Fahrenheit!
¾ Solar flares are a sudden explosion of energy caused by tangling, crossing or
reorganizing of magnetic field lines near sunspots.
 102
.


¾ The surface of the Sun is a very busy place. It has electrically charged gases that
generate areas of powerful magnetic forces. These areas are called magnetic
fields.
¾ The Sun’s gases are constantly moving, which tangles, stretches and twists the
magnetic fields. This motion creates a lot of activity on the Sun's surface,
called solar activity.
Q.72) Which of the following are radioactive elements?
1. Hassium
2. Promethium
3. Technetium
4. Zirconium
5. Nobelium
Select the correct answer using the code given below :
(a) 1, 2 and 3 only
(b) 4 and 5 only
(c) 1, 2, 3 and 5 only
(d) 1, 2, 3, 4 and 5
EXPLANATION:
Radioactivity is the natural phenomenon of the spontaneous disintegration of unstable atomic nuclei
to form more energetically stable atomic nuclei with the emission of both energy and particles called
Radiation.
¾ Hassium is a highly radioactive metal belonging to the transuranium group with atomic number
108. The German physicist's Peter Armbruster and Gottfried Munzenberg discovered this element
in the German state called Hesse. So, It got the name Hassium. At present, it is only used for
research purposes.
¾ Promethium is a radioactive material with atomic number 61 named after the Greek Prometheus
and was discovered by Jacob. A. Marinsky, Lawrence E. Glendenin, and Charles D. Coryell. It is
mostly used for research purposes, and further, it is used in specialized atomic batteries. These
are roughly the size of a drawing pin and are used for pacemakers, guided missiles, and radios. It
can also be used as a source of x-rays and radioactivity in measuring instruments.
¾ Technetium was the silver-grey radioactive element with atomic number 43 and the first artificially
produced element. It was discovered by Carlo Perrier and Emilio Segre in 1937. It was created by
bombarding molybdenum atoms with deuterons and is used for medical diagnostic studies and
as a corrosion inhibitor for steel.
¾ Zirconium is a hard, silvery metal that is very resistant to corrosion. It got its name from the
Arabic word, 'zargun,' meaning gold. Martin Heinrich Klaproth discovers it in 1789. But it is not
a radioactive element, but it is used as a semi-precious gemstone and in making ceramics.
¾ Nobelium is a radioactive metal with a half-life of only 58 minutes, named after Alfred Nobel, the
founder of the Nobel prize. Still, it is not used in any outside research or biological activities.
So, Option (c) is correct.

Q.73) Consider the following statements:


1. Silicon and Germanium are commonly used in microprocessor manufacturing.
2. Due to the absence of free electron zero resistivity can achieve in the superconductor.
3. Superconductivity can be achieved at room temperature.
 103
.


Which of the statements given above is/are correct ?


(a) 1 only
(b) 2 and 3 only
(c) 1 and 3 only
(d) 1, 2 and 3
EXPLANATION:

A semiconductor is the foundation of modern electronics driving the next phase of digital
transformation under Industry 4.0. These are materials that have conductivity between conductors
and non conductors or insulators. It can be pure elements, such as silicon or germanium, gallium
arsenide, or cadmium selenide compounds. Semiconductors such as silicon, Cadmium, and
Germanium are used for manufacturing microprocessors used in Cell phones, Computers, household
appliances, Automobiles, etc. So, Statement 1 is correct.

A superconductor is a material that achieves superconductivity, a state of matter with no electrical


resistance, and does not allow magnetic fields to penetrate.
Electricity is the movement of free electrons in a conducting material like copper. While In conductors,
the free electrons move randomly and frequently collide with one another and other particles in the
material, thus offering resistance to the current flow. Here, a lot of energy is lost in the form of heat
Energy.
In a superconducting state, all the electrons align themselves in a particular direction and move
without obstruction in a "coherent" manner. Because of zero resistance, superconducting materials
can save huge amounts of energy and be used to make highly efficient electrical appliances. Thus,
unlike Conductors, free electrons are absent in Superconductors and hence it has zero resistivity. So,
Statement 2 is correct.

Critical Temperature is a temperature at which the material becomes superconductive, where the
material shows Zero resistance. Scientists have been able to make materials superconduct only at
temperatures much below zero degrees C, which makes it practically very difficult.
But recently, a team led by a Professor from the Indian Institute of Science (IISc), Bengaluru,
confirmed that the material in the form of nanosized films and pellets made of silver nanoparticles
embedded in a gold matrix exhibited superconductivity at room temperature and Pressure. But, silver
and gold independently do not exhibit superconductivity. So, Statement 3 is correct.

Q.74) With reference to Goa Civil Code, consider the following statements :
1. It is an extension of the 1867 French Civil Code to France’s overseas provinces (that included
Goa).
2. The law doesn’t recognise bigamy or polygamy for Muslims.
3. It grants an exception to a Hindu man to marry once again if his wife doesn’t conceive a child by
the age of 21 or a male child by the age of 30.
Which of the above statements are correct ?
(a) 1 and 2 only
(b) 2 and 3 only
(c) 1 and 3 only
(d) 1, 2 and 3

 104
.


EXPLANATION:
The Goa Civil Code also called the Goa Family Law, is the set of civil laws that governs the native
residents of the Indian State of Goa and Damon. The Goa civil code was introduced after Portuguese
Goa and Damon were elevated from being mere Portuguese colonies to the status of a Provincial
Ultramarine (Overseas possession) in 1869 AD. So, Statement 1 is not correct.

Bigamy is the act of marrying someone while already having been legally married to some one else who
is alive. The Goa Civil Code doesn't recognise bigamy or polygamy, including for Muslims but grants
an exception to a Hindu man to marry once again if his wife doesn't conceive a child by the age of 21
or a male child by the age of 30. So, Statements 2 and 3 are correct.

ADDITIONAL INFORMATION:

UNIFORM CIVIL CODE


Civil Laws ¾ Civil Law is the Law of the State or the Law of the land. The area of laws and
justice affects an individual's legal status.
¾ This branch of Law deals with rights, duties and obligations of individual
members of the society among themselves
¾ Civil Law includes various aspects such as laws relating to a property, contract,
tort, family, trade, intellectual property and environment etc.

Criminal Laws ¾ Criminal Law is a body of rules that defines the conduct prohibited by the State
for being harmful to public safety and welfare and also prescribes punishment
to be imposed for the commission of such acts.
¾ The Criminal Law of India is codified in the Indian Penal Code, 1860, Criminal
Procedure Code, 1973 and Indian Evidence Act, 1872. These are known as Major
Criminal Acts. Besides these Criminal Acts, there are some other minor Criminal
Acts, such as Narcotics and Psychotropic Substances Act, Arms Act, Drugs and
Cosmetics Act, Dowry Prohibition Act, etc.
personal laws ¾ Personal Law is the Law by which an individual is governed in respect of various
matters, such as principles relating to marriage, divorce, maintenance, adoption,
inheritance, guardianship, succession, etc.

Uniform Civil ¾ The Uniform Civil Code, which comes under Article 44 of the Constitution of
Code India, proposes to introduce personal laws that would apply to all citizens
equally, irrespective of their religion, gender, caste, etc.
¾ Uniform Civil Code essentially refers to a common set of laws governing personal
matters such as marriage, divorce, adoption, inheritance, and succession.
¾ Article 44 states, The State shall endeavor to secure for the citizens a uniform
civil code throughout the territory of India. However, since the article comes
under the directive principles of state policy, they are regarded as only guidelines,
and it is not mandatory to use them.
Q.75) Consider the following statement related to Light Interferometer Gravitational Observatory (LIGO):
(a) LIGO help to prove the general theory of relativity with help of laser.
(b) Like large hadron collider (LHC), LIGO operates in vacuum chambers.
(c) Collision of two black holes can produce gravitational waves.
(d) A Single object cannot produce Gravitational waves.

 105
.


Which of the statements given above is/are correct ?


(a) 3 only
(b) 1 and 2 only
(c) 1, 2 and 3 only
(d) 1, 2, 3 and 4
EXPLANATION:

In his general theory of relativity, Albert Einstein was the first to predict electromagnetic waves 100
years ago.
The general theory of relativity is defined as when massive accelerating objects such as neutron stars
or black holes orbit each other. They would disrupt space-time so that 'waves' of distorted space would
radiate from the source. These ripples travel at the speed of light through the universe, carrying
information about their origins and invaluable clues to the nature of gravity itself.
Thus, the LIGO detector, which uses Laser to detect Gravitational waves, supported the general theory
of relativity. So, Statement 1 is correct.
The Large Hadron Collider (LHC) is the world’s largest and most powerful particle accelerator. It
consists of a 27-kilometre ring of superconducting magnets with a number of accelerating structures
to boost the energy of the particles along the way.
LIGO's vacuum volume is the third largest in the world, where after the Large Hadron Collider (LHC)
in Switzerland, and NASA's "Space Simulation Vacuum Chamber". So, Statement 2 is correct.

When two black holes collide, they will merge to become one bigger black hole, which is very violent,
and these mergers would produce tremendous energy and send massive ripples through the Space–
time fabric of the Universe. These ripples are called as Gravitational waves and they travel at the speed
of light (186000 miles per second). So, Statement 3 is correct.

Every object with mass that accelerates produce Gravitational waves. These can be Human, Cars,
airplane etc. Thus, Single object can also produce gravitational waves. So, Statement 4 is not correct.

ADDITIONAL INFORMATION:
LIGO LIGO stands for Laser Interferometer Gravitational-Wave Observatory. It is the world's
largest gravitational wave observatory, which consists of a pair of huge interferometers,
each having two arms that are 4 km long. These two LIGO detectors were used to detect
Gravitational signals, where both works as one unit to ensure remarkable precision.
There are only two LIGO Detectors there to study Gravitational waves; they are at
1. Hanford in Washington
2. Livingston in Louisiana.
Recently Recently, 225 hectares of land have been handed over to the project authorities for setting
in News up the LIGO Observatory in the Hingoli Taluk of the Parbhani District of Maharashtra.
If Hingoli LIGO Detector is Completed, it will be the third Gravitational Wave detector
worldwide.

Q.76) With reference to the Producer gas consider the following statements :
1. Producer gas is a colorless gas that mainly consists of Methane and hydrogen.
2. Producer gas is generated by burning coal or coke without the presence of oxygen.
 106
.


3. Mostly used as fuel in the iron and steel manufacturing industries.


Which of the statements given above is/are correct ?
(a) 1 and 2 only
(b) 2 and 3 only
(c) 3 only
(d) 1, 2 and 3
EXPLANATION:

Producer gas is a fuel gas obtained when coal or coke is burnt with air deficiency and a controlled
amount of moisture. It is a mixture of flammable gases like Carbon monoxide and hydrogen and
nonflammable gases like nitrogen and carbon dioxide.
Its average composition includes,
¾ Carbon Monoxide = 22.3%
¾ Hydrogen = 8.12%
¾ Nitrogen = 52.55%
¾ Carbon dioxide = 3%
Thus, it has only the above-mentioned compounds and doesn't contain Methane. So, Statement 1 is
not correct.

It is produced when coke or other carbonaceous material such as anthracite is burnt with controlled
moisture and oxygen supply, but not without a complete absence of oxygen. A higher calorific value
could be obtained by using oxygen-enriched air with steam. So, Statement 2 is not correct.

Producer gas was used primarily as industrial fuel for iron and steel manufacturing, such as firing
coke ovens and blast furnaces, cement, and ceramic kilns, or mechanical power through gas Engines.
It is used as a reducing agent in metallurgical operations and also used to power gas turbines, Spark
ignited engines, or Diesel internal combustion engines. So, Statement 3 is correct.

ADDITIONAL INFORMATION:

GASES

Producer When air is passed over the red-hot fuel, carbon monoxide is produced in an exothermic
Gas reaction. The nitrogen in the air remains unchanged and dilutes the gas, so it has a low
calorific value. The gas may be used to power gas turbines which are suited to fuels of
low calorific value.

Water gas The mixture of CO and H2 is called water gas. This mixture of CO and H2
and is used for synthesizing methanol and several hydrocarbons; it is also called synthesis
Syn Gas gas or 'syngas.' Nowadays, 'syngas' is produced from sewage, sawdust, scrap wood,
newspapers, etc. The process of producing 'syngas' from coal is called coal gasification.

Town gas Town gas contains hydrogen (H2), carbon monoxide (CO), carbon dioxide (CO2),
methane (CH4), nitrogen (N2), and volatile hydrocarbons.
It is made by blowing air and steam over an incandescent fuel bed, usually of coke or
coal. It is also called "coal gas."Gas was used for lighting before the advent of electric
lighting (Electric Bulb) and for heating and cooking before natural gas became widely
available.

 107
.


Q.77) Consider the following statements regarding Polymerase Chain Reaction (PCR) technique :
1. PCR helps in making many copies of a small amount of DNA.
2. DNA molecule is first cooled to break it up into two strands followed by heating with primers so
that they bind to the DNA.
3. RT-PCR diagnosis used for COVID-19 employs (NAAT) method.
Which of the statements given above is/are not correct ?
(a) 1 and 2 only
(b) 2 only
(c) 2 and 3 only
(d) 1, 2 and 3
EXPLANATION:
Polymerase chain reaction (PCR), is also known as "molecular photocopying," is a fast and inexpensive
technique used to "amplify" - copy - small segments of DNA. Because significant amounts of a sample
of DNA are necessary for molecular and genetic analyses, studies of isolated pieces of DNA are nearly
impossible without PCR amplification.
Once amplified, the DNA produced by PCR can be used in many different laboratory procedures. PCR
is also valuable in a number of laboratory and clinical techniques, including DNA fingerprinting,
detection of bacteria or viruses (particularly AIDS), and diagnosis of genetic disorders.
So, Statement 1 is correct.
The steps in PCR are:
1. To amplify a segment of DNA using PCR, the sample is first heated so the DNA denatures, or
separates into two pieces of single-stranded DNA.
2. Primers are added and the DNA is cooled so that they bind to the DNA.
3. Next, an enzyme called "Taq polymerase" synthesizes - builds - two new strands of DNA, using the
original strands as templates.
This process results in the duplication of the original DNA, with each of the new molecules containing
one old and one new strand of DNA.
Then each of these strands can be used to create two new copies, and so on, and so on. The cycle of
denaturing and synthesizing new DNA is repeated as many as 30 or 40 times, leading to more than
one billion exact copies of the original DNA segment.
The entire cycling process of PCR is automated and can be completed in just a few hours. It is directed
by a machine called a thermocycler, which is programmed to alter the temperature of the reaction
every few minutes to allow DNA denaturing and synthesis.
Heating is followed by cooling, So, Statement 2 is not correct.

 108
.


A Nucleic Acid Amplification Test, or NAAT, is a type of viral diagnostic test for SARS-CoV-2, which
causes COVID-19. NAATs for SARS-CoV-2 specifically identify the RNA (ribonucleic acid) sequences
that comprise the virus's genetic material.
The NAAT procedure works by first amplifying or making many copies of the virus’s genetic material,
if any is present in a person’s specimen. Amplifying those nucleic acids enables NAATs to detect very
small amounts of SARS-CoV-2 RNA in a specimen, making these tests highly sensitive for diagnosing
COVID-19.
NAATs can use many different methods to amplify nucleic acids and detect the virus, including but
not limited to:
x Reverse transcription polymerase chain reaction (RT-PCR)
x Isothermal amplification including:
o Nicking endonuclease amplification reaction (NEAR)
o Transcription mediated amplification (TMA)
o Loop-mediated isothermal amplification (LAMP)
o Helicase-dependent amplification (HDA)
o Clustered regularly interspaced short palindromic repeats (CRISPR)
o Strand displacement amplification (SDA)
So, Statement 3 is correct.

 109
.


ADDITIONAL INFORMATION :
RT PCR ¾ RT–PCR is a variation of PCR, or polymerase chain reaction. The two techniques
use the same process except that RT–PCR has an added step of reverse
transcription of RNA to DNA, or RT, to allow for amplification.
¾ This means PCR is used for pathogens, such as viruses and bacteria, that already
contain DNA for amplification, while RT–PCR is used for those containing RNA
that needs to be transcribed to DNA for amplification.
¾ Both techniques can be performed in ‘real time’, which means results are visible
almost immediately, while when used ‘conventionally’, results are only visible at
the end of the reaction.
¾ PCR is one of the most widely used diagnostic tests for detecting pathogens,
including viruses, that cause diseases such as Ebola, African swine fever and
foot-and-mouth disease. Since the COVID-19 virus only contains RNA, real time
or conventional RT–PCR is used to detect it.

How RT PCR ¾ A sample is collected from the parts of the body where the COVID-19 virus
works in real gathers, such as a person’s nose or throat.
time ¾ The sample is treated with several chemical solutions that remove substances
such as proteins and fats and that extract only the RNA present in the sample.
¾ This extracted RNA is a mix of the person’s own genetic material and, if present,
the virus’s RNA.
¾ The RNA is reverse transcribed to DNA using a specific enzyme. Scientists then
add additional short fragments of DNA that are complementary to specific parts
of the transcribed viral DNA.
¾ If the virus is present in a sample, these fragments attach themselves to target
sections of the viral DNA.
¾ Some of the added genetic fragments are used for building DNA strands during
amplification, while the others are used for building the DNA and adding marker
labels to the strands, which are then used to detect the virus.
¾ The mixture is then placed in an RT–PCR machine. The machine cycles through
temperatures that heat and cools the mixture to trigger specific chemical
reactions that create new, identical copies of the target sections of viral DNA.

Q.78) Catalytic converter is used for:


(a) Conversion of biomass into ethanol
(b) Conversion of sewage sludge into biogas
(c) Conversion of methane hydrates into natural gas
(d) Conversion of toxic gases emitted from an engine into steam
EXPLANATION:
A catalytic converter is a device fixed in the exhaust system of a vehicle(front part) which is close to
the engine and it to reduce the emission of gaseous pollutants, such as carbon monoxide (CO), nitrogen
oxides (NOx) and hydrocarbons (HCs). It uses a chamber called a catalyst to convert the harmful
compounds (toxic gases) from an engine's emissions into safe gases, like steam. So, Option (d) is
correct.
 110
.


ADDITIONAL INFORMATION:

CATALYTIC CONVERTER

About ¾ A catalytic converter is a device that uses a catalyst to convert harmful


Catalytic compounds into harmless ones.
Converter ¾ Catalytic converters have expensive metals such as platinum-palladium and
rhodium as catalysts and are fitted into automobiles to reduce the emission of
poisonous gases.
¾ As the exhaust passes through the catalytic converter.
¾ The exhaust passes through the catalytic converter, unburnt hydrocarbons are
converted into carbon dioxide and water, and carbon monoxide and nitric oxide
are changed to carbon dioxide and nitrogen gas, respectively.
¾ For example, A Catalytic Converter is used in car and it works to split up the
unsafe molecules in the gases that a car produces before they get released into
the air.

Fermentation The conversion of biomass into ethanol is called fermentation. During fermentation,
microorganisms (e.g., bacteria and yeast) metabolize plant sugars and produce
ethanol.
Anaerobic ¾ Biogas is produced by the anaerobic (oxygen-free) digestion of organic materials
digestion such as sewage sludge.
¾ Anaerobic digestion is the biological degradation of organic matter in the absence
of free oxygen.
¾ Biogas is produced from the anaerobic digestion (AD) of organic matter, such as
manure, MSW, sewage sludge, biodegradable wastes, and agricultural slurry,
under anaerobic conditions with the help of microorganisms.

 111
.


Methane ¾ Methane is a clean and economical fuel and estimated that one cubic meter of
Hydrates methane hydrate contains 160-180 cubic meters of methane.
¾ Methane hydrate is formed when hydrogen-bonded water and methane gas come
into contact at high pressures and low temperatures in oceans.

As of September 2020, India has found that the methane hydrate deposits in the
Krishna-Godavari (KG) basin are of biogenic origin.

Biogas ¾ Biogas is a mixture of gases (predominantly methane) produced by microbial


activity and may be used as fuel.
¾ Methanobacterium bacteria are commonly found in anaerobic sludge during
sewage treatment. These bacteria are also present in cattle's rumen (a part of the
stomach).
¾ Thus, the excreta (dung) of cattle, commonly called gobar, is rich in these bacteria.
Dung can be used for biogas generation, commonly called gobar gas.
¾ By-products of wastewater treatment are sludge and biogas.

Q.79) Which of the statements given below about Rakhigarhi Archeological site is/are not correct?
1. It was the largest town and regional trade center of the Indus valley Civilisation.
2. It is located in the Ghaggar-Hakra river plain.
3. Recently it became the first Harappan site to get UNESCO World Heritage Site tag.
Select the correct answer using the code given below :
(a) 1 only
(b) 1 and 2 only
(c) 2 only
(d) 3 only
EXPLANATION:
Rakhigarhi is one of the biggest Harappan civilisation sites spread across 300 hectares in Hisar,
Haryana. It was one of the regional commercial trading centers of the Indus valley civilisation. So,
Statement 1 is correct.

 112
.


Rakhigarhi is now a small village. The site is situated at the center of the Ghaggar-Hakra basin (in the
valley of the now dry course of the Drishadvati River, a tributary of the Saraswati River) in the
Narnaund Tehsil of Hissar district, Haryana. So, Statement 2 is correct.

The Harappan city of Dholavira, present-day Gujarat, was named the 40th Indian site on UNESCO's
World Heritage list. It is the first site of the ancient Indus Valley Civilisation (IVC) in India to get the
tag. So, Statement 3 is not correct.

ADDITIONAL INFORMATION:

INDUS VALLEY CIVILISATION

About ¾ Harappan civilization was discovered in 1920–22 when two of its most
important sites were excavated, Harappa and Mohenjodaro.
¾ Based on the archaeological findings, the Harappan civilization has
been dated between 2600 B.C–1900 BC and is one of the world's oldest
civilizations.
¾ It is also sometimes referred to as the 'Indus Valley civilization' and Indo
-Saraswati civilization.
Early Harappan phase ¾ It was marked by some town-planning in the form of mud structures,
(3500 BC–2600 BC) elementary trade, arts, and crafts, etc.

Mature Harappan ¾ It was the period in which we noticed well-developed towns with burnt
phase (2600 BC–1900 brick structures, inland and foreign trade, crafts of various types, etc.
BC)
Late Harappan phase ¾ It was the phase of decline during which many cities were abandoned,
(1900 BC–1400 BC) and the trade disappeared, leading to the gradual decay of the
significant urban traits

 113
.


Q.80) Consider the following statements about Navigation Systems :


1. IRNSS is a global navigation system developed indigenously in India.
2. GAGAN is a Satellite Based Augmentaion System (SBAS) launched by India to ensure the safety
of civilian aviation by augmenting the IRNSS Signal quality.
3. Ionospere layer of the atmosphere can interfere with radio communication.
Which of the statements given above is/are not correct?
(a) 1 and 2 only
(b) 1 only
(c) 2 and 3 only
(d) 3 only
EXPLANATION:

IRNSS (Indian Regional Navigational Satellite System), popularly known as Navigation with Indian
Constellation (NavIC) is an autonomous regional satellite navigation system developed by Indian
Space Research Organization (ISRO) to implement an independent and indigenous regional
spaceborne navigation system for national applications. The other Global systems are GPS (US),
GLONASS (Russia), BeiDou (China), and Galileo (European Union). Thus, IRNSS is a regional
Navigational System, not a Global Navigation system. So, Statement 1 is not correct.

GAGAN (GPS Aided GEO Augmented Navigation) is a Satellite Based Augmentation System (SBAS)
launched by ISRO and the Airport Authority of India (AAI) to ensure civilian aviation safety by
augmenting the Global Positioning System. India has become the first Asian-Pacific country to develop
such a system for aviation use successfully.

 114
.


It uses a constellation of satellites and a network of ground stations to provide necessary


augmentations to the GPS standard positioning service (SPS) navigation signal for air traffic
management. Thus, GAGAN uses GPS, not IRNSS. So, Statement 2 is not Correct.

The ionosphere is a very active part of the atmosphere, and it grows and shrinks depending on the
energy it absorbs from the Sun. The name ionosphere comes from the fact that gases in these layers
are excited by solar radiation to form ions with an electrical charge.
The ionosphere also plays a role in our everyday communications and navigation systems. Radio and
GPS signals travel through this layer of the atmosphere; the ionosphere is where Earth’s atmosphere
meets space. So, Statement 3 is correct.

Q.81) Consider the following statements regarding a Bacteriophage :


1. It is a bacteria that can integrate its own DNA into DNA of another bacteria.
2. It hijacks the host bacterium's cellular machinery to produce the bacteriophage’s own proteins.
3. Bacteriophages cannot infect and replicate in human cells.
Which of the statements given above is/are correct ?
(a) 1 and 2 only
(b) 2 only
(c) 2 and 3 only
(d) 1, 2 and 3
EXPLANATION:

A bacteriophage is a type of virus that infects bacteria. The word "bacteriophage" means "bacteria
eater," which are so small they do not even have a single cell but are instead just a piece of DNA
surrounded by a protein coat.
Thus, they are viruses, not Bacteria. So, Statement 1 is not correct.

After affecting the Bacteria, the bacteriophage hijacks the bacterium's cellular machinery to prevent it
from producing bacterial components and instead forces the cell to produce viral components. So,
Statement 2 is correct.
 115
.


Bacteriophages can infect and replicate only in bacterial cells. It cannot infect and replicate in human
cells, but Bacteriophages are an important part of the human Microbiome. The reason for the
replication in human cells is yet to be found, and research is going on. So, Statement 3 is correct.

ADDITIONAL INFORMATION:
About ¾ The virus that infects bacteria is called bacteriophage or simply phage.
¾ Phages exhibit two different types of the life cycle.
(i) Virulent or lytic cycle
(ii) Temperate or lysogenic cycle.

Lytic Cycle Intra-cellular multiplication of the phage ends in the lysis of the host bacterium and
the release of progeny virions.
Lysogenic The temperate phages enter into a symbiotic relationship with the host cells. There is
Cycle no death or lysis of the host cells. Once inside the host cell, the temperate phage
nucleic acid integrates with the bacterial genome.

Q.82) Consider the following statements regarding Quarks :


1. Quarks are elementary particles which come up in six flavours such as up, down, charm, strange,
top and bottom.
2. The Large Hadron Collider beauty (LHCb) experiment investigates the slight difference between
the matter and antimatter by studying the "Strange quark" particle.
3. Protons and electrons are elementary particles made up of three different quarks
Which of the statements given above is/are correct ?
(a) 1 and 2 only
(b) 1 only
(c) 2 and 3 only
(d) 1, 2 and 3

 116
.


EXPLANATION:

Quarks are the elementary particles that are the smallest known building blocks of the Universe. The
present standard model has six "flavors" of quarks. They are up, down, charm, strange, top and bottom.
So, Statement 1 is correct.

The Large Hadron Collider beauty (LHCb) experiment specializes in investigating the slight differences
between matter and antimatter by studying a type of particle called the "beauty quark" or "b quark." It
is not by studying the strange quark particle. So, Statement 2 is not correct.

Leptons are elementary particles which carry one unit of electric charge or neutral. Therefore, the
charged leptons are the electrons, muons and taus. Each of these types has a negative charge and a
distinct mass. Electrons, the lightest leptons, have a mass of only 1/1,840 that of a proton. The neutral
Leptons are called Neutrino, which has no electric charge and no significant mass.
On Contrast, Protons and Neutrons are made up of two kinds of quarks, up and down quarks.
A quark is an elementary particle and a fundamental constituent of matter. Protons are composed of
two up quarks and a down quark, while neutrons are composed of two downs and an up. The up-and-
down quarks are the lightest varieties. Because more-massive particles tend to decay into less massive
ones, the up-and-down quarks are also the most common in the Universe; therefore, protons and
neutrons make up most of the matter in the Universe.
Thus, Protons are made up of two different Quarks and electrons are not made up of Quarks.
So, Statement 3 is not correct.

Q.83) Consider the following statements :


1. If a star in a distant galaxy rotates in a direction away from the earth, the red shift occurs when
electromagnetic radiation is emitted by the stars.
2. Doppler effect is used by traffic police to check the over-speeding of vehicles.
3. Volcanoes, Earthquakes and Ocean waves produces infrasonic sound.
Which of the statements given above is/are correct ?
(a) 1 and 2 only
(b) 2 and 3 only
(c) 1 and 3 only
(d) 1, 2 and 3
EXPLANATION:

 117
.


Redshift and blueshift are used by astronomers to work out how far an object is from Earth. As an
object moves away from us (Earth), the sound or light waves emitted by the object are stretched out,
which makes them have a lower pitch and move them toward the red end of the electromagnetic
spectrum, where light has a longer wavelength. In the case of light waves, this is called redshift.
As an object moves towards us (Earth), sound and light waves are bunched up, so the pitch of the
sound is higher, and light waves are moved towards the blue end of the electromagnetic spectrum,
where light has a shorter wavelength. In the case of light waves, this is called blueshift.
Thus if a star rotates in a direction away from the earth, the light from the object (electromagnetic
radiation) occurs in the form of a redshift. So, Statement 1 is correct.

A sound wave or electromagnetic wave of known frequency is sent toward a moving object. Some part
of the wave is reflected from the object and its frequency is detected by the monitoring station. This
change in frequency is called the Doppler shift.
Radar detectors are electronic devices that help detect radio waves or radio signals. These are forms
of electromagnetic energy that are emitted from radars, such as those used by police to locate cars
driving over the speed limit.
For example, if the car moves toward the radar device, the return signal wave frequency increases.
Then, this radar gun can use this change in frequency to determine the speed at which the car is
moving due to the Doppler effect which helps the police to check the speed. So, Statement 2 is
correct.

Normal human ears hear only the frequencies of 20Hz to 20000 Hz. Sounds of less than 20 Hz are
infrasonic waves and sounds of more than 20,000 Hz are called ultrasonic waves.
The term "infrasonic" applied to sound refers to sound waves below the frequencies of audible sound,
and nominally includes anything under 20 Hz. Sources of infrasound in nature include volcanoes,
avalanches, earthquakes, and meteorites. Even ocean storms and waves generate a lot of infrasound.
So, Statement 3 is correct.

Q.84) Macolin Convention, recently in news, is related to :


(a) Cybercrime and data security
(b) Reversing the decline in the health of the world's oceans
(c) Private sector investment in infrastructure development
(d) Manipulation of sports competitions
EXPLANATION:

¾ The Council of Europe Convention on the Manipulation of Sports Competitions, known as the
Macolin Convention, is a multilateral treaty that checks match-fixing.

 118
.


¾ The convention was concluded in Macolin/Magglingen, Switzerland. It came into force on


September 1, 2019.
¾ The 12th meeting of Interpol's Match-Fixing Task Force (IMFTF), in which the Central Bureau of
Investigation was one of the participants, concluded recently with a call for harmonised global
efforts to curb competition manipulation. So, Option (d) is correct.

ADDITIONAL INFORMATION:
INTERPOL

About ¾ The International Criminal Police Organization is inter-governmental. It


has 195 member countries and helps police work together to make the
world safer.
¾ Each member country hosts an INTERPOL National Central Bureau
(NCB). This connects their national law enforcement with other countries
¾ The CBI is designated as the National Central Bureau of India for Interpol.

 119
.


Q.85) Which of the following statements are not correct regarding Monoclonal Antibodies (mAbs) ?
1. They are man-made proteins that act like human antibodies in the immune system.
2. Being highly reactive, mAbs currently have limited scope as they cannot bind to more than one
protein simultaneously.
3. CAR – T cell therapy involves artificially engineered White Blood cells to destroy cancer cells.
Select the correct answer using the code given below :
(a) 1 and 3 only
(b) 2 only
(c) 2 and 3 only
(d) 1, 2 and 3
EXPLANATION:

The body's immune system attacks any foreign substances by making large numbers of antibodies. An
antibody is a protein that sticks to a specific protein called an antigen. Antibodies circulate throughout
the body until they find and attach to the antigen. Once attached, they can force other parts of the
immune system to destroy the cells containing the antigen.
Researchers can design antibodies that specifically target a certain antigen. They can then make many
copies of that antibody in the lab. These are known as monoclonal antibodies (mAbs or Moabs).
Monoclonal antibodies are thus, proteins to stimulate the immune system. And they are man-made
proteins that act like human antibodies. So, statement 1 is correct.

A type of monoclonal antibodies mAbsincludes, bispecific monoclonal antibody (BsMAb, BsAb) which
is an artificial protein that can simultaneously bind to two different types of antigens.
Bispecific antibodies are currently approved for the treatment of leukemias and for cancer
immunotherapy. So, statement 2 is not correct.

 120
.


CAR T cell therapyis a type of cancer immunotherapy treatment ,in which, a patient’s Tcellswhich is a
part of White Blood Cells ,are genetically engineered in the laboratory, so they will bind to specific
proteins(antigens) on cancer cells and kill them. So, statement 3 is correct.

ADDITIONAL INFORMATION:

MONOCLONAL ANTIBODIES (mAbs)

Antibodies ¾ In 1975 the invention of monoclonal antibodies was done by George Kohler and
Cesar Milstein.
¾ Antibodies are protective proteins produced by your immune systemin response
to the presence of a foreign substance called an antigen(a substance that causes
the body to make a specific immune response).
¾ It is also called as immunoglobulin

Bispecific ¾ Bispecific antibodies act as a bridge between cancer cells and cytotoxic T
antibodies in lymphocytes.
cancer ¾ They bind CD3 antigens on CTLs and a specific antigen on cancer cells (e.g. CD19),
treatment which in turn activates the T cells and promotes the lysis of cancer cells.

About ¾ Monoclonal antibodies are identical immunoglobulins (proteins are involved in the
Monoclonal immunological reaction and are therefore called immuno-globulins) generated
Antibodies from a single stimulated B cell clone.
¾ The monoclonal antibodies show specific reactivity to only one epitope (antigenic
determinant, portion of a foreign protein, or antigen capable of stimulating an
immune response) of the antigen and are generally produced artificially.
¾ Deriving from a single clone of dividing cells.

Uses of mAbs ¾ mAbs are currently used to treat cancer.


 121
.


¾ Monoclonal antibodies are used as therapeutic agents in chemotherapy and other


immune diseases.

Blood ¾ Blood is a special connective tissue consisting of a fluid matrix, plasma, and
formed elements.

Plasma ¾ Plasma is a straw coloured, viscous fluid constituting nearly 55 per cent of the
blood. 90-92 per cent of plasma is water and proteins contribute 6-8 per cent of
it
¾ Fibrinogen, globulins and albumins are the major proteins.

Red blood ¾ Erythrocytes or red blood cells (RBC) are the most abundant of all the cells in
cells blood.
¾ They have a red coloured, iron containing complex protein called haemoglobin.

Leucocytes ¾ Leucocytes are also known as white blood cells (WBC) as they are colourless due
to the lack of haemoglobin. They are nucleated and are relatively lesser in number
which averages 6000-8000 mm–3 of blood.

Two main Granulocytes Agranulocytes.


categories of Neutrophils Lymphocytes
WBCs ¾ Neutrophils are the most abundant ¾ Lymphocytes (20-25 per cent)
cells (60-65 per cent) of the total are of two major types – ‘B’ and
WBCs. ‘T’ forms. Both B and T
Eosinophils lymphocytes are responsible
¾ Eosinophils (2-3 per cent) resist for immune responses of the
infections and are also associated body.
with allergic reactions. Monocytes.
Basophils ¾ Monocytes are a type of white
¾ basophils are the least (0.5-1 per blood cell that fights off
cent). bacteria, viruses and fungi.
¾ secrete histamine, serotonin, heparin,
etc., and are involved in inflammatory
reactions

Q.86) Consider the following statements with reference to Electromagnetic waves :


1. In vacuum, all electromagnetic waves have the same speed.
2. Electromagnetic waves can travel through solid materials.
3. Infrared radiation can be used to relieve muscle pain and tension.
Which of the statements given above are correct ?
(a) 1 and 2 only
(b) 2 and 3 only
 122
.


(c) 1 and 3 only


(d) 1, 2 and 3
EXPLANATION:

Properties of Electromagnetic Waves are:


¾ Electromagnetic waves are produced by accelerated charges.
¾ They do not require any material medium for propagation and can travel through any region.
¾ In an electromagnetic wave, the electric and magnetic field vectors are at right angles to each other
and to the direction of propagation. Hence electromagnetic waves are transverse in nature.
¾ The energy in an electromagnetic wave is equally divided between electric and magnetic field
vectors.
¾ They travel in vacuum or free space with a velocity of light (c). [c = 3 x 10 8 m/s]
¾ The electromagnetic waves being chargeless, are not deflected by electric and magnetic fields.
So, Statements 1 and 2 are correct.

Uses of Infra-red waves are:


¾ Infrared lamps are used in physiotherapy [To relieve muscle pain and tension].
¾ Infrared photographs are used in weather forecasting.
¾ As infrared radiations are not absorbed by air, thick fog, mistetc, they are used to take photograph
of long distance objects.
¾ Infra red absorption spectrum is used to study the molecular structure.
So, Statement 3 is correct.

ADDITIONAL INFORMATION

Electromagnetic Radiations

 123
.


Q.87) Consider the following statements regarding Stem cells :


1. Stem cells are special cells that can make copies of themselves and change into many different
kinds of cells.
2. Stem cells in umbilical cord blood transplant can completely cure HIV patients.
Which of the statements given above is/are correct ?
(a) 1 only
(b) 2 only
(c) Both 1 and 2
(d) Neither 1 nor 2
EXPLANATION:

Stem cells are undifferentiated (unspecialized) cells in our body that can undergo cell division (mitosis),
differentiate into specialized cell types, and redivide to produce more stem cells. In adults, stem cells
may be obtained from an embryo, the umbilical cord, and bone marrow. So, Statement 1 is Correct.

 124
.


Recently an HIV-positive woman who received a blood stem cell transplant to treat acute myeloid
leukemia appears to have been cured of HIV. The stem cells obtained from the umbilical cord blood
contain a gene variant that makes them resistant to HIV infection. Hence it is found that Stem cells in
umbilical cord blood transplants can completely cure HIV patients. So, Statement 2 is correct.
ADDITIONAL INFORMATION:

Stem Cell Medical research shows that stem cell therapy can replace tissues damaged due to
Technology human disease. Several adult stem cell therapies, such as bone marrow transplants to
treat blood cancer, exist.
Stem cells have potential uses as given below:
x To replace damaged tissues
x To study human development
x To test new drugs
x To devise methods of gene therapy

AIDS Human immunodeficiency virus or HIV is an infection that attacks the immune system
by destroying the body’s immune cells called CD4, which help it respond to infection.
Once HIV attacks the CD4 cells, it starts replicating and destroying them, weakening
the body’s immune system and making it more prone to certain “opportunistic
infections” that take advantage of the weak immune system.
Transmission :
¾ Unprotected sex,
¾ Transfusion of contaminated blood,
¾ Sharing needles and syringes,
¾ From a mother with HIV to her infant during pregnancy.

Q.88) Consider the following statements regarding the Dark Energy :


1. As per Einstein’s Theory of Relativity, "empty space" can possess its own energy.
2. As more space comes into existence with time, energy gets diluted and expansion of the
universe will be decelerated
 125
.


3. The effect of gravity supports the effect of dark energy in the expansion of the universe.
Which of the statements given above is/are not correct ?
(a) 1 only
(b) 1 and 3 only
(c) 2 and 3 only
(d) None of the above
EXPLANATION:

Dark energy, the mysterious form of energy that makes up about 68% of the universe, has intrigued
physicists and astronomers for decades. It has been noted as “the most profound mystery in all of
science”. Albert Einstein was the first person to realize that empty space is not nothing. According to
his‘ Theory of Relativity’, the first property that he discovered is that it is possible for more space can
come into existence, and also his second prediction is that "empty space" can possess its own energy.
So, Statement 1 is correct.

As this energy is a property of space itself, it would not be diluted as space expands. As more space
comes into existence, more of this energy-of-space would appear. As a result, this form of energy would
cause the universe to expand faster and faster and cannot be decelerated. So, Statement 2 is not
correct.
As the Universe expands, gravity as an attractive force pulls on the universe and slows down the
expansion over time. But due to the effect of Dark energy, the Universe expands faster as this energy
is a part of space itself. Hence, the effect of gravity does not support the effect of dark energy in the
expansion of the universe. So, Statement 3 is not correct.

 126
.


ADDITIONAL INFORMATION:

DARK MATTER

About Dark matter is a form of invisible matter or mass exists in space only. Dark matter slows
down the universe’s expansion. Dark matter is ideal for the co-existence of galaxies and the
sustainability of the universe.
DARK ENERGY

About ¾ Dark Energy is a hypothetical form of energy that exerts a negative, repulsive pressure,
behaving like the opposite of gravity.
¾ It has been hypothesized to account for the observational properties of distant-type
supernovas, which show the universe going through an accelerated period of expansion.
¾ Like Dark Matter, Dark Energy is not directly observed, but rather inferred from
observations of gravitational interactions between astronomical objects.
¾ It makes up 72% of the total mass-energy density of the universe. The other dominant
contributor is Dark Matter, and a small amount is due to atoms or baryonic matter.

Q.89) With regard to Kashmiri culture, the term “Raad” refers to :


(a) Kahwa tea
(b) Handicraft fair
(c) Folk dance
(d) Floating garden

 127
.


EXPLANATION:

¾ Recently, under Azadi Ka Amrit Mahotsav under the Mission Youth, the government has decided
to develop five tourist villages within Dal Lake in Srinagar which included Kachri Mohalla, Sofi
Mohalla, TindaMohallah, Akhoon Mohalla, and a vegetable market. These villages will have all the
basic facilities for inhabitants as well as tourists and local visitors.
¾ Dal Lake, the urban lake, is integral to tourism and recreation in Kashmir and is named the “Jewel
in the crown of Kashmir” or “Srinagar’s Jewel”. The lake is also an important source for commercial
operations in fishing and water plant harvesting.
¾ The lake covers an area of 18 square kilometers (6.9 sq mi) and is part of a natural wetland, which
covers 21.1 square kilometers (8.1 sq mi), including its floating gardens. The floating gardens,
known as “Raad” in Kashmiri, blossom with lotus flowers during July and August. The wetland is
divided by causeways into four basins — Gagribal, Lokut Dal, Bod Dal, and Nagin. So, Option (d)
is correct.

Q.90) With reference to the Immune system of the Human body, consider the following statements:
1. WBCs are biconvex and nucleated cells in the blood.
2. WBCs have a longer life span than red blood cells.
3. Monocytes are primarily associated with Hyper-sensitivity reactions in Human Body.
Which of the statements given above is/are not correct ?
(a) 1 only
(b) 1 and 2 only
(c) 2 and 3 only
(d) 1, 2 and 3
EXPLANATION:

Leucocytes are also known as white blood cells (WBC) as they are colorless due to the lack of
hemoglobin. They are nucleated cells and heterogeneous in nature. The normal concentration of white
blood cells in the blood varies from 4000 and 10,000 per micro-liter and they are irregularly shaped
cells.
Whereas Platelets are small, irregularly shaped clear cell fragments in circulating blood and they have
a biconvex disc structure with an equatorial diameter of 2–3 μm and are anucleate. So, Statement
1 is not correct.

Leucocytes are generally short-lived, that is for about 13 to 20 days. But the RBCs have an average
life span of 120 days after which they are destroyed in the spleen (the graveyard of RBCs). So,
Statement 2 is not correct.

The two main categories of WBCs – are granulocytes and agranulocytes. Neutrophils, eosinophils, and
basophils are different types of granulocytes, while lymphocytes and monocytes are agranulocytes.
In that, Eosinophils (2-3 percent) resist infections, also associated with allergic and Hypersensitivity
reactions. While Monocytes are involved in phagocytosis and degradation of necrotic material. So,
Statement 3 is not correct.

 128
.


ADDITIONAL INFORMATION:
BLOOD

RBC ¾ In human males, there are 5 million red blood cells per cubic millimeter, and in
females, -4.5 to 5 million. Erythrocytes are packed with hemoglobin molecules to
transport oxygen to the tissues.
¾ In a single red blood corpuscle, nearly 280 million (2.8 X lo8) molecules of hemoglobin
are packaged. Hemoglobin concentrations in humans range from 14 to 16% in males
and 12 to 14% in females.
¾ Concentrations less than this amount are indicative of anemia. Anaemia essentially
results due to iron deficiency in the body. Iron or heme is the prosthetic group of the
hemoglobin molecule.

WBC ¾ White blood cells, leucocytes, are cells with various shaped nuclei. Leucocytes average
5-9 thousand per cubic millimeter in normal blood. The count of children is higher.
¾ White blood cells are one of the body's major defense mechanisms against bacteria,
viruses, and other foreign substances that have entered the bloodstream. During
infections, the number of WBCs increases, a parameter used to diagnose infections.
Platelets ¾ Platelets also called thrombocytes, are cell fragments produced from megakaryocytes
(special cells in the bone marrow). Blood normally contains 1,500,00-3,500,00
platelets mm–3.
¾ Platelets can release a variety of substances most of which are involved in the
coagulation or clotting of blood. A reduction in their number can lead to clotting
disorders which will lead to excessive loss of blood from the body.
Q.91) Consider the following statements regarding International Liquid Mirror Telescope (ILTM) :
1. The ILTM is a Space Telescope placed at Lagrangian Point 2 which is 1.5 mn km away from earth.
2. In ILTM, Mercury is used as the reflective liquid which is protected from wind by a thin
transparent film of mylar.
 129
.


3. They cannot be turned and pointed in any direction.


Which of the statements given above is/are correct ?
(a) 1 only
(b) 2 only
(c) 2 and 3 only
(d) 1, 2 and 3
EXPLANATION:

A unique International Liquid-Mirror Telescope (ILMT) has recently been installed at the Devasthal
Observatory campus of the Aryabhatta Research Institute of Observational Sciences (ARIES), an
autonomous institute of the Department of Science and Technology (DST), Government of India, in
Nainital, Uttarakhand. India's first liquid mirror telescope observes asteroids, supernovae, space
debris, and all other celestial objects from an altitude of 2450 meters in the Himalayas. Thus, it is not
placed at the Lagrangian Point. So, Statement 1 is not correct.

The ILMT is made up of three components:


¾ A primary mirror is formed by a rotating container with a highly reflective liquid (mercury)
protected from the wind by a thin transparent film of mylar.
¾ An air compressor operates an air bearing on which theLiquid Mirrorsites.
¾ A drive system. So, Statement 2 is correct.

The ILMT is entirely dedicated to a photometric/Astrometric direct imaging survey. The liquid mirror
telescopes cannot be tilted, so they cannot track like conventional telescopes.
So, Statement 3 is correct.

ADDITIONAL INFORMATION:

Liquid Mirror Telescope

¾ A rotating liquid surface takes the shape of a paraboloid under the constant pull of gravity and
centrifugal acceleration, which grows stronger with the distance from the central axis.
¾ The parabolic shape occurs because a liquid surface always sets its local surface perpendicular to
the net acceleration it experiences. In this case, it becomes stronger and more inclined with distance
from the central axis.
¾ Consequently, a Liquid Mirror Telescope can be made by using a liquid mirror as the primary mirror
and putting a camera at the focal point of the paraboloid.
¾ Liquid mirror telescopes are Zenithal pointing telescopes: they can only see a small field of view
around the Zenith (point along the local vertical direction). Pointing objects, as done when using
classical technology telescopes, is no longer possible.
¾ Integrating images with LMTs is done using a new real-time imaging technique called Time Delayed
Integration (TDI) (described below).
¾ Thanks to the Earth's rotation, the telescope scans a strip of constant declination equal to the
observatory's latitude. Because an LMT observes the same region of the sky night after night, it is
possible either to co-add the images taken on different nights to improve the limiting magnitude
attainable with an LMT or to subtract images taken on different nights to make a variability study
of the corresponding strip of sky.
¾ Consequently, an LMT is well suited to perform photometric variability studies of the strip of the
sky it observes.

 130
.


Q.92) With reference to Collective Security Treaty Organization (CSTO), consider the following
statements :
1. It is an intergovernmental military alliance signed among the Latin American countries.
2. It is formed under the framework of commonwealth of Independent states.
3. The headquarters is located in Mexico.
Which of the statements given above is/are correct ?
(a) 1 only
(b) 2 only
(c) 3 only
(d) 1, 2 and 3 only
EXPLANATION:

Collective Security Treaty Organization (CSTO) is an intergovernmental military alliance established as


a Treaty among Armenia, Kazakhstan, Kyrgyzstan, Russia, Tajikistan, and Uzbekistan on May 15,
1992. Three other post-Soviet states—Azerbaijan, Belarus, and Georgia—signed in 1993, so the treaty
took effect in 1994 and it was converted into the Organization in 2002. It promotes cooperation in
military and security spheres among its member-states. Hence, this organization is among the Central
Asian countries and not between the Latin American countries. So, Statement 1 is not correct.

The Collective Security Treaty Organization was formed under the framework of the Commonwealth of
Independent States and serves as a mutual defense alliance among its members. This organization is
headquartered in the Russian capital of Moscow. So, Statement 2 is correct and Statement 3 is not
correct.

ADDITIONAL INFORMATION:

COLLECTIVE SECURITY TREATY ORGANISATION (CSTO)

 131
.


Recently ¾ Recently, it celebrated its 20th year anniversary


in news

About ¾ The Treaty also provides a mechanism for consultations in case of a threat to the
security, territorial integrity, and sovereignty of one or more member-states or a threat
to international peace and security.
¾ The Charter of the Organization was adopted on October 7, 2002.
¾ The CSTO received the status of observer at the United Nations General Assembly on
December 2, 2004.

Collective ¾ It is the highest body of the Organization and consists of heads of the Member States.
Security ¾ It considers the fundamental issues of the Organization’s activities and makes
Council decisions aimed at the realization of its goals and objectives, as well as provides
coordination and joint activities of the Member States for the realization of these goals.
¾ Sessions of the CSC are held alternately in the Member States as necessary, but at
least once a year.
¾ The chairmanship of the Council is transferred in the order of the Russian alphabet
unless the Council decides otherwise.

Q.93) Erythroblastosis Foetalis is a condition in which second baby of the couple may die in the womb
of the mother due to Rhesus (Rh) incompatibility. What may be the probable combination of the
blood typology of parents ?
(a) Father having Rh+ve and Mother having Rh -ve
(b) Father having Rh+ve and Mother having Rh +ve
(c) Father having Rh-ve and Mother having Rh -ve
(d) Father having Rh-ve and Mother having Rh +ve

EXPLANATION:

¾ The Rh factor is a molecule on the surface of red blood cells in some people. Blood is Rh-
positive if red blood cells have the Rh factor and Rh-negative if they do not.

 132
.


¾ An Rh-ve person, if exposed to Rh+ve blood, will form specific antibodies against the Rh antigens.
Therefore, the Rh group should also be matched before transfusions. A special case of Rh
incompatibility (mismatching) has been observed between the Rh-ve blood of a pregnant mother
and with Rh+ve blood of the foetus.
¾ Rh antigens of the foetus do not get exposed to the Rh-ve blood of the mother in the first
pregnancy as the two types of blood are well separated by the placenta. However, during the
delivery of the first child, there is a possibility of exposure of the maternal blood to small amounts
of the Rh+ve blood from the foetus. In such cases, the mother starts preparing antibodies against
Rh antigen in her blood.
¾ In the case of her subsequent pregnancies, the Rh antibodies from the mother (Rh-ve) can leak
into the blood of the foetus (Rh+ve) and destroy the foetal RBCs. This could be fatal to the foetus
or could cause severe anaemia and jaundice in the baby. This condition is called erythroblastosis
foetalis. This can be avoided by administering anti-Rh antibodies to the mother immediately after
the delivery of the first child.
¾ Erythroblastosis fetalis classically results from Rho(D) incompatibility, which may develop
when a woman with Rh-negative blood is impregnated by a man with Rh-positive blood and
conceives a fetus with Rh-positive blood, sometimes resulting in hemolysis. So, Option (a) is
correct.

Q.94) The State of Inequality in India Report was released by :


(a) NITI Aayog
(b) RBI
(c) Economic Advisory Council to the Prime Minister
(d) Ministry of Finance
EXPLANATION:

The State of Inequality in India Report was released by Economic Advisory Council to the Prime
Minister (EAC-PM). The report has been written by the Institute for Competitiveness and presents a
holistic analysis of the depth and nature of inequality in India. The report compiles information on
inequities across sectors of health, education, household characteristics, and the labour market. As
the report presents, inequities in these sectors make the population more vulnerable and trigger a
descent into multidimensional poverty. So, Option (c) is correct.

ADDITIONAL INFORMATION:

STATE OF INEQUALITY IN INDIA REPORT

Recently ¾ A recent report titled ‘The State of Inequality in India’ was released by the Institute
in news for Competitiveness, brought out in response to the request of the Economic Advisory
Council to the Prime Minister (EAC-PM).
Highlights ¾ According to the report, education and household conditions have improved
enormously due to targeted efforts through several social protection schemes,
especially in the area of water availability and sanitation that have increased the
standard of living. It is emphasized that education and cognitive development from
the foundational years is a long-term corrective measure for inequality.
¾ By 2019-20, 95% of schools have functional toilet facilities on the school premises
(95.9% functional boy’s toilets and 96.9% functional girl’s toilets). 80.16% of schools
 133
.


have functional electricity connections with States and Union Territories like Goa,
Tamil Nadu, Chandigarh, Delhi, Dadra and Nagar Haveli and Daman and Diu,
Lakshadweep, and Puducherry have achieved universal (100%) coverage of functional
electricity connections.
¾ The Gross Enrolment Ratio has also increased between 2018-19 and 2019-20 at the
primary, upper primary, secondary, and higher secondary. In terms of improvement
in household conditions, emphasis on providing access to sanitation and safe
drinking water has meant leading a dignified life for most households.

Q.95) With reference to Aqua regia, consider the following statements :


1. It is a mixture of Nitric acid and Sulphuric acid.
2. It is known as royal water.
3. It can dissolve gold.
Which of the statements given above is/are correct ?
(a) 1 and 2 only
(b) 2 only
(c) 2 and 3 only
(d) 3 only
EXPLANATION:

Aqua regiais a solution comprised of a 3:1 mixture of hydrochloric acid (HCL) and nitric acid (HNO3).
So, Statement 1 is not correct.

Aqua regia is a Latin word known as royal water. So, Statement 2 is correct.

It is commonly used to remove noble metals such as gold, platinum and palladium from substrates,
particularly in microfabrications and microelectronics labs. It is one of the few reagents that can
dissolve gold and platinum. Aqua regia solutions are extremely corrosive and may result in explosion
or skin burns if not handled with extreme caution. So, Statement 3 is correct.

ADDITIONAL INFORMATION:
NOBLE METALS AND NOBLE GAS

Noble metals ¾ The noble metals are a group of metals that resist oxidation and corrosion in moist
air.
¾ The noble metals are not easily attacked by acids. They are the opposite of the base
metals, which more readily oxidize and corrode.
¾ There is more than one list of noble metals. The following metals are considered
noble metals (listed in order of increasing atomic number)
ƒ Ruthenium
ƒ Rhodium
ƒ Palladium
ƒ Silver
ƒ Osmium
ƒ Iridium
ƒ Platinum
ƒ Gold
 134
.


¾ Sometimes mercury is listed as a noble metal. Other lists include rhenium as a


noble metal.
¾ Strangely, not all corrosion-resistant metals are considered to be noble metals. For
example, although titanium, niobium and tantalum are extremely corrosion-
resistant, they are not noble metals.
¾ While acid resistance is a quality of noble metals, there are difference in how the
elements are affected by acid attack.
¾ Platinum, gold, and mercury dissolve in the acid solution aqua regia, while iridium
and silver do not.
¾ Palladium and silver dissolve in nitric acid. Niobium and tantalum resist all acids,
including aqua regia.
Noble gas ¾ The elements in Group 18 have their valence shell orbitals completely filled and,
therefore, react with only a few elements under certain conditions. Therefore, they
are now known as noble gases.
¾ Group 18 consists of six elements: helium, neon, argon, krypton, xenon and radon.
All these are gases and chemically unreactive.

Q.96) With reference to the Law Commission of India, consider the following statements :
1. It is a non - statutory body constituted by the notification of the Ministry of Law and Justice.
2. Its recommendations are only advisory in nature.
3. The first such Law commission was established during the period of Lord Metcalf.
4. National Green Tribunal Act, 2010 was established based on the recommendation of the
commission.
Which of the statements given above is/are correct ?
(a) 1 and 3
(b) 2 and 3 only
(c) 2 and 4 only
(d) 1, 2 and 4

 135
.


EXPLANATION:

Law Commission of India is a non-statutory body, in fact, it is an executive body that is constituted by
a notification of the Government of India, Ministry of Law & Justice, and Department of Legal Affairs
with a definite term of reference to carry out research in the field of law and the Commission makes
recommendations to the Government (in the form of Reports) as per its terms of reference. So,
Statement 1 is correct.

This commission is constituted by the Government from time to time. It was originally constituted in
1955 and is reconstituted every three years. The recommendations of the commission are not binding
on the government, it is only advisory in nature. So, Statement 2 is correct.

The first Law Commission was established during colonial rule in India, by the East India
Company under the Charter Act of 1833, and was set up by Lord Macaulay. During that period, the
Governor General was Lord William Bentinck and not Lord Metcalf. After that, three more Commissions
were established in pre-independent India.
Law Commission was assigned the following tasks:
1. Codification of penal law;
2. The law applicable to non-Hindus and non-Muslims in respect of
their various rights;
3. Codification of civil and criminal procedural law etc, etc.
So, Statement 3 is not correct.

By following the recommendations of the commission, various reforms were made to the Indian
administration,
¾ Recommendations made in Report no 165 resulted in the enactment of The Right of Children to
Free and Compulsory Education Act, 2009.
¾ National Green Tribunal was setup as per recommendations of the Supreme Court, Law
Commission and India’s international law obligations to develop national laws on environment and
implement them effectively. So, Statement 4 is correct.

ADDITIONAL INFORMATION:

LAW COMMISSION OF INDIA

Recently in ¾ The 22nd law commission is without a chairperson and members since its
news inception in February 2020

Post ¾ After the commencement of the Government of India Act, of 1919 the legislative
Independence power was exercised by the Indian Legislature constituted there. This Act was
Developments followed by the Government of India Act, of 1935.
¾ With the passing of the Indian Independence Act, of 1947, India became a
Dominion and the Dominion Legislature made laws from 1947 to 1949 under the
provisions of section 100 of the Government of India Act 1935 as adapted by the
India (Provisional Constitution) Order 1947. Under the Constitution of India
which came into force on the 26th of January 1950, the legislative power is vested
in Parliament and the State Legislatures.
¾ The Central Government established the First Law Commission in 1955 with the
then Attorney-General of India, Mr. M. C. Setalvad, as its Chairman.

 136
.


Achievements Based on the various recommendations of this commission, various legislative


measures were made and they are,
¾ Religious Endowments Act 1863,
¾ Official Trustees Act 1864,
¾ Carriers Act 1865, Indian Companies Act 1866,
¾ General Clauses Act 1868,
¾ Divorce Act 1869,
¾ Court fees Act 1870,
¾ Land Acquisition Act 1870,
¾ Female infanticide prevention Act 1870,
¾ code of criminal procedure revised 1872
¾ Indian Contract Act 1872,
¾ Indian Evidence Act 1872,
¾ Special Marriages Act 1872, etc.

Q.97) With reference to Excretory organs consider the following statements :


1. Skin is one of the excretory organs of the human being.
2. Adrenal gland is situated on the top of the kidney.
3. Digestion of protein in the intestine produces ammonia which is processed into urea in the liver.
Which of the statements given above is/are correct ?
(a) 1 only
(b) 2 and 3 only
(c) 1 and 2 only
(d) 1, 2 and 3
EXPLANATION:

Our body needs to excrete some harmful elements and waste. Our body requires some mineral salts
to be thrown out in the form of sweating by one of the excretory organs, the skin. So, Statement 1 is
correct.
The adrenal glands play a vital role in the body's fight-or-flight response. They generate stress
hormones that activate physiological adaptations necessary to counteract changes in the external
environment. There are two triangular adrenal glands located on the top of each kidney. So,
Statement 2 is correct.

 137
.


Ammonia (NH3) is a waste product bacteria in the intestines primarily make when digesting protein.
Normally, ammonia is processed in the liver, where it's transformed into another waste product called
Urea. The Urea is then carried to kidneys, where it's eliminated in the urine. So, Statement 3 is
correct.

ADDITIONAL INFORMATION:

EXCRETORY SYSTEM

About ¾ Every cell of our body keeps on doing something or the other. Therefore, some
waste is always found there.
¾ These elements are harmful and should not be collected in the body. It should
be thrown out of the human body.
¾ The process of throwing out harmful or waste material is called excretion,
and the groups of organs related to it are called the excretion system.

Types Our body excretes or excretion by three types


¾ Lungs: we throw out polluted carbon dioxide produced during breathing.
¾ Skin: some mineral salts are required by our body and thrown out in the form
of sweating.
¾ Urine: Our body comprises water, and a balance of water is required in the
human body. It should not be more or less. Urine takes chemical wastes and
extra water out of our bodies.

 138
.


Q.98) Consider the following rivers :


1. Aruna river
2. Tamor
3. Indrawati
4. Sunkoshi
Which of the above is/are tributaries of Kosi river?
(a) 1 and 2 only
(b) 3 and 4 only
(c) 4 only
(d) 1, 2, 3 and 4
EXPLANATION:
The Kosi was called Kausika in Rig-Veda and is situated in Nepal and Bihar in northern India. It is a
major tributary of the Ganges. This river is mentioned in the epic 'Mahabharata' as Kausiki. The Kosi
River drains the northern slopes of the Himalayas in the Tibet Autonomous Region and the southern
slopes of Nepal. It is the largest river basin of Nepal. Indrawati, Sun Koshi, TamaKoshi, Likhu,
DudhKoshi, Arun, and Tamor are the major seven tributaries of the Koshi river system. So, Option (d)
is correct.

ADDITIONAL INFORMATION:

KOSI RIVER

Recently ¾ A recent case study reveals that River Kosi became very unstable after embankments
in news were made on either side of it

About ¾ Koshi river system is a transboundary river originating from Tibetan Plateau, crosses
the Himalayas, and flows through the Mahabharat range and Siwalik hills, reaching
the plains of eastern Nepal and finally meeting the Ganges in India.

 139
.


¾ Koshi river system drains about 45% area out of 87,970 sq. Km in Nepal Along with
these river tributaries, the Koshi basin comprises about 845 glaciers and 599 glacial
lakes towards the North.
¾ The average flow of the Koshi river at the confluence is around 1500 m3/s (recorded at
Chatara station). These seven tributaries meet at Triveni, from where it is called Sapta-
Koshi. Precipitation in this basin is influenced by the summer monsoon, with 80%
annual precipitation.

Q.99) With reference to Foreign Contribution Regulation Act (FCRA), consider the following statements:
1. FCRA is implemented by the Ministry of External Affairs.
2. FCRA regulates foreign donations and ensures that such contributions do not adversely affect
the internal security of the country.
3. Once granted, FCRA registration is valid for ten years.
Which of the statements given above are not correct ?
(a) 1 and 2 only
(b) 2 and 3 only
(c) 1 and 3 only
(d) 1, 2 and 3 only
EXPLANATION:

The Foreign Contribution Regulation Act is implemented by Ministry of Home Affairs. So, Statement
1 is not correct.

The FRCA is an act to consolidate the law to regulate the acceptance and utilisation of foreign
contribution or foreign hospitality by certain individuals or associations or companies. It also prohibits
the acceptance and utilisation of foreign contribution or foreign hospitality for any activities
detrimental to the national interest and for matters connected with it. So, statement 2 is correct.
The registration once granted by the central government shall be valid for a period of five years. So,
Statement 3 is not correct.

 140
.


Q.100) With reference to Kanheri caves, consider the following statements :


1. Kanheri cave and Tulsi lake are located inside the Protected Area of Sanjai Gandhi National
Park.
2. The cave belong to both the Hinayana and Mahayana phases of Buddhist architecture.
3. The earliest reference of Kanheri is found in Chinese Traveller Fa-Hein Accounts, who visited
India during 399-411 CE.
Which of the statements given above are correct ?
(a) 1 and 2 only
(b) 2 and 3 only
(c) 1 and 3 only
(d) 1, 2 and 3
EXPLANATION:

Kanheri Caves is located deep inside the Sanjay Gandhi National Park. It is an 87 km² protected area
in Mumbai, Maharashtra, with the Tulsi (second largest lake of Mumbai) and Vihar lakes inside it. So,
Statement 1 is correct.
Fa-Hien visited India during the reign of Chandra Gupta. He became the first traveller from China who
visit India. Kanheri was mentioned in the travelogues of foreign travellers. The earliest reference to
Kanheri is ascribed to Fa-Hein, who visited India during 399-411 CE. The Kanheri caves comprise
more than 110 different rock-cut monolithic excavations, one of the largest single excavations
undertaken during the Hinayana phase of Buddhism and several Mahayana stylistic architecture. And
it contains a few printings of the Vajrayana order. So, Statements 2 and 3 are correct.

ADDITIONAL INFORMATION:

Tulsi ¾ Tulsi Lake is a fresh water lake in northern Mumbai on the Salsette Island.
Lake ¾ The lake was formed as a result of controlling Tasso River.

Vihar Vihar Lake is located near Vihar village on the Mithi River within the precincts (a part of
lake a territory with definite bounds) of the Borivali National Park, also called the Sanjay
Gandhi National Park, in North Mumbai.

 141

You might also like